UP Board Solutions for Class 12 Maths Chapter 6 Application of Derivatives

UP Board Solutions for Class 12 Maths Chapter 6 Application of Derivatives (अवकलज के अनुप्रयोग) are part of UP Board Solutions for Class 12 Maths. Here we have given UP Board Solutions for Class 12 Maths Chapter 6 Application of Derivatives (अवकलज के अनुप्रयोग)

Board UP Board
Textbook NCERT
Class Class 12
Subject Maths
Chapter Chapter 6
Chapter Name Application of Derivatives
Exercise Ex 6.1, Ex 6.2, Ex 6.3, Ex 6.4, Ex 6.5
Number of Questions Solved 109
Category UP Board Solutions

UP Board Solutions for Class 12 Maths Chapter 6 Application of Derivatives

प्रश्नावली 6.1

प्रश्न 1.
वृत्त के क्षेत्रफल के परिवर्तन की दर इसकी त्रिज्या r के सापेक्ष ज्ञात कीजिए, जबकि
(a) r = 3 सेमी है
(b) r = 4 सेमी है।
हल-
(a) माना वृत्त का क्षेत्रफल A है, तब
UP Board Solutions for Class 12 Maths Chapter 6 Application of Derivatives image 1
अत: क्षेत्रफल के परिवर्तन की दर 6π सेमी²/सेकण्ड है।
(b) उपरोक्त की भाँति स्वयं हल कीजिए।[उत्तर : 8π सेमी²/से]

UP Board Solutions

प्रश्न 2.
एक घन का आयतन 9 सेमी3/से की दर से बढ़ रहा है। यदि इसकी कोर की लम्बाई 10 सेमी है तो इसके पृष्ठ का क्षेत्रफल किस दर से बढ़ रहा है?
हल-
माना घन की कोर = x सेमी, घन का आयतन = V तथा पृष्ठ क्षेत्रफल = S
तब V = x3 तथा S = 6x2. जहाँ x समय t को फलन है।
UP Board Solutions for Class 12 Maths Chapter 6 Application of Derivatives image 2
अतः पृष्ठ क्षेत्रफल 3.6 सेमी²/से की दर से बढ़ रहा है।

प्रश्न 3.
एक वृत्त की त्रिज्या समान रूप से 3 सेमी/से की दर से बढ़ रही है। ज्ञात कीजिए की वृत्त का क्षेत्रफल किस दर से बढ़ रहा है जब त्रिज्या 10 सेमी है?
हल-
मानी वृत्त की त्रिज्या r सेमी है, तब वृत्त का क्षेत्रफल A = πr² सेमी²
प्रश्नानुसार, [latex ]\frac { dr }{ dt }=3[/latex] सेमी/से …(i)
UP Board Solutions for Class 12 Maths Chapter 6 Application of Derivatives image 3
अत: क्षेत्रफल के परिवर्तन की दर 60π सेमी²/सेकण्ड है।

प्रश्न 4.
एक परिवर्तनशील घन का किनारा 3 cm/s की दर से बढ़ रहा है घन का आयतन किस दर से बढ़ रहा है जबकि किनारा 10 cm लम्बा है?
हल-
माना घन का आयतन = V तथा भुजा = a है, तब V = a3
ज्ञात है
[latex ]\frac { da }{ dt }=3[/latex] सेमी/से, a = 10 सेमी
∴ समय के सापेक्ष आयतन के परिवर्तन की दर
UP Board Solutions for Class 12 Maths Chapter 6 Application of Derivatives image 4
अतः जब घन का किनारा 10 cm लम्बा हो तब घन का आयतन 900 cm2/s की दर से बढ़ रहा है।

प्रश्न 5.
एक स्थिर झील में एक पत्थर डाला जाता है और तरंगें वृत्तों में 5 सेमी/से की गति से चलती है। जब वृत्ताकार तरंग की त्रिज्या 8 सेमी है तो उस क्षण घिरा हुआ क्षेत्रफल किस दर से बढ़
हल-
दिया है- [latex ]\frac { dr }{ dt }=5[/latex] सेमी/से, r = 8 सेमी
माना तरंगों से बने वृत्त का क्षेत्रफल A सेमी² है।
तब A = πr²
UP Board Solutions for Class 12 Maths Chapter 6 Application of Derivatives image 5
अतः जब तरंग की त्रिज्या 8 सेमी हो तब तरंगों द्वारा घिरा हुआ क्षेत्रफल 80 π सेमी²/से की दर से बढ़ रहा है।

UP Board Solutions

प्रश्न 6.
एक वृत्त की त्रिज्या 0.7 सेमी/से की दर से बढ़ रही है। इसकी परिधि की वृद्धि की दर क्या है। जब r = 4.9 सेमी है?
हल-
माना वृत्त की त्रिज्या r सेमी है, तब परिधि C = 2πr
प्रश्नानुसार,
UP Board Solutions for Class 12 Maths Chapter 6 Application of Derivatives image 6
अत: वृत्त की परिधि 1.4π सेमी/से की दर से बढ़ रही है।

प्रश्न 7.
एक आयत की लम्बाई x, 5 सेमी/मिनट की दर से घट रही है और चौड़ाई y, 4 सेमी/मिनट की दर से बढ़ रही है। जब x = 8 सेमी और y = 6 सेमी है। तब आयत के
(a) परिमाप
(b) क्षेत्रफल के परिवर्तन की दर ज्ञात कीजिए।
हल-
ज्ञात है- [latex ]\frac { dx }{ dt }=5[/latex] सेमी/मिनट
तथा [latex ]\frac { dy }{ dt }=4[/latex] सेमी/मिनट
माना आयत का क्षेत्रफल = A सेमी², परिमाप = p सेमी
लम्बाई = x सेमी, चौड़ाई = y सेमी
(a) परिमाप p = 2(x + y)
UP Board Solutions for Class 12 Maths Chapter 6 Application of Derivatives image 7
अत: आयत का क्षेत्रफल 2 सेमी2/सेमी की दर से बढ़ रहा है।

प्रश्न 8.
एक गुब्बारा जो सदैव गोलाकर रहता है, एक पम्प द्वारा 900 सेमी3/सेकण्ड की दर से फुलाया जाता है। गुब्बारे की त्रिज्या के परिवर्तन की दर ज्ञात कीजिए जब त्रिज्या 15 सेमी है।
हल-
माना गुब्बारे की त्रिज्या = r तथा आयतन = V
UP Board Solutions for Class 12 Maths Chapter 6 Application of Derivatives image 8
UP Board Solutions for Class 12 Maths Chapter 6 Application of Derivatives image 9

प्रश्न 9.
एक गुब्बारा जो सदैव गोलाकार रहता है कि त्रिज्या परिवर्तनशील है। त्रिज्या के सापेक्ष आयतन के परिवर्तन की दर ज्ञात कीजिए जब त्रिज्या 10 सेमी है।
हल-
माना गुब्बारे का आयतन = V तथा त्रिज्या = r
UP Board Solutions for Class 12 Maths Chapter 6 Application of Derivatives image 10
अतः जब त्रिज्या 10 सेमी हो तब गुब्बारे का आयतन 400 π सेमी3/सेमी की दर से बढ़ता है।

प्रश्न 10.
एक 5 मी लम्बी सीढी दीवार के सहारे झुकी है। सीढ़ी का नीचे का सिरा जमीन के अनुदिश दीवार से दूर 2.0 मी/से की दर से खींचा जाता है। दीवार पर इसकी ऊँचाई किस दर से घट रही है जबकि सीढ़ी को नीचे का सिरा दीवार से 4 मी दूर है?
हल-
माना दीवार OC है तथा किसी क्षण सीढ़ी AB की स्थिति इस प्रकार है कि OA = x और OB = y
UP Board Solutions for Class 12 Maths Chapter 6 Application of Derivatives image 11
अत: दीवार पर सीढ़ी की ऊँचाई 8/3 मी/से की दर से घट रही है।

प्रश्न 11.
एक कण वक्र 6y = x3 + 2 के अनुगत गति कर रहा है। वक्र पर उन बिन्दुओं को ज्ञात कीजिए जबकि x निर्देशांक की तुलना में y निर्देशांक 8 गुना तीव्रता से बदल रहा है।
हल-
दिया है-
6y = x3 + 2 और [latex ]\frac { dy }{ dt } =8\frac { dx }{ dt } [/latex]
t के सापेक्ष अवकलन करने पर,
UP Board Solutions for Class 12 Maths Chapter 6 Application of Derivatives image 12
UP Board Solutions for Class 12 Maths Chapter 6 Application of Derivatives image 13

प्रश्न 12.
हवा के बुलबुले की त्रिज्या, [latex ]\frac { 1 }{ 2 }[/latex] सेमी/सेकण्ड की दर से बढ़ रही है। बुलबुले का आयतन किस दर से बढ़ रहा है जबकि त्रिज्या 1 सेमी है?
हल-
माना बुलबुले की त्रिज्या = r तथा बुलबुले का आयत
UP Board Solutions for Class 12 Maths Chapter 6 Application of Derivatives image 14
UP Board Solutions for Class 12 Maths Chapter 6 Application of Derivatives image 15
अत: बुलबुले का आयतन 2π सेमी3/से की दर से बढ़ रहा है।

प्रश्न 13.
एक गुब्बारा जो सदैव गोलाकार रहता है, का परिवर्तनशील व्यास [latex ]\frac { 3 }{ 2 }(2x+1)[/latex] है। x के सापेक्ष आयतन के परिवर्तन की दर ज्ञात कीजिए।
हल-
प्रश्नानुसार गोलाकार गुब्बारे का व्यास = [latex ]\frac { 3 }{ 2 }(2x+1)[/latex]
UP Board Solutions for Class 12 Maths Chapter 6 Application of Derivatives image 16
UP Board Solutions for Class 12 Maths Chapter 6 Application of Derivatives image 17

प्रश्न 14.
एक पाइप से रेत 12 सेमी3/से की दर से गिर रही है। गिरती रेत जमीन पर एक ऐसा शंकु बनाती है जिसकी ऊँचाई सदैव आधार की त्रिज्या का छठा भाग है।रेत से बने शंकु की ऊँचाई किस दर से बढ़ रही है जबकि ऊँचाई 4 सेमी है?
हल-
माना किसी क्षण t है पर शंकु की त्रिज्या r, ऊँचाई h तथा आयतन V है।
[latex ]h=\frac { r }{ 6 }(2x+1)[/latex]
⇒ r = 6h
UP Board Solutions for Class 12 Maths Chapter 6 Application of Derivatives image 18

प्रश्न 15.
एक वस्तु की x इकाइयों के उत्पादन की कुल लागत C (x) Rs में
C(x) = 0.007x3 – 0.003x2 + 15x + 4000
से प्राप्त होती है। सीमान्त लागत ज्ञात कीजिए जबकि 17 इकाइयों का उत्पादन किया जाता है।
हल-
प्रश्नानुसार, C(x) = 0.007x3 – 0.003x2 + 15x + 4000
UP Board Solutions for Class 12 Maths Chapter 6 Application of Derivatives image 19
= 6.069 – 0.102 + 15
= 20.967
अतः 17 इकाइयों के उत्पादन की सीमान्त लागत Rs 20.967 है।

UP Board Solutions

प्रश्न 16.
किसी उत्पाद की x इकाइयों के विक्रय से प्राप्त कुल आय R(x) Rs में R(x) = 13x2 + 26x + 15 से प्राप्त होती है। सीमान्त आय ज्ञात कीजिए जब x = 7 है।
हल-
प्रश्नानुसार, R(x) = 13x2 + 26x + 15
UP Board Solutions for Class 12 Maths Chapter 6 Application of Derivatives image 20
(MR)x=7 = 26 x 7 + 26
= 182 + 26
= 208
अत: अभीष्ट सीमान्त आय Rs 208 है।

प्रश्न 17.
एक वृत्त की त्रिज्या r = 6 सेमी पर r के सापेक्ष क्षेत्रफल में परिवर्तन की दर है :
(a) 10 π
(b) 12 π
(c) 8 π
(d) 11 π
हल-
मानी वृत्त का क्षेत्रफल = A तथा त्रिज्या = r
क्षेत्रफल A = πr²
UP Board Solutions for Class 12 Maths Chapter 6 Application of Derivatives image 21
अत: विकल्प (b) सही है।

प्रश्न 18.
एक उत्पाद की x इकाइयों के विक्रय से प्राप्त कुल आय रुपयों में R(x) = 3x² + 36x + 5 से प्रदत्त है। जब x = 15 है तो सीमान्ते आये है :
(a) 116
(b) 96
(c) 90
(d) 126
हल-
दिया है- R(x) = 3x² + 36x +5
सीमान्त ।
सीमान्त आय =
UP Board Solutions for Class 12 Maths Chapter 6 Application of Derivatives image 22
अब, x = 15, सीमान्त आय = 6 × 21 = Rs 126
अत: विकल्प (d) सत्य है।

प्रश्नावली 6.2

प्रश्न 1.
दिखाइए कि दिया गया फलन f, f(x) = x3 – 3x² + 4x, x ∈ R, R पर निरन्तर वृद्धिमान फलन है।
हल-
दिया गया फलन
f(x) = x3 – 3x² + 4x
f ‘(x) = 3x² – 6x +4
UP Board Solutions for Class 12 Maths Chapter 6 Application of Derivatives image 23
= 3(x – 1)² + 1 > 0, ∀ x∈R
∵ f ‘(x) > 0, ∀ x∈R
∴ f(x), R पर निरन्तर वृद्धिमान फलन है।

UP Board Solutions

प्रश्न 2.
सिद्ध कीजिए कि R पर f(x) = 3x + 17 निरन्तर वृद्धिमान फलन है।
हल-
दिया गया फलन f(x) = 3x + 17
f ‘(x) = 3 > 0, ∀ x∈R
f ‘(x) > 0, ∀ x∈R
∴ f(x), R पर निरन्तर वृद्धिमान फलन है।

प्रश्न 3.
सिद्ध कीजिए कि f(x) = sin x द्वारा दिया गया फलन
(a) (0, π/2) में निरन्तर वृद्धिमान है।
(b) (π/2, π) में निरन्तर ह्रासमान है।
(c) (0, π) में न तो वृद्धिमान है और न ह्रासमान।
हल-
(a) f(x) = sin x
⇒ f ‘(x) = cos x
UP Board Solutions for Class 12 Maths Chapter 6 Application of Derivatives image 24
अन्तराल (0, π/2) में निरन्तर वृद्धिमान तथा अन्तराल (π/2, π) में निरन्तर ह्रासमान है।
∴ फलन अन्तराल (0, π) में न तो वृद्धिमान है और न ह्रासमान,

प्रश्न 4.
अन्तराल ज्ञात कीजिए जिनमें f(x) = 2x² – 3x द्वारा दिया गया फलन
(a) निरन्तर वृद्धिमान है,
(b) निरन्तर ह्रासमान है।
हल-
(a) दिया गया फलन f(x) = 2x² – 3x
f ‘(x) = 4x – 3 > 0, ∀ x > [latex ]\frac { 3 }{ 4 }[/latex]
∴ f(x), अन्तराल (3/4, ∞) पर निरन्तर वृद्धिमान है।

(b) पुनः f ‘(3) = 4x – 3< 0, ∀ x < [latex ]\frac { 3 }{ 4 }[/latex]
∴ f(x), अन्तराल (-∞,3/4) पर निरन्तर ह्रासमान है।

UP Board Solutions

प्रश्न 5.
अन्तराल ज्ञात कीजिए जिनमें f(x) = 2x3 – 3x2 – 36x + 7 से दिया फलन f (a) निरन्तर वृद्धिमान है, (b) निरन्तर ह्रासमान है।
हल-
(a) दिया गया फलन f(x) = 2x3 – 3x2 – 36x +7
f ‘(x) = 6x2 – 6x – 36 = 6(x2 – x – 6).
UP Board Solutions for Class 12 Maths Chapter 6 Application of Derivatives image 25
UP Board Solutions for Class 12 Maths Chapter 6 Application of Derivatives image 26

प्रश्न 6.
अन्तराल ज्ञात कीजिए जिनमें निम्नलिखित फलन निरन्तर वर्धमान अथवा हासमान है
(a) f(x) = x² + 2x + 5
(b) f (x) = 10 – 6x – 2x²
(c) f (x) = – 2x3 – 9x2 – 12x + 1
(d) f(x) = 6 – 9x – x²
(e) f(x) = (x + 1)3 (x – 3)3
हल-
(a) ज्ञात है- f (x) = x2 + 2x + 5
f ‘ (x) = 2x + 2 = 2 (x + 1)
f ‘ (x) = 0 ⇒ 2 (x + 1) ⇒ x = – 1
x = – 1 संख्या रेखा को दो भागों में बांटता है। यह भाग अन्तराल (-∞ , -1) तथा (-1, ∞ ) है।
(- ∞ , – 1) में f ‘ (x) = – ऋणात्मक
अत: अन्तराल (-∞ , -1) में फलन f निरन्तर ह्रासमान है।
(-1, ∞ ) में f ‘ (x) = + धनात्मक
अतः अन्तराल (-1, ∞ ) फलन f निरन्तर वर्धमान है।
(b) ज्ञात है. f (x) = 10 – 6x – 2x²
f ‘ (x) = – 6 – 4x = – 2 (3 + 2x)
UP Board Solutions for Class 12 Maths Chapter 6 Application of Derivatives image 27
UP Board Solutions for Class 12 Maths Chapter 6 Application of Derivatives image 28
UP Board Solutions for Class 12 Maths Chapter 6 Application of Derivatives image 29

प्रश्न 7.
सिद्ध कीजिए कि
UP Board Solutions for Class 12 Maths Chapter 6 Application of Derivatives image 30
अपने सम्पूर्ण प्रान्त में एक वृद्धिमान फलन है।
हल-
दिया गया फलन
UP Board Solutions for Class 12 Maths Chapter 6 Application of Derivatives image 31
UP Board Solutions for Class 12 Maths Chapter 6 Application of Derivatives image 32

प्रश्न 8.
x के उन मानों को ज्ञात कीजिए जिनके लिए y = [x(x – 2)]² एक वर्धमान फलन है।
हल-
ज्ञात है- y = [x (x – 2)]² = x² (x + 4 – 4x)
= x4 – 4x3 + 4x2
x के सापेक्ष अवकलन करने पर,
UP Board Solutions for Class 12 Maths Chapter 6 Application of Derivatives image 33
∴ x = 0, x = 1, x = 2 से वास्तविक संख्या रेखा के चार भाग अन्तराल (-∞, 0), (0, 1), (1, 2), (2, 2) बनते हैं।
अन्तराल (- ∞, 0) में f ‘ (x) = (-) (-) (-) = – ve (ऋणात्मक)
अतः फलन f निरन्तर ह्रासमान है।
अन्तराल (0, 1) में f ‘ (x) = (+) (-) (-) = + ve (धनात्मक)
अतः फलन f निरन्तर वर्धमान है।
अन्तराल (1, 2) में f ‘ (x) = (+) (+) (-) = – ve (ऋणात्मक)
अतः फलन f निरन्तर ह्रासमान है।
अन्तराल (2, ∞) में f ‘ (x) = (+) (+) (+) = +ve (धनात्मक)
अतः फलन f निरन्तर वर्धमान है।
इस प्रकार (0, 1) ∪ (2, ∞) में फलन f वर्धमान है तथा (-∞, 0) ∪ (1, 2) में फलन ह्रासमान है।

UP Board Solutions

प्रश्न 9.
सिद्ध कीजिए कि [0, π/2] में
UP Board Solutions for Class 12 Maths Chapter 6 Application of Derivatives image 34
θ का एक वृद्धिमान फलन है।
हल-
UP Board Solutions for Class 12 Maths Chapter 6 Application of Derivatives image 35

प्रश्न 10.
सिद्ध कीजिए कि लघुगणकीय फलन (0,∞) में निरन्तर वर्धमान फलन है।
हल-
ज्ञात है– f (x) = log x, x > 0
f ‘(x) = [latex ]\frac { 1 }{ x }[/latex] = धनात्मक, x > 0 के लिए
अतः लघुगणकीय फलन अन्तराल (0, ∞) के लिए निरन्तर वर्धमान है। इति सिद्धम्

प्रश्न 11.
सिद्ध कीजिए कि (-1,1) में f (x) = x² – x + 1 से प्रदत्त फलन न तो वर्धमान है। और न ही ह्रासमान है।
हल-
दिया है | f (x) = x² – x + 1
UP Board Solutions for Class 12 Maths Chapter 6 Application of Derivatives image 36
UP Board Solutions for Class 12 Maths Chapter 6 Application of Derivatives image 37
इस प्रकार (-1, 1) में f ‘(x) का चिह्न एक नहीं है।
अतः इस अन्तराल में यह फलन न तो वर्धमान है और न ही ह्रासमान है। इति सिद्धम्

प्रश्न 12.
निम्नलिखित में कौन से फलन (0,[latex]\frac { \pi }{ 2 } [/latex]) में निरन्तर ह्रासमान है?
(A) cos x
(B) cos 2x
(C) cos 3x
(D) tan x
हल-
(A) माना f (x) = cos x, ∴ f ‘ (x) = – sin x
अन्तराल (0, π/ 2) में, sin x = + धनात्मक ⇒f ‘ (x) = – ऋणात्मक
अतः फलन f निरन्तर ह्रासमान है।
(B) माना f (x) = cos 2x
∴ f ‘(x) = – 2 sin 2x
UP Board Solutions for Class 12 Maths Chapter 6 Application of Derivatives image 38

प्रश्न 13.
निम्नलिखित अन्तरालों में से किस अन्तराल में f (x) = x100 + sin x – 1 द्वारा प्रदत्त फलन f निरन्तर ह्रासमान है ?
UP Board Solutions for Class 12 Maths Chapter 6 Application of Derivatives image 39
हल-
UP Board Solutions for Class 12 Maths Chapter 6 Application of Derivatives image 40

प्रश्न 14.
a का वह न्यूनतम मान ज्ञात कीजिए जिसके लिए अन्तराल [1, 2] में f(x) = x² + ax + 1 से दिया गया फलन निरन्तर वृद्धिमान है।
हल-
दिया गया फलन
f(x) = x² + ax + 1
f ‘(x) = 2x + a
अन्तराल [1, 2] में f ‘(x) का न्यूनतम मान f ‘(1) = 2 + a होगा
∵ f(x) अन्तराल [1, 2] में निरन्तर वृद्धिमान है ∴ f ‘(x) ≥ 0
∴ 2 + a ≥ 0
⇒ a≥ -2
अत: a का न्यूनतम मान -2 है।

UP Board Solutions

प्रश्न 15
माना[-1, 1] से असंयुक्त एक अन्तराल I हो तो सिद्ध कीजिए कि I में f(x) = [latex]x+\frac { 1 }{ x }[/latex] से दिया गया फलन f निरन्तर वृद्धिमान है।
हल-
दिया गया फलन f(x) = [latex]x+\frac { 1 }{ x }[/latex]
UP Board Solutions for Class 12 Maths Chapter 6 Application of Derivatives image 41
∴ (x – 1)(x + 1) > 0
∴ f ‘(x) > 0
⇒ f(x) निरन्तर वृद्धिमान है जब x∈ (1, ∞)
अतः f(x), I पर निरन्तर वृद्धिमान है।

प्रश्न 16.
सिद्ध कीजिए कि फलन f(x) = log sin x,(0,[latex]\frac { \pi }{ 2 } [/latex]) में निरन्तर वर्धमान और ([latex]\frac { \pi }{ 2 } [/latex],π) में निरन्तर ह्रासमान है।
हल-
दिया है- f(x) = log sin x
UP Board Solutions for Class 12 Maths Chapter 6 Application of Derivatives image 42
UP Board Solutions for Class 12 Maths Chapter 6 Application of Derivatives image 43

प्रश्न 17.
सिद्ध कीजिए कि फलन f(x) = log | cos x|; (0, π/2) निरन्तर ह्रासमान और (π/2, π) में निरन्तर वृद्धिमान है।
हल-
दिया गया फलन f(x) = log cos x
UP Board Solutions for Class 12 Maths Chapter 6 Application of Derivatives image 44

प्रश्न 18.
सिद्ध कीजिए कि R में दिया गया फलन f(x) = x3 – 3x2 + 3x – 100 वर्धमान है।
हल-
ज्ञात है- f (x) = x3 – 3x2 + 3x – 100
∴f ‘(x) = 3x2 – 6x + 3 = 3 (x2 – 2x + 1) = 3(x – 1)2
∀x∈ R, f ’(x) = धनात्मक
अतः फलन f वर्धमान है। इति सिद्धम्

UP Board Solutions

प्रश्न 19.
निम्नलिखित में से किस अन्तराल में y = x2e-x वर्धमान है?
(a) (-∞, ∞)
(b) (-2, 0)
(c) (2, ∞)
(d) (0, 2)
हल-
दिया है- f (x) = x2e-x
UP Board Solutions for Class 12 Maths Chapter 6 Application of Derivatives image 45

प्रश्नावली 6.3

प्रश्न 1.
वक्र y = 3x4 – 4x के x = 4पर स्पर्श रेखा की प्रवणता ज्ञात कीजिए।
हल-
दिया है, वक्र का समीकरण y = 3x4 -4x
UP Board Solutions for Class 12 Maths Chapter 6 Application of Derivatives image 46
= 4[3 x 64 – 1]
= 4[192 – 1]
= 4 x 191
= 764
∴स्पर्श रेखा की प्रवणता = 764

प्रश्न 2.
वक्र [latex ]y=\frac { x-1 }{ x-2 }[/latex],x ≠ 2 के x = 10 पर स्पर्श रेखा की प्रवणता ज्ञात कीजिए।
हल-
दिया है, वक्र का समीकरण [latex ]y=\frac { x-1 }{ x-2 }[/latex],x ≠ 2
दोनों पक्षों का x के सापेक्ष अवकलन करने पर
UP Board Solutions for Class 12 Maths Chapter 6 Application of Derivatives image 47

प्रश्न 3.
वक्र y = x3 – x + 1 की स्पर्श रेखा की प्रवणता उस बिन्दु पर ज्ञात कीजिए जिसका x-निर्देशांक 2 है।
हल-
दिया है, वक्र का समीकरण y = x3 – x + 1
UP Board Solutions for Class 12 Maths Chapter 6 Application of Derivatives image 48

प्रश्न 4.
वक्र y = x3 – 3x + 2 की स्पर्श रेखा की प्रवणता उस बिन्दु पर ज्ञात कीजिए जिसका x – निर्देशांक 3 है।
हल-
UP Board Solutions for Class 12 Maths Chapter 6 Application of Derivatives image 49

प्रश्न 5.
वक्र x = a cos3θ, y= a sin3θ के θ = [latex]\frac { \pi }{ 4 } [/latex] पर अभिलम्ब की प्रवणता ज्ञात कीजिए।
हल-
दिया है, वक्र को समीकरण x = a cos3θ तथा y = a sin3θ
दोनों पक्षों का θ के सापेक्ष अवकलन करने पर,
UP Board Solutions for Class 12 Maths Chapter 6 Application of Derivatives image 50

प्रश्न 6.
वक्र x = 1 – a sin θ, y = b cos² θ के θ = [latex]\frac { \pi }{ 2 } [/latex] पर अभिलम्ब की प्रवणता ज्ञात कीजिए।
हल-
दिया है, वक्र का समीकरण x = 1 – a sin θ तथा y = b cos² θ
दोनों पक्षों का θ के सापेक्ष अवकलन करने पर,
UP Board Solutions for Class 12 Maths Chapter 6 Application of Derivatives image 51

प्रश्न 7.
वक्र y = x3 – 3x– 9x + 7 पर उन बिन्दुओं को ज्ञात कीजिए जिन पर स्पर्श रेखायें x-अक्ष के समान्तर हैं।
हल-
UP Board Solutions for Class 12 Maths Chapter 6 Application of Derivatives image 52

UP Board Solutions

प्रश्न 8.
वक्र y = (x – 2)² पर एक बिन्दु ज्ञात कीजिए जिस पर स्पर्श रेखा बिन्दुओं (2,0) और (4,4) को मिलाने वाली रेखा के समान्तर है।
हल-
दिया है, वक्र का समीकरण y = (x – 2)²
दोनों पक्षों का x के सापेक्ष अवकलन करने पर,
UP Board Solutions for Class 12 Maths Chapter 6 Application of Derivatives image 53

प्रश्न 9.
वक्र y = x3 – 11x + 5 पर उस बिन्दु को ज्ञात कीजिए जिस पर स्पर्श रेखा y = x – 11 है।
हल-
दिया है, वक्र का समीकरण y = x3 – 11x + 5
UP Board Solutions for Class 12 Maths Chapter 6 Application of Derivatives image 54

प्रश्न 10.
प्रवणता -1 वाली सभी रेखाओं का समीकरण ज्ञात कीजिए जो वक़ [latex ]y=\frac { 1 }{ x-1 }[/latex],x ≠ -1 को स्पर्श करती है।
हल-
दिया है, वक्र का समीकरण [latex ]y=\frac { 1 }{ x-1 }[/latex]
UP Board Solutions for Class 12 Maths Chapter 6 Application of Derivatives image 55
UP Board Solutions for Class 12 Maths Chapter 6 Application of Derivatives image 56

प्रश्न 11.
प्रवणता 2 वाली सभी रेखाओं का समीकरण ज्ञात कीजिए जो वक्र [latex ]y=\frac { 1 }{ x-3 }[/latex],x ≠ 3 को स्पर्श करती है।
हल-
दिया है, वक्र का समीकरण [latex ]y=\frac { 1 }{ x-3 }[/latex]
दोनों पक्षों का x के सापेक्ष अवकलन करने पर,
UP Board Solutions for Class 12 Maths Chapter 6 Application of Derivatives image 57

प्रश्न 12.
प्रवणता 0 वाली सभी रेखाओं का समीकरण ज्ञात कीजिए जो वक्र
UP Board Solutions for Class 12 Maths Chapter 6 Application of Derivatives image 58
को स्पर्श करती है।
हल-
दिया है, वक्र का समीकरण
UP Board Solutions for Class 12 Maths Chapter 6 Application of Derivatives image 58
दोनों पक्षों को x के सापेक्ष अवकलन करने पर,
UP Board Solutions for Class 12 Maths Chapter 6 Application of Derivatives image 59

UP Board Solutions

प्रश्न 13.
वक्र
UP Board Solutions for Class 12 Maths Chapter 6 Application of Derivatives image 60
पर उन बिन्दुओं को ज्ञात कीजिए जिन पर स्पर्श रेखाएँ
(i) x-अक्ष के समान्तर हैं,
(ii) y-अक्ष के समान्तर हैं।
हल-
दिया है, वक्र का समीकरण
UP Board Solutions for Class 12 Maths Chapter 6 Application of Derivatives image 60
दोनों पक्षों का x के सापेक्ष अवकलन करने पर,
UP Board Solutions for Class 12 Maths Chapter 6 Application of Derivatives image 61

प्रश्न 14.
दिए वक्रों पर निर्दिष्ट बिन्दुओं पर स्पर्श रेखा और अभिलम्ब के समीकरण ज्ञात कीजिए
(i) y = x4 – 6x3 + 13x2 – 10x + 5 के (0, 5) पर
(ii) y = x4 – 6x3 + 13x2 – 10x + 5 के (1, 3) पर
(iii) y = x3 के (1, 1) पर .
(iv) y = x² के (0, 0) पर
(v) x = cost, y = sin t के [latex]t=\frac { \pi }{ 4 } [/latex] पर
हल-
UP Board Solutions for Class 12 Maths Chapter 6 Application of Derivatives image 62
UP Board Solutions for Class 12 Maths Chapter 6 Application of Derivatives image 63
UP Board Solutions for Class 12 Maths Chapter 6 Application of Derivatives image 64

प्रश्न 15.
वक्र y = x² – 2x + 7 की स्पर्श रेखा का समीकरण ज्ञात कीजिए, जो
(a) रेखा 2x – y + 9 = 0 के समान्तर है।
(b) रेखा 5y – 15x = 13 पर लम्ब है।
हल-
दिया है, वक्र का समीकरण y = x² – 2x + 7
दोनों पक्षों का x के सापेक्ष अवकलन करने पर,
UP Board Solutions for Class 12 Maths Chapter 6 Application of Derivatives image 65
UP Board Solutions for Class 12 Maths Chapter 6 Application of Derivatives image 66

प्रश्न 16.
सिद्ध कीजिए कि वक्र y = 7x3 + 11 के उन बिन्दुओं पर स्पर्श रेखाएँ समान्तर हैं जहाँ x = 2 तथा x = – 2 है।
हल-
दिया है, वक्र का समीकरण y = 7x3 + 11
दोनों पक्षों का x के सापेक्ष अवकलन करने पर, [latex ]\frac { dy }{ dx }[/latex] = 21 x²
जब x = 2, तब स्पर्श रेखा की प्रवणता = 21 x 2² = 21 x 4 = 84
जब x = -2, तब स्पर्श रेखा की प्रवणता = 21 x (-2)² = 84
x = 2 तथा x = -2 पर स्पर्श रेखा की प्रवणता समान हैं।
अतः इन बिन्दुओं पर स्पर्श रेखाएँ समान्तर हैं। इति सिद्धम्

प्रश्न 17.
वक्र y = x3 पर उन बिन्दुओं को ज्ञात कीजिए जिन पर स्पर्श रेखा की प्रवणता बिन्दु के y-निर्देशांक के बराबर है।
हल-
दिया है, वक्र की समीकरण y = x3
दोनों पक्षों का x के सापेक्ष अवकलन करने पर, [latex ]\frac { dy }{ dx }[/latex] = 3x²
UP Board Solutions for Class 12 Maths Chapter 6 Application of Derivatives image 67
UP Board Solutions for Class 12 Maths Chapter 6 Application of Derivatives image 68

प्रश्न 18.
वक्र y = 4x3 – 2x5, पर उन बिन्दुओं को ज्ञात कीजिए जिन पर स्पर्श रेखाएँ मूलबिन्दु से होकर जाती हैं।
हल-
दिया है, वक्र का समीकरण y = 4x3 – 2x5
UP Board Solutions for Class 12 Maths Chapter 6 Application of Derivatives image 69

प्रश्न 19.
वक्र x² + y2 – 2x – 3 = 0 के उन बिन्दुओं पर स्पर्श रेखाओं के समीकरण ज्ञात कीजिए जहाँ पर वे x-अक्ष के समान्तर हैं।
हल-
दिया है, वक्र का समीकरण x² + y² – 2x – 3 = 0
दोनों पक्षों का x के सापेक्ष अवकलन करने पर,
UP Board Solutions for Class 12 Maths Chapter 6 Application of Derivatives image 70

प्रश्न 20
वक्र ay2 = x3 के बिन्दु (am2, um3)पर अभिलम्ब का समीकरण ज्ञात कीजिए और m का मान बताइए जिसके लिए अभिलम्ब बिन्दु (a, 0) से होकर जाता है।
हल-
वक्र ay2 = x3 ….(1)
समीकरण (1) का x के सापेक्ष अवकलन करने पर,
UP Board Solutions for Class 12 Maths Chapter 6 Application of Derivatives image 71

प्रश्न 21
UP Board Solutions for Class 12 Maths Chapter 6 Application of Derivatives image 72
हल-
UP Board Solutions for Class 12 Maths Chapter 6 Application of Derivatives image 73

UP Board Solutions

प्रश्न 22.
परवलय y² = 4ax के बिन्दु (at², 2at) पर स्पर्श रेखा और अभिलम्ब के समीकरण ज्ञात कीजिए।
हल-
दिया है, वक्र का समीकरण y² = 4ax
दोनों पक्षों का x के सापेक्ष अवकलन करने पर,
UP Board Solutions for Class 12 Maths Chapter 6 Application of Derivatives image 74

प्रश्न 23
UP Board Solutions for Class 12 Maths Chapter 6 Application of Derivatives image 75
हल-
UP Board Solutions for Class 12 Maths Chapter 6 Application of Derivatives image 76

प्रश्न 24.
अतिपरवलय [latex ]\frac { { x }^{ 2 } }{ { a }^{ 2 } } -\frac { { y }^{ 2 } }{ { b }^{ 2 } } =1[/latex] के बिन्दु (x0, y0) पर स्पर्श रेखा तथा अभिलम्ब के समीकरण ज्ञात कीजिए।
हल-
दिया है, वक्र का समीकरण [latex ]\frac { { x }^{ 2 } }{ { a }^{ 2 } } -\frac { { y }^{ 2 } }{ { b }^{ 2 } } =1[/latex]
दोनों पक्षों का x के सापेक्ष अवकलन करने पर,
UP Board Solutions for Class 12 Maths Chapter 6 Application of Derivatives image 77
UP Board Solutions for Class 12 Maths Chapter 6 Application of Derivatives image 78

प्रश्न 25.
वक्र [latex ]y=\sqrt { 3x-2 } [/latex] की उन स्पर्श रेखाओं के समीकरण ज्ञात कीजिए जो रेखा 4x – 2y + 5 = 0 के समान्तर है।
हल-
दिया है, वक्र का समीकरण [latex ]y=\sqrt { 3x-2 } [/latex] …(1)
दोनों पक्षों का x के सापेक्ष अवकलन करने पर,
UP Board Solutions for Class 12 Maths Chapter 6 Application of Derivatives image 79
UP Board Solutions for Class 12 Maths Chapter 6 Application of Derivatives image 80

प्रश्न 26.
वक्र y = 2x2 + 3sin x के x = 0 पर अभिलम्ब की प्रवणता है
(A) 3
(B) [latex ]\frac { 1 }{ 3 }[/latex]
(C) 3
(D) [latex ]-\frac { 1 }{ 3 }[/latex]
हल-
दिया है, वक्र का समीकरण y = 2x² + 3 sin x
दोनों पक्षों का x के सापेक्ष अवकलन करने पर, [latex ]\frac { dy }{ dx }=4x+3cosx[/latex]
UP Board Solutions for Class 12 Maths Chapter 6 Application of Derivatives image 81
अतः विकल्प (D) सही है।

प्रश्न 27.
किस बिन्दु पर y = x + 1, वक्र y² = 4x की स्पर्श रेखा है?
(A) (1,2)
(B) (2,1)
(C) (1,- 2)
(D) (-1, 2)
हल-
दिया है, वक्र का समीकरण y² = 4x …(1)
दोनों पक्षों का x के सापेक्ष अवकलन करने पर,
UP Board Solutions for Class 12 Maths Chapter 6 Application of Derivatives image 82
UP Board Solutions for Class 12 Maths Chapter 6 Application of Derivatives image 83

प्रश्नावली 6.4

प्रश्न 1.
अवकल का प्रयोग करके निम्नलिखित में से प्रत्येक का सन्निकट मान दशमलव के तीन स्थानों तक ज्ञात कीजिए
(i) [latex ]\sqrt { 25.3 } [/latex]
(ii) [latex ]\sqrt { 49.5 } [/latex]
(iii) [latex ]\sqrt { 0.6 } [/latex]
(iv) (0.009)1/3
(v) (0.999)1/10
(vi) (15)1/4
(vii) (26)1/3
(viii) (255)1/4
(ix) (82)1/4
(x) (401)1/2
(xi) (0.0037)1/2
(xii) (26.57)1/3
(xiii) (81.5)1/4
(xiv) (3,968)3/2
(xv) (32.15)1/5
हल-
UP Board Solutions for Class 12 Maths Chapter 6 Application of Derivatives image 84
UP Board Solutions for Class 12 Maths Chapter 6 Application of Derivatives image 85
UP Board Solutions for Class 12 Maths Chapter 6 Application of Derivatives image 86
UP Board Solutions for Class 12 Maths Chapter 6 Application of Derivatives image 87
UP Board Solutions for Class 12 Maths Chapter 6 Application of Derivatives image 88
UP Board Solutions for Class 12 Maths Chapter 6 Application of Derivatives image 89
UP Board Solutions for Class 12 Maths Chapter 6 Application of Derivatives image 90
UP Board Solutions for Class 12 Maths Chapter 6 Application of Derivatives image 91
UP Board Solutions for Class 12 Maths Chapter 6 Application of Derivatives image 92
UP Board Solutions for Class 12 Maths Chapter 6 Application of Derivatives image 93
UP Board Solutions for Class 12 Maths Chapter 6 Application of Derivatives image 94
UP Board Solutions for Class 12 Maths Chapter 6 Application of Derivatives image 95
UP Board Solutions for Class 12 Maths Chapter 6 Application of Derivatives image 96

प्रश्न 2.
f(2.01) का सन्निकट मान ज्ञात कीजिए जबकि f(x) = 4x² + 5x + 2
हल-
माना x = 2 और x + ∆x = 2.01 तब ∆x = 0.01 = dx (∵∆Y = dx)
UP Board Solutions for Class 12 Maths Chapter 6 Application of Derivatives image 97

UP Board Solutions

प्रश्न 3.
f(5.001) का सन्निकट मान ज्ञात कीजिए जहाँ f(x) = x3 – 7 x² + 15
हल-
UP Board Solutions for Class 12 Maths Chapter 6 Application of Derivatives image 98

प्रश्न 4.
x मी भुजा वाले घन की भुजा में 1% की वृद्धि होने के कारण घन के आयतन में होने वाला सन्निकट परिवर्तन ज्ञात कीजिए।
हल-
माना घन का आयतन V = x3
UP Board Solutions for Class 12 Maths Chapter 6 Application of Derivatives image 99
घन के आयतन में सन्निकट परिवर्तन 0.03 x3 मी है।

प्रश्न 5.
x मी भुजा वाले घन की भुजा में 1% ह्रास होने के कारण घन के पृष्ठ क्षेत्रफल में होने वाला सन्निकट परिवर्तन ज्ञात कीजिए।
हल-
घन का पृष्ठ क्षेत्रफल S = 6x2
[latex ]\frac { dS }{ dx }=12x[/latex]
UP Board Solutions for Class 12 Maths Chapter 6 Application of Derivatives image 100
घन के आयतन में सन्निकट परिवर्तन -0.12 x2 मी2 है।

प्रश्न 6.
एक गोले की त्रिज्या 7 मी मापी जाती है जिसमें 0.02 मी की त्रुटि है। इसके आयतन के परिकलन में सन्निकट त्रुटि ज्ञात कीजिए।
हल-
ज्ञात है- गोले की त्रिज्या = 7 मी ।
∆r = त्रिज्या में अशुद्धि = 0.02 मी
UP Board Solutions for Class 12 Maths Chapter 6 Application of Derivatives image 101

प्रश्न 7.
एक गोले की त्रिज्या 9 मी मापी जाती है जिसमें 0.03 मी की त्रुटि है। इसके पृष्ठ क्षेत्रफल के परिकलन में सन्निकट त्रुटि ज्ञात कीजिए।
हल-
ज्ञात है- r = गोले की त्रिज्या = 9 मी
∆r = त्रिज्या में अशुद्धि = 0.03
UP Board Solutions for Class 12 Maths Chapter 6 Application of Derivatives image 102

प्रश्न 8.
यदि f (x) = 3x² + 15x + 5 हो तो f (3.02) का सन्निकट मान है–
(A) 47.66
(B) 57.66
(C) 67.66
(D) 77.66
हल-
f (3.02) = f (3) + df (3) [3.02 = 3 + 0.02]
यदि f (x) = 3x² + 15x + 5 …(1)
f ‘(x) = 6x + 15
समी० (1) में x = 3 रखने पर,
f (3) = 3 x 9 + 15 x 3 + 5 = 27 + 45 + 5 = 77
df (x) = f ‘(x) x ∆x = (6x + 15) x ∆x
= (6 x 3 + 15) x 0.02 [∴ x = 3, ∆ x = 0.02]
= (18 + 15) x 0.02
= 33 x 0.02 = 0.66
∴ f (3.02) = f (3) + df (3) = 77 + 0.66 = 77.66
अत: विकल्प (D) सही है।

प्रश्न 9.
भुजा में 3% वृद्धि के कारण भुजा x के घन के आयतन में सन्निकट परिवर्तन है
(A) 0.06 x3 मी3
(B) 0.6 x3 मी3
(C) 0.09 xमी3
(D) 0.9 xमी3
हल-
चूँकि घन का आयतन V = x3 (∵ भुजा = x मी)
भुजा में वृद्धि, ∆x = 3% = x का
UP Board Solutions for Class 12 Maths Chapter 6 Application of Derivatives image 103
अत: विकल्प (C) सही है।

प्रश्नावली 6.5

प्रश्न 1.
निम्नलिखित दिए गए फलनों के उच्चतम या निम्नतम मान, यदि कोई हो तो ज्ञात कीजिए
(i) f (x) = (2x – 1)² + 3
(ii) f (x) = 9x² + 12x + 2
(iii) f (x) = -(x – 1)² + 10
(iv) g(x) = x3 + 1
हल-
(i) दिया गया फलन f(x) = (2x – 1)² + 3
(2x – 1)² का कम-से-कम मान = 0,
⇒ f(x) ≥ 3; ∀ x∈R
∴ f (x) का निम्नतम मान = 3
(ii) दिया गया फलन f (x) = 9x² + 12x + 2 = 9x² + 12x + 4 – 2
= (3x + 2)² – 2
(3x + 2)² का निम्नतम मान = 0,
⇒ f (x) ≥ -2; ∀ x∈R
∴ f (x) का निम्नतम मान = -2
(iii) दिया गया फलन f (x) = – (x – 1)² + 10
– (x – 1)² का उच्चतम मान = 0
⇒f (x) ≤ 10; ∀ x∈R
∴f का उच्चतम मान = 10
(iv) यहाँ g(x) = x3 + 1.
g ‘(x) = 3x² जो x ∈ R के लिए धनात्मक है।
g ‘(x) = 3x² ≥ 0; ∀ x∈R
अत: g एक वर्धमान फलन है।
∴ इसका कोई न्यूनतम तथा अधिकतम मान नहीं है।

UP Board Solutions

प्रश्न 2.
निम्नलिखित दिए गए फलनों के उच्चतम मान या निम्नतम मान, यदि कोई हो तो ज्ञात कीजिए
(i) f(x) = |x + 2| – 1
(ii) g(x) = -|x + 1| + 3
(iii) h(x) = sin (2x) + 5
(iv) f(x) =|sin 4x + 3|
(v) h(x) = x + 1, x∈(-1,1)
हल-
(i) दिया गया फलन f(x) =|x + 2| – 1, f (x)≥ -1; ∀ x∈R
|x + 2| को निम्नतम मान 0 है।
∴ f का निम्नतम मान = -1
|x + 2| कर उच्चतम मान अनन्त हो सकता है।
अत: उच्चतम मान का अस्तित्व नहीं है।
(ii) दिया गया फलन g(x) = -|x + 1| + 3; g (3) ≤ 3∀ x∈R
-|x +1| का उच्चतम मान = 0
g(x) = -|x + 1| + 3 का उच्चतम मान = 0 + 3 = 3
तथा निम्नतम मान का अस्तित्व नहीं है।
(iii) दिया गया फलन h(x) = sin (2x) + 5
हम जानते हैं कि -1 ≤ sin 2x ≤ 1
⇒ 4 ≤ 5 + sin 2x ≤ 6
sin 2x का उच्चतम मान = 1
UP Board Solutions for Class 12 Maths Chapter 6 Application of Derivatives image 104

प्रश्न 3
निम्नलिखित फलनों के स्थानीय उच्चतम या निम्नतम, यदि कोई हो तो ज्ञात कीजिए तथा स्थानीय उच्चतम या स्थानीय निम्नतम माने, जैसी स्थिति हो, भी ज्ञात कीजिए।
UP Board Solutions for Class 12 Maths Chapter 6 Application of Derivatives image 105
हल-
(i) दिया गया फलन f(x) = x²
⇒ f ‘(x) = 2x
यदि f ‘(x) = 0 तब 2x = 0 या x = 0
f ‘(x) जैसे ही x = 0 से होकर आगे बढ़ता है तब इसका चिह्न ऋणात्मक से धनात्मक में बदल जाता है।
∴x = 0 पर f स्थानीय मान निम्नतम है।
स्थानीय निम्नतम मान = f (0) = 0
UP Board Solutions for Class 12 Maths Chapter 6 Application of Derivatives image 106
UP Board Solutions for Class 12 Maths Chapter 6 Application of Derivatives image 107
UP Board Solutions for Class 12 Maths Chapter 6 Application of Derivatives image 108UP Board Solutions for Class 12 Maths Chapter 6 Application of Derivatives image 109
UP Board Solutions for Class 12 Maths Chapter 6 Application of Derivatives image 110
UP Board Solutions for Class 12 Maths Chapter 6 Application of Derivatives image 111

प्रश्न 4
सिद्ध कीजिए कि निम्नलिखित फलनों को उच्चतम या निम्नतम मान नहीं है–
(i) f (x) = ex
(ii) g(x) = log x
(iii) h(x) = x3 + x2 + x + 1
हल-
(i) दिया गया फलन f ‘(x) = ex
∴f ‘(x) = ex
f ‘(x), x∈R कभी भी शून्य के समान नहीं है।
अत: f का कोई उच्चतम या निम्नतम मान नहीं है। इति सिद्धम्
UP Board Solutions for Class 12 Maths Chapter 6 Application of Derivatives image 112

UP Board Solutions

प्रश्न 5
प्रदत्त अन्तरालों में निम्नलिखित फलनों के निरपेक्ष उच्चतम मान और निरपेक्ष निम्नतम मान ज्ञात कीजिए
UP Board Solutions for Class 12 Maths Chapter 6 Application of Derivatives image 113
हल-
(i) दिया गया फलन f(x) = x3, अन्तराल [-2, 2]
f ‘(x) = 3x2
यदि f ‘(x) = 0, तब 3x² = 0
⇒ x = 0
x = -2 पर, f(-2) = (-2)3 = – 8
x = 0 पर, f(0) = (0)3 = 0
UP Board Solutions for Class 12 Maths Chapter 6 Application of Derivatives image 114
UP Board Solutions for Class 12 Maths Chapter 6 Application of Derivatives image 115

प्रश्न 6
यदि लाभ फलन p(x) = 41 – 72x – 18x² से प्रदत्त है तो किसी कम्पनी द्वारा अर्जित उच्चतम लाभ ज्ञात कीजिए।
हल-
दिया गया फलन लाभ p(x) = 41 -72x – 18x² …(1)
p’ (x) = – 72 – 36x = – 36 (2 + x)
p ” (x) = – 36
यदि p ‘(x) = 0, तब – 36 (2 + x) = 0 ⇒ 2 + x = 0 ∴ x = -2
p ‘(x) = – ve
अतः x = -2 पर p(x) उच्चतम है।
∴उच्चतम लाभ = p(-2)
[समी० (1) में x  = -2 रखने पर]
= 41 – 72 (-2)2 – 18 (-2)²
= 41 + 144 – 72
= 43 इकाई

प्रश्न 7
अन्तराल [0, 3] पर 3x4 – 8x3 + 12x2 – 48x + 25 के उच्चतम मान और निम्नतम मान ज्ञात कीजिए।
हल-
माना f (x) = 3x4 – 8x3 + 12x2 – 48x + 25
f ‘(x) = 12x3 – 24x2 + 24x – 48
= 12 [x3 – 2x2 + 2x – 4] = 12 [x² (x – 2) + 2 (x – 2)]
= 12 (x – 2) (x2 + 2)
यदि f ‘(x) = 0, तब x – 2 = 0 ⇒ x = 2
UP Board Solutions for Class 12 Maths Chapter 6 Application of Derivatives image 116

प्रश्न 8
अन्तराल [0, 2π] के किन बिन्दुओं पर फलन sin 2 x अपना उच्चतम मान प्राप्त करता है।
हल-
माना f (x) = sin 2x, अन्तराल [0, 2π]
f ‘(x) = 2 cos 2x
UP Board Solutions for Class 12 Maths Chapter 6 Application of Derivatives image 117
UP Board Solutions for Class 12 Maths Chapter 6 Application of Derivatives image 118

प्रश्न 9.
फलन sin x + cos x का उच्चतम मान क्या है?
हल-
माना f (x) = sin x + cos x, अन्तराल [0, 2π]
f ‘(x) = cos x – sin x
उच्चतम व निम्नतम मान के लिए,
UP Board Solutions for Class 12 Maths Chapter 6 Application of Derivatives image 119

प्रश्न 10.
अन्तराल [1,3] में 2x3 – 24x + 107 का महत्तम मान ज्ञात कीजिए। इसी फलन का अन्तराला [-3,-1] में भी महत्तम मान ज्ञात कीजिए।
हल-
माना
f (x) = 2x3 – 24x + 107, अन्तराल [1, 3]
f ‘(x) = 6x² – 24
उच्चतम व निम्नतम मान के लिए, f ‘(x) = 0
⇒ 6x2 – 24 = 0 ⇒ 6x2 = 24 ⇒ x2 = 4 ⇒ x = ±2
अन्तराल [1, 3] के लिए f(x) = 2x3 – 24x + 107 में x के मान रखने पर,
x = 1 पर, f(1) = 2(1)3 – 24 (1) + 107 = 2 – 24 + 107 = 85
x = 3 पर, f (3) = 2(3)3 – 24 (3) + 107 = 54 – 72 + 107 = 89
x = 2 परे, f(2) = 2(2)3 – 24(2) + 107 = 16 – 48 + 107 = 75
इस प्रकार अधिकतम मान f (x) = 89,
x = 3 पर, अन्तराल [-3,-1] के लिए हम x = – 3, – 2, – 1 पर f(x) का मान ज्ञात करते हैं।
x = – 3 पर, f(-3) = 2(-3)3 – 24 (-3) + 107
= – 54 + 72 + 107 = – 54 + 179 = 125
x = – 1 पर f(-1) = 2 (-1)3 – 24 (-1) + 107 = -2 +24 + 107 = 129
x = – 2 पर f(-2) = 2(-2)3 – 24 (-2) + 107 = -16 + 48 +107 = 139
इस प्रकार अधिकतम मान f (x) = 139, x = -2 पर।

UP Board Solutions

प्रश्न 11.
यदि दिया है कि अन्तराल [0,2] में x = 1 पर फलन x4 – 62x2 + ax + 9 उच्चतम मान प्राप्त करता है तो a का मान ज्ञात कीजिए।
हल-
माना f(x) = x4 – 62x2 + ax + 9
f ‘(x) = 4x3 – 124x + a
उच्चतम व निम्नतम मान के लिए, f ‘(x) = 0
⇒ 4x3 – 124x + a = 0
दिया है, x = 1 पर, f उच्चतम है ⇒ f (1) = 0
4x3 – 124x + a = 0 में x = 1 रखने पर
4 x 1 – 124 x 1 + a = 0 ⇒ 4 – 124 + a = 0 ⇒ – 120 + a = 0
a = 120
इसलिए a का मान 120 है।

प्रश्न 12.
[0,2π] पर x + sin 2x का उच्चतम और निम्नतम मान ज्ञात कीजिए।
हल-
माना f(x) = x + sin 2x
f ‘(x) = 1 + 2 cos 2x
उच्चतम व निम्नतम मान के लिए, f ‘(x) = 0
⇒ 1 + 2 cos 2x = 0 ⇒ cos2x = [latex]-\frac { 1 }{ 2 }[/latex]
UP Board Solutions for Class 12 Maths Chapter 6 Application of Derivatives image 120

प्रश्न 13.
ऐसी दो संख्याएँ ज्ञात कीजिए जिनका योग 24 है और जिनका गुणनफल उच्चतम हो।
हल-
माना पहली संख्या = x तब दूसरी संख्या = 24 – x है।
प्रश्नानुसार, उनका गुणनफल p = x(24 – x) = 24x – x² …(1)
उच्चतम व निम्नतम मान के लिए, [latex]\frac { dp }{ dx }=0[/latex]
समी० (1) का x के सापेक्ष अवकलन करने पर,
UP Board Solutions for Class 12 Maths Chapter 6 Application of Derivatives image 121

UP Board Solutions

प्रश्न 14.
ऐसी दो धन संख्याएँ x और y ज्ञात कीजिए ताकि x + y = 60 और xy3 उच्चतम हो।
हल-
दिया है,
x + y = 60
x = 60 – y …(1)
माना xy3 = P …(2)
समीकरण (1) से x का मान समीकरण (2) में रखने पर,
UP Board Solutions for Class 12 Maths Chapter 6 Application of Derivatives image 122
UP Board Solutions for Class 12 Maths Chapter 6 Application of Derivatives image 123

प्रश्न 15.
ऐसी दो धन संख्याएँ x और y ज्ञात कीजिए जिनका योग 35 हो और गुणनफल x2y5 उच्चतम हो।
हल-
दो धन संख्याएँ x, y हैं।
दिया है, x + y = 35
⇒ y = 35 – x …(1)
प्रश्नानुसार, माना गुणनफल p = x2y5 …(2)
समीकरण (1) से y का मान समीकरण (2) में रखने पर,
p = x2 (35 – x)5
दोनों पक्षों का x के सापेक्ष अवकलन करने पर,
UP Board Solutions for Class 12 Maths Chapter 6 Application of Derivatives image 124

प्रश्न 16.
ऐसी दो धन संख्याएँ ज्ञात कीजिए जिनका योग 16 हो और जिनके घनों का योग निम्नतम हो।
हल-
UP Board Solutions for Class 12 Maths Chapter 6 Application of Derivatives image 125

प्रश्न 17.
18 सेमी भुजा के टिन के किसी वर्गाकार टुकड़े से प्रत्येक कोने पर एक वर्ग काटकर तथा इस प्रकार बने टिन के फलकों को मोड़कर ढक्कन रहित एक सन्दूक बनाना है। काटे जाने वाले वर्ग की भुजा कितनी होगी जिससे सन्दूक का आयतन उच्चतम होगा?
हल-
माना वर्ग की प्रत्येक भुजा x सेमी काटी गई है।
∴ सन्दूक के लिए,
लम्बाई = 18 – 2x
चौड़ाई = 18 – 2x
ऊँचाई = x
आयतन V = ल० × चौ० × ऊँ०
= x(18 – 2x) (18 – 2x)
= x(18 – 2x)x² …(1)
दोनों पक्षों का x के सापेक्ष अवकलन करने पर,
UP Board Solutions for Class 12 Maths Chapter 6 Application of Derivatives image 126
UP Board Solutions for Class 12 Maths Chapter 6 Application of Derivatives image 127

UP Board Solutions

प्रश्न 18
45 सेमी लम्बी और 24 सेमी चौड़ी आयताकार लोहे की एक चादर के चारों कोनों से समान भुजा का एक वर्गाकार निकालने के पश्चात् खुला हुआ एक सन्दुक बनाया जाता है। वर्गों की भुजा की माप ज्ञात कीजिये जिसके काटने पर बने सन्दूक का आयतन महत्तम होगा।
हल-
माना अभीष्ट वर्ग की भुजा x है तब ।।
सन्दूक की लम्बाई = (45-2x)
तथा सन्दूक की चौड़ाई = (24-2x)
सन्दूक की ऊँचाई = x
∴ सन्दूक का आयतन
V = (45 – 2x) (24 – 2x) x
UP Board Solutions for Class 12 Maths Chapter 6 Application of Derivatives image 128
UP Board Solutions for Class 12 Maths Chapter 6 Application of Derivatives image 129
∴x = 5 पर V का मान महत्तम होगा।
∴ वर्ग की भुजा 5 सेमी होगी।

प्रश्न 19.
सिद्ध कीजिए कि एक दिए वृत्त के अन्तर्गत सभी आयतों में वर्ग का क्षेत्रफल उच्चतम होता है।
हल-
माना a त्रिज्या के वृत्त के अन्तर्गत आयत की लम्बाई x तथा चौड़ाई y है।
चित्र ABC में,
AC = व्यास = 2a
UP Board Solutions for Class 12 Maths Chapter 6 Application of Derivatives image 130

प्रश्न 20.
सिद्ध कीजिए कि दिए हुए सम्पूर्ण पृष्ठ और महत्तम आयतन के लम्बवृत्तीय बेलन की ऊँचाई , उसके आधार के व्यास के बराबर है।
हल-
माना बेलन की ऊँचाई h तथा आधार की त्रिज्या r है।
पुनः माना बेलन का सम्पूर्ण पृष्ठ S और आयतन V है, तब
UP Board Solutions for Class 12 Maths Chapter 6 Application of Derivatives image 131
UP Board Solutions for Class 12 Maths Chapter 6 Application of Derivatives image 132

प्रश्न 21.
100 सेमी3 आयतन वाले डिब्बे सभी बेलनाकार (लम्ब वृत्तीय) डिब्बों में से न्यूनतम पृष्ठ क्षेत्रफल वाले डिब्बे की विमाएँ ज्ञात कीजिए।
हल-
माना बेलनाकार डिब्बों की त्रिज्या r और ऊँचाई h है।
आयतन = πr²h = 100 सेमी3
UP Board Solutions for Class 12 Maths Chapter 6 Application of Derivatives image 133

प्रश्न 22.
28 मीटर लम्बे तार के दो टुकड़े करके एक को वर्ग तथा दूसरे को वृत्त के रूप में मोड़ा जाता है। दोनों टुकड़ों की लम्बाई ज्ञात कीजिए यदि उनसे बनी आकृतियों को संयुक्त क्षेत्रफल न्यूनतम है।
हल-
तार की लम्बाई l = 28 मी
माना वर्ग की भुजा x तथा वृत्त की त्रिज्या r है, तब
l = वर्ग का परिमाप + वृत्त की परिधि = 4x + 2πr = 28 …(1)
माना संयुक्त क्षेत्रफल A है।
A = वर्ग की क्षेत्रफल + वृत्त का क्षेत्रफल = x² + πr²
UP Board Solutions for Class 12 Maths Chapter 6 Application of Derivatives image 134

UP Board Solutions

प्रश्न 23.
सिद्ध कीजिए कि R त्रिज्या के गोले के अन्तर्गत विशालतम शंकु का आयतन गोले के आयतन का [latex ]\frac { 8 }{ 27 }[/latex] होता है।
हल-
माना V, AB गोले के अन्तर्गत विशालतम शंकु का आयतन है। स्पष्टतया अधिकतम आयतन के लिए शंकु का अक्ष गोले की ऊँचाई के साथ होना चाहिए।
माना ∠AOC = θ,
∴ AC, शंकु के आधार की त्रिज्या = R sin θ, जहाँ R गोले की त्रिज्या है।
UP Board Solutions for Class 12 Maths Chapter 6 Application of Derivatives image 135
UP Board Solutions for Class 12 Maths Chapter 6 Application of Derivatives image 136
UP Board Solutions for Class 12 Maths Chapter 6 Application of Derivatives image 137

प्रश्न 24.
दर्शाइये कि एक निश्चित आयतन के शंक्वाकार डेरे के बनाने में कम-से-कम कपड़ा लगेगा जब उसकी ऊँचाई आधार की त्रिज्या के √2 गुना होगी।
हल-
माना शंकु की ऊँचाई h, त्रिज्या r तथा तिरछी ऊँचाई l है।
UP Board Solutions for Class 12 Maths Chapter 6 Application of Derivatives image 138

प्रश्न 25.
सिद्ध कीजिए कि दी हुई तिर्यक ऊँचाई और महत्तम आयतन वाले शंकु का अर्द्ध शीर्ष कोण tan-1√2 होता है।
हल-
माना शंकु की त्रिज्या = r, अर्द्धशीर्ष ∠BAM = θ
ऊँचाई = h; तिर्यक ऊँचाई = l
ऊर्ध्वाधर ऊँचाई, h = AM = l cos θ
शंकु की त्रिज्या, r = MC = l sin θ
UP Board Solutions for Class 12 Maths Chapter 6 Application of Derivatives image 139
UP Board Solutions for Class 12 Maths Chapter 6 Application of Derivatives image 140
UP Board Solutions for Class 12 Maths Chapter 6 Application of Derivatives image 141

प्रश्न 26.
सिद्ध कीजिए कि दिए हुए पृष्ठ और महत्त्म आयतन वाले लम्बवृत्तीय शंकु का अर्द्धशीर्ष कोण [latex ]{ sin }^{ -1 }\left( \frac { 1 }{ 3 } \right) [/latex] होता है।
हल-
माना शंकु की त्रिज्या r, तिरछी ऊँचाई l सम्पूर्ण पृष्ठ S तथा आयतन V है।
सम्पूर्ण पृष्ठ S = πr (r + l) या πrl = S – πr²
UP Board Solutions for Class 12 Maths Chapter 6 Application of Derivatives image 142
UP Board Solutions for Class 12 Maths Chapter 6 Application of Derivatives image 143

UP Board Solutions

प्रश्न 27.
वक्र x² = 2y पर (0, 5) से न्यूनतम दूरी पर स्थित बिन्दु है
(A) (2√2, 4)
(B) ( 2√2 , 0)
(C) (0, 0)
(D) (2, 2)
हल-
माना वक्र x² = 2y पर कोई बिन्दु P(x, y) है।
दिया हुआ बिन्दु A (0, 5) है।
PA² = (x – 0)² + (y – 5)² = z (माना)
Z = x² + (y – 5)² …(1)
तथा वक्र x² = 2y …(2)
x² का मान समी० (1) में रखने पर,
Z = 2y + (y – 5)² =2y + y² + 25 – 10y = y² + 25 – 8y
दोनों पक्षों का y के सापेक्ष अवकलन करने पर, [latex ]\frac { dZ }{ dy }=2y-8[/latex]
उच्चतम व निम्नतम मान के लिए, [latex ]\frac { dZ }{ dy }=0[/latex]
UP Board Solutions for Class 12 Maths Chapter 6 Application of Derivatives image 144

प्रश्न 28.
x के सभी वास्तविक मानों के लिए!
UP Board Solutions for Class 12 Maths Chapter 6 Application of Derivatives image 145
का न्यूनतम मान है–
(A) 0
(B) 1
(C) 3
(D) [latex]\frac { 1 }{ 3 }[/latex]
हल-
UP Board Solutions for Class 12 Maths Chapter 6 Application of Derivatives image 146

प्रश्न 29.
[x (x – 1) + 1]1/3,0≤x≤1 का उच्चतम मान है
(A) [latex]{ \left( \frac { 1 }{ 3 } \right) }^{ \frac { 1 }{ 3 } }[/latex]
(B) [latex]\frac { 1 }{ 2 }[/latex]
(C) 1
(D) 0
हल-
माना y = [x (x – 1) + 1]1/3
दोनों पक्षों का x के सापेक्ष अवकलन करने पर,
UP Board Solutions for Class 12 Maths Chapter 6 Application of Derivatives image 147
UP Board Solutions for Class 12 Maths Chapter 6 Application of Derivatives image 148
उच्चतम मान = 1
अत: विकल्प (C) सही है।

We hope the UP Board Solutions for Class 12 Maths Chapter 6 Application of Derivatives (अवकलज के अनुप्रयोग) help you. If you have any query regarding UP Board Solutions for Class 12 Maths Chapter 6 Application of Derivatives (अवकलज के अनुप्रयोग), drop a comment below and we will get back to you at the earliest.

UP Board Solutions for Class 11 Geography: Fundamentals of Physical Geography Chapter 15 Life on the Earth

UP Board Solutions for Class 11 Geography: Fundamentals of Physical Geography Chapter 15 Life on the Earth (पृथ्वी पर जीवन)

These Solutions are part of UP Board Solutions for Class 11 Geography. Here we have given UP Board Solutions for Class 11 Geography: Fundamentals of Physical Geography Chapter 15 Life on the Earth (पृथ्वी पर जीवन)

पाठ्य-पुस्तक के प्रश्नोत्तर

1. बहुवैकल्पिक प्रश्न
प्रश्न (1) निम्नलिखित में से कौन जैवमण्डल में सम्मिलित हैं?
(क) केवल पौधे ।
(ख) केवल प्राणी
(ग) सभी जैव व अजैव जीव
(घ) सभी जीवित जीव
उत्तर-(घ) सभी जीवित जीव।

प्रश्न (i) उष्णकटिबन्धीय घास के मैदान निम्न में से किस नाम से जाने जाते हैं?
(क) प्रेयरी ।
(ख) स्टेपी
(ग) सवाना
(घ) इनमें से कोई नहीं
उत्तर-(ग) सवाना।।

प्रश्न (iii) चट्टानों में पाए जाने वाले लोहांश के साथ ऑक्सीजन मिलकर निम्नलिखित में से क्या बनाती है?
(क) आयरन कार्बोनेट
(ख) आयरन ऑक्साइड
(ग) आयरन नाइट्राइट
(घ) आयरन सल्फेट
उत्तर-(ख) आयरन ऑक्साइड।

प्रश्न (iv) प्रकाश-संश्लेषण प्रक्रिया के दौरान प्रकाश की उपस्थिति में कार्बन डाइऑक्साइड जल के साथ मिलकर क्या बनाती है? |
(क) प्रोटीन
(ख) कार्बोहाइड्रेट्स
(ग) एमिनो एसिड
(घ) विटामिन
उत्तर-(ख) कार्बोहाइड्रेट्स।

2. निम्नलिखित प्रश्नों के उत्तर लगभग 30 शब्दों में दीजिए
प्रश्न (i) पारिस्थितिकी से आप क्या समझते हैं?
उत्तर-‘पारिस्थितिकी’ शब्द का अंग्रेजी पर्यायवाची शब्द ‘इकोलॉजी’ (Ecology) है। ‘इकोलॉजी’ ग्रीक भाषा,के दो पदों ‘Oikos’ तथा ‘Logos’ से मिलकर बना है। Oikos का अर्थ ‘निवासस्थान’ तथा Logos का अर्थ ‘अध्ययन करना है। इस प्रकार ‘इकोलॉजी’ का शाब्दिक अर्थ ‘निवास-स्थान के अध्ययन से है। दूसरे शब्दों में, जीवों को उनके निवास स्थान के सन्दर्भ में अध्ययन करना ही पारिस्थितिकी (Ecology) कहलाता है।
विद्वानों द्वारा पारिस्थितिकी की निम्नलिखित परिभाषाएँ दी गई हैं
1. 1971 में प्रकाशित ओडम की पुस्तक ‘Fundamentals of Ecology’ में पारिस्थितिकी की एक नवीन परिभाषा निम्न प्रकार प्रस्तुत की गई है
“पारिस्थितिकी, पारिस्थितिक-तन्त्र की संरचना और क्रिया का अध्ययन है।”

अत: यह कहा जा सकता है कि पारिस्थितिकी जैविक एवं पर्यावरण के आपसी सम्बन्धों तथा अन्त:प्रभावों का अध्ययन है।

प्रश्न (ii) पारितन्त्र (Ecological System) क्या है? संसार के प्रमुख पारितन्त्र के प्रकारों को बताइए।
उत्तर-किसी विशेष क्षेत्र में किसी विशेष समूह के जीवधारियों का भूमि, जल अथवा वायु से अन्तर्सम्बन्ध जिसमें ऊर्जा प्रवाह व पोषण श्रृंखलाएँ स्पष्ट रूप से समायोजित हों, उसे पारितन्त्र कहा जाता है। पारितन्त्र मुख्यतः दो प्रकार के होते हैं-1. स्थलीय पारितन्त्र (Terrestrial) तथा जलीय पारितन्त्र (Aquatic)। स्थलीय पारितन्त्र को पुनः विभिन्न प्रकार के बायोम में विभक्त किया जाता है; जैसे-घास बायोम, वन बायोम आदि। जबकि जलीय पारितन्त्र को समुद्री पारितन्त्र व ताजे जल के पारितन्त्र में विभक्त किया जाता है।

प्रश्न (iii) खाद्य श्रृंखला क्या है? चराई खाद्य श्रृंखला का एक उदाहरण देते हुए इसके अनेक स्तर बताएँ।
उत्तर-खाद्य श्रृंखला में एक स्तर से दूसरे स्तर पर ऊर्जा प्रवाह ही खाद्य श्रृंखला (Food Chain) कहलाती है।
चराई खाद्य श्रृंखला (Grazing Food-chain) पौधों से आरम्भ होकर मांसाहारी तृतीयक उपभोक्ता तक जाती है। इसमें शाकाहारी मध्यम स्तर पर होता है। उदाहरण के लिए-पौधा/पादप → गाय/खरगोश → शेर या घास → टिड्डे → मेंढक → सर्प → बाज। चराई खाद्य श्रृंखला लघु आकारीय तथा वृहत् आकारीय दोनों होती है। जिस श्रृंखला में तीन स्तर होते हैं, उसे लघु चराई खाद्य शृंखला तथा जिसमें पाँच या इससे अधिक स्तर होते हैं उसे वृहत् चराई श्रृंखला कहा जाता है।

प्रश्न (iv) खाद्य जाल (Food web) से आप क्या समझते हैं? उदाहरण सहित बताएँ।।
उत्तर-सामान्यतः आहार श्रृंखला एक पोषण स्तर से दूसरे पोषण स्तर तक संचालित होती है, परन्तु यह एक सरल रैखिक तन्त्र नहीं है बल्कि अन्त:ग्रन्थित श्रृंखलाओं के रूप में ऊर्जा का प्रवाह जैविक एवं अजैविक संघटकों के बीच होता है। इस प्रकार एक जटिल तन्त्र की व्यवस्था विकसित होती है, जिसे आहार जाल कहा जाता है। उदाहरण के लिए-एक चूहा जो अन्न पर निर्भर है वह अनेक द्वितीयक उपभोक्ताओं का भोजन है और तृतीय मांसाहारी अनेक द्वितीयक जीवों से अपने भोजन की पूर्ति करते हैं। इस प्रकार प्रत्येक मांसाहारी जीव एक से अधिक प्रकार के शिकार पर निर्भर है, परिणामस्वरूप खाद्य श्रृंखला आपस में जुड़ी हुई है। अतः प्रजातियों के इस प्रकार जुड़े होने को ही खाद्य जाल कहा जाता है।

प्रश्न (v) बायोम क्या है?
उत्तर-बायोम पौधों व प्राणियों का एक समुदाय है जो एक बड़े भौगोलिक क्षेत्र में पाया जाता है। संसार के कुछ प्रमुख उदाहरण हैं—वन बायोम, घास बायोम, जलीय बायोम, मरुस्थलीय बायोम तथा उच्च प्रदेशीय बायोम आदि।।

3. निम्नलिखित प्रश्नों के उत्तर लगभग 150 शब्दों में दीजिए
प्रश्न (i) संसार के विभिन्न वन बायोम (Forestbiomes) की महत्त्वपूर्ण विशेषताओं का वर्णन करें।
उत्तर-संसार के प्रमुख वन बायोम तथा उनकी विशेषताएँ निम्नलिखित हैं
1. उष्णकटिबन्धीय वन-ये वन दो प्रकार के होते हैं–(i) उष्णकटिबन्धीय आर्द्र वर्षा वन तथा
(ii) उष्ण कटिबन्धीय पर्णपाती वन।। उष्णकटिबन्धीय वर्षा वन भूमध्य रेखा के समीप मिलते हैं। इन वनों में तापमान 25° से० के लगभग रहता है। यहाँ वर्षा 200 सेमी से अधिक होती है तथा तापान्तर कम रहता है। इन वृक्षों की लम्बाई 25 से 30 मीटर होती है। इन वनों में सघनता अधिक पाई जाती है।

2. शीतोष्ण कटिबन्धीय वन-ये वन मध्य अक्षांशों में उत्तरी अमेरिका, उत्तरी-पूर्वी एशिया तथा पश्चिमी और मध्य यूरोप में पाए जाते हैं। इन वनों के क्षेत्र में तापमान 30° से० तथा वर्षा 75 से 150 सेमी तक रहती है। शीतोष्ण कटिबन्धीय वनों में ओक, बीच, मैपल, हेमलोक आदि वृक्ष मिलते हैं।

3. टैगा वन-टैगा-वनों का विस्तार 50° से 60° उत्तरी अक्षांशों में मिलता है। ये वन उत्तरी यूरेशिया, उत्तरी अमेरिका तथा साइबेरिया में विस्तृत हैं। इन वनों को कोणधारी वन भी कहते हैं। वृक्षों की पत्तियाँ नुकीली होती हैं। इनमें पाईने, फर तथा स्थूस प्रमुख वृक्ष हैं। इस क्षेत्र में तापमान बहुत कम रहता है तथा वर्षा बर्फ के रूप में होती है।

प्रश्न (ii) जैव भू-रासायनिक चक्र क्या है? वायुमण्डल में नाइट्रोजन का यौगिकीकरण कैसे होता है? वर्णन करें।
उत्तर-जैव भू-रासायनिक चक्र
जैवमण्डल में जीवधारी व पर्यावरण के बीच में रासायनिक तत्त्वों के चक्रीय प्रवाह को जैव भू-रासायनिक चक्र कहते हैं। यह चक्र जीवों द्वारा रासायनिक तत्त्वों के अवशोषण से आरम्भ होता है। जिसमें वायु, जल व मिट्टी में विघटन से इसकी पुनरावृत्ति होती रहती है, जिसमें रासायनिक तत्त्वों का सन्तुलन पौधों व प्राणी ऊतकों के चक्रीय प्रवाह द्वारा बना रहता है।

वायुमण्डल में नाइट्रोजन (79%) एक प्रमुख गैस है। कुछ जीव इसका उपयोग स्वतन्त्र रूप से वायु द्वारा करते हैं, जबकि कुछ जीव प्रत्यक्ष रूप से इसे ग्रहण करने में असमर्थ रहते हैं। वायु में स्वतन्त्र रूप में पाई जाने वाली नाइट्रोजन को मृदा जीवाणु व नील-हरित शैवाल प्रत्यक्ष रूप से ग्रहण कर लेते हैं। किन्तु सामान्यतया नाइट्रोजन यौगिकीकरण द्वारा ही प्रयोग में लाई जाती है। स्वतन्त्र नाइट्रोजन का प्रमुख स्रोत मिट्टी के सूक्ष्म जीवाणुओं की क्रिया से सम्बन्धित पौधों की जड़ों व रंध्रों वाली मृदा है जहाँ से यह वायुमण्डल में पहुँचती है। वायुमण्डल में नाइट्रोजन का यौगिकीकरण बिजली चमकने व कोसमिक रेडिएशन द्वारा होता है, किन्तु महासागरों में इस यौगिकीकरण में जलीय जीवों का महत्त्वपूर्ण स्थान है।

प्रश्न (iii) पारिस्थितिक सन्तुलन क्या है? इसके असन्तुलन को रोकने के महत्त्वपूर्ण उपायों की चर्चा करें।
उत्तर-किसी पारितन्त्र या आवास में जीवों के समुदाय में परस्पर गतिक साम्यता की अवस्था ही पारिस्थितिक सन्तुलन कहलाती है। यह तभी सम्भव है, जब जीवधारियों की विविधता अपेक्षाकृत स्थायी रहे। इसे पारितन्त्र में हर प्रजाति की संख्या के एक स्थायी सन्तुलन के रूप में भी वर्णित किया जा सकता है। यह सन्तुलन निश्चित प्रजातियों में प्रतिस्पर्धा व आपसी सहयोग से होता है। कुछ प्रजातियों के जीवित रहने के संघर्ष से भी पर्यावरण सन्तुलन प्राप्त किया जा सकता है। पारिस्थितिक सन्तुलने इस बात पर भी निर्भर करता है कि कुछ प्रजातियाँ अपने भोजन व जीवित रहने के लिए दूसरी प्रजातियों पर निर्भर रहती हैं, जिससे प्रजातियों की संख्या निश्चित रहती है और सन्तुलन बना रहता है; जैसे—विशाल घास के मैदानों में शाकाहारी जीव अधिक संख्या में होते हैं और मांसाहारी जीव अधिक नहीं होते हैं, अत: इनकी संख्या नियन्त्रित रहती है।

पारिस्थितिक असन्तुलन को रोकने के उपाय-पारिस्थितिक असन्तुलन को रोकने के मुख्य उपाय निम्नलिखित हैं

  1. वृक्षारोपण में वृद्धि करना।
  2. वन्य पशुओं का संरक्षण एवं इनके शिकार पर प्रतिबन्ध लगाना।
  3. झूमिंग कृषि पद्धति पर प्रतिबन्ध लगाना।
  4. निर्वनीकरण को नियन्त्रित करना।
  5. मनुष्य की जीवन शैली में ऐसा परिवर्तन लाना जिससे पर्यावरण हस्तक्षेप में वह कमी आए तथा पर्यावरण संरक्षण के प्रति सतर्क हो सके।
  6. जनसंख्या वृद्धि पर नियन्त्रण।।

परीक्षोपयोगी प्रश्नोत्तर

बहुविकल्पीय प्रश्न
प्रश्न 1. पारिस्थितिक तन्त्र के सम्बन्ध में निम्नलिखित में से कौन-सा कथन सत्य है?
(क) यह एक संवृत तन्त्र है।
(ख) सम्पूर्ण जैवमण्डल एक पारिस्थितिक तन्त्र है।
(ग) मानव द्वारा निर्मित कार्यात्मक तन्त्र है।
(घ) प्रदूषण वृद्धि तन्त्र है।
उत्तर-(ख) सम्पूर्ण जैवमण्डल एक पारिस्थितिक तन्त्र है।

प्रश्न 2. निम्नलिखित में से सर्वोच्च या अन्तिम उपभोक्ता है
(क) चीता
(ख) शेर
(ग) बाज
(घ) ये सभी
उत्तर-(घ) ये सभी।

प्रश्न 3. निम्नलिखित में अपघटक जीव है ।
(क) कवक
(ख) जीवाणु
(ग) मृतोपजीवी
(ध) ये सभी
उत्तर-(घ) ये सभी।

प्रश्न 4. पारिस्थितिक तन्त्र की कार्यप्रणाली निर्भर करती है
(क) उपभोक्ता पर ।
(ख) स्वपोषित पर ।
(ग) वियोंजक पर
(घ) ऊर्जा प्रवाह पर ।
उत्तर-(क) उपभोक्ता पर।।

अतिलघु उत्तरीय प्रश्न

प्रश्न 1. पारिस्थितिक तन्त्र की दो विशेषताओं का उल्लेख कीजिए।
उत्तर-1. पारिस्थितिक तन्त्र एक क्रियाशील इकाई है, जिसमें जैव तथा अजैव तत्त्व परस्पर एक-दूसरे को प्रभावित करते हैं। इस तन्त्र की सक्रियता से ही जैव तत्त्व उत्पादित होते हैं।
2. पारिस्थितिक तन्त्र ऊर्जा (सूर्य ऊर्जा) द्वारा संचालित होता है तथा अपनी कार्यप्रणाली द्वारा अन्य | तत्त्वों में ऊर्जा का प्रवाह करता है।

प्रश्न 2. पारिस्थितिकी तन्त्रों के दो प्रमुख घटकों के नाम लिखिए।
उत्तर-पारिस्थितिकी तन्त्र के दो प्रमुख घटकों के नाम हैं—(i) अजैविक घटक, (ii) जैविक घटक।

प्रश्न 3. स्थलीय पारितन्त्र के घटकों का उल्लेख कीजिए।
उत्तर-स्थलीय. पारितन्त्र में वन, घास के मैदान, मरुस्थल आदि आते हैं।

प्रश्न 4. स्वच्छ जलीय एवं सागरीय पारितन्त्र में क्या अन्तर है।
उत्तर-स्वच्छ जलीय पारितन्त्र नदी, झील, तालाब आदि से मिलकर बनता है, जो प्रायः मीठे जल को धारण करते हैं। इसके विपरीत सागरीय पारितन्त्र खारे पानी से युक्त सागरों एवं महासागरों से मिलकर बना होता है।

प्रश्न 5. पारिस्थितिक असन्तुलन को परिभाषित कीजिए।
या पारिस्थितिकीय असन्तुलन क्या है ?
उत्तर-पारिस्थितिक-तन्त्र के किसी भी घटक का वांछित एवं आवश्यक मात्रा से कम हो जाना अथवा अधिक हो जाना पारिस्थितिक असन्तुलन कहलाता है। प्रत्येक घटक का उस अनुपात में रहना जिससे इस तन्त्र के अन्य घटकों पर कोई हानिकारक प्रभाव न हो, पारिस्थितिक सन्तुलन कहलाता है।

प्रश्न 6. जैवमण्डल से आप क्या समझते हैं?
उत्तर-स्थल, जल और वायुमण्डल की सम्मिलित संकीर्ण पेटी जैवमण्डल कहलाती है। इस पेटी के अन्तर्गत विभिन्न प्रकार के जीव-जन्तु, पेड़-पौधे, पशु-पक्षी, कीड़-मकोड़े, सूक्ष्म जीवाणु, मछली आदि सम्मिलित हैं। इन जीवों का आकार सूक्ष्म जीवणु से लेकर विशालकाय सील व ह्वेल मछली तथा कल्लक से लेकर बरगद के विशाल वृक्ष तक होता है।

प्रश्न 7. प्रकाश-संश्लेषण क्या है?
उत्तर-प्रकाश-संश्लेषण वह प्रक्रिया है, जिसके अन्तर्गत पेड़-पौधे वायुमण्डल से कार्बन डाइऑक्साइड और मिट्टी से खनिज एवं जल लेकर सौर ऊर्जा द्वारा जैव पदार्थों का संश्लेषण करते हैं। पेड़-पौधों की पत्तियों में व्याप्त पर्णहरित (Chlorophyll) नामक हरे वर्णक द्वारा प्रकाश-संश्लेषण सम्भव होता है।

प्रश्न 8. प्राथमिक उपभोक्ता किन्हें कहते हैं?
उत्तर-जे जीव जो अपने भोजन के लिए पेड़-पौधों, घास, तृणमूल आदि पर आश्रित रहते हैं, प्राथमिक या शाकाहारी उपभोक्ता कहलाते हैं। उदाहरण के लिए-हिरन एवं खरगोश।

प्रश्न 9. सर्वाहारी या सर्वभक्षी उपभेक्ता से क्या अभिप्राय है?
उत्तर-वे जीव जो जीव-जन्तुओं और पेड़-पौधों दोनों से ही अपना भोजन प्राप्त करते हैं, सर्वाहारी या सर्वभक्षी उपभोक्ता कहलाते हैं। मनुष्य इसका उत्तम उदाहरण है, क्योंकि वह शाकाहारी एवं मांसाहारी दोनों की प्रकार का भोजन ग्रहण करर्ता है।

प्रश्न 10. अपघटक या विघटक से क्या तात्पर्य है?
उत्तर-कुछ ऐसे जीव, जो सड़े-गले पौधों तथा मृत जीव-जन्तुओं के ऊतकों का विघटने या अपचयन कर अपना भोजन बना लेते हैं, अपघटक या अपरदभोजी उपभोक्ता कहलाते हैं। जीवाणु, कवक, दीमक, केंचुए और मैगट ऐसे ही विघटक जीव हैं।

प्रश्न 11, खाद्य-श्रृंखला किसे कहते हैं?
उत्तर-मानव सहित सभी जीव-जन्तु अपनी भोजन सम्बन्धी आवश्यकता-पूर्ति के लिए एक-दूसरे पर निर्भर हैं। उदाहरण के लिए-हिरन, खरगोश, भेड़, बकरी आदि जीव पेड़-पौधों से अपना भोजन प्राप्त करते हैं, परन्तु लोमड़ी खरगोश को और शेर हिरन को खा जाता है। इस प्रकार घास (पौधों) से खरगोश में, खरगोश से लोमड़ी में तथा लोमड़ी से शेर में ऊर्जा का प्रवाह होता है। इस प्रकार ऊर्जा का प्रवाह या स्थानान्तरण खाद्य श्रृंखला कहलाता है।

प्रश्न 12. पारिस्थितिक क्षमता से क्या अभिप्राय है?
उत्तर-एक पोषण स्तर से दूसरे पोषण स्तर में स्थानान्तरित ऊर्जा की मात्रा को पारिस्थितिक क्षमता कहते हैं, परन्तु एक पोषण स्तर से दूसरे पोषण स्तर में ऊर्जा के स्थानान्तरण में भिन्नता पाई जाती है। यह क्षमता 5 से 20 प्रतिशत के मध्य पाई जाती है।

प्रश्न 13. ज्वारनदमुख पारितन्त्र से क्या अभिप्राय है?
उत्तर-नदी एवं सागर का वह मिलन स्थल, जहाँ खारे एवं मृदुल जल का मिश्रण होता है, ज्वारनदमुख पारितन्त्र कहलाता है। इस क्षेत्र में पौधों का विकास तेजी से होता है जिसमें जीवों को भोजन की प्राप्ति होती रहती है।

प्रश्न 14. तृतीयक उपभोक्ता किन्हें कहते हैं?
उत्तर-वे मांसाहारी प्राणी जो अन्य मांसाहारी प्राणियों को खाते हैं, तृतीयक उपभोक्ता कहलाते हैं; जैसे–साँप मेंढक को तथा बाज या गिद्ध साँप को खा जाता है।

प्रश्न 15. अपघटक से क्या तात्पर्य है?
उत्तर-अपघटक वे मृतोपजीवी जीवाणु या कवक आदि होते हैं जो पेड़-थौधों एवं जीव-जन्तुओं तथा कार्बनिक पदार्थों को सड़ा-गलाकर एवं विघटित करके सूक्ष्म एवं सरल कार्बनिक एवं अकार्बनिक यौगिकों में बदल देते हैं।

प्रश्न 16. शीतलन प्रणाली में कौन-सी गैस का उपयोग किया जाता है?
उत्तर-शीतलन प्रणाली में फ्रेओन तथा क्लोरोफ्लोरो कार्बन गैस का उफ्यौगं किया जाता है।

प्रश्न 17, पारिस्थितिकी (Ecology) का क्या अर्थ है?
उत्तर-पारिस्थितिकी; पारिस्थितिक विज्ञान की वह शाखा है जो विभिन्न प्रकार के जीवों तथा उनके औतिक पर्यावरण के अन्तर्सम्बन्धों का अध्ययन करती है।

प्रश्न 18. पारिस्थितिक सन्तुलन से आप क्या समझते हैं?
उत्तर-प्राकृतिक पर्यावरण के अन्तर्गत जैवमंण्डल में विभिन्न जीवों के बीच पूर्ण रूप से निर्मित सन्तुलन को ‘पारिस्थितिक सन्तुलन’ कहा जाता है।

प्रश्न 19. लियानास किसे कहते हैं?
उत्तर-वृक्षों से आवृत्त जंगल का वह क्षेत्र जहाँ एक छाता जैसा स्वरूप बन जाता है और नीचे के पौधे. बेल आदि सूर्य के प्रकाश को रोकते हैं, ऐसे क्षेत्र को लियानास कहा जाता है।

प्रश्न 20. जूप्लैंकटन क्या है?
उत्तर-वह सूक्ष्म जीव जो महासागरीय जल में पाए जाते हैं, जूप्लैंकटने कॅहलाते हैं।

प्रश्न 21. जीवोम या बायोम का क्या अर्थ है?
उत्तर-भूपृष्ठ पर जलवायु-दशाओं के अनुसार भिन्न-भिन्न प्रकार की वनस्पति पाई जाती हैं। एकसमन जलवायु-दशाओं वाले भागों में पेड़-पौधों के समुदायों के पृथक्-पृथकै समूह तथा विशेष प्रकार के जीव-जन्तु पाए जाते हैं, जिन्हें जीवोम (Biome) कहते हैं।

प्रश्न 22. सर्वाहारी (सर्वभक्षी) जीव भोजन के लिए किस पार निर्भर करते हैं?
उत्तर-सर्वाहारी (सर्वभक्षी) जीव भोजन के लिए पेड़-पौधों तथा जीव-जन्तुओं दोनों पर निर्भर करते हैं।

प्रश्न 23. उत्पादक या स्वपौषी जीव से क्या अभिप्राय है?
उतर-वे जीव जो भौतिक पर्यावरण से अपना भोजन स्वयं बना लेते हैं, उत्पादक या स्वपोषी जीव कहलाते हैं। हरे पेड़-पौधे एवं सभी प्रकार की वनस्पति प्राथमिक उत्पादकं हैं।

प्रश्न 24. जीवोम को प्रभावित करने वाले कारकों के नाम बताइए।।
उत्तर-जीवोम को प्रभावित वाले कारकों के नाम हैं—आर्द्रता, तापमान, मिट्टीं, उच्चावच, सूर्य-प्रकाश, एवं सागरीय जल तथा उसका उच्चावच।

प्रश्न 25. जैविक घटक किस पर निर्भर करते हैं।
उत्तर-जैविक घटक सर्वाधिक जलवायु पर निर्भर होते हैं। जलवायु प्रणियों और पौधों की क्रियाओं कों नियन्त्रित करती है। अत: स्थल, जल और वायुमण्डल जैविक घटकों के निर्धारक तत्त्व हैं।

प्रश्न 26. जैवमण्डल में असन्तुलन की स्थिति क्यों उत्पन्न लेती है।
उत्तर-जैवमण्डल की रचना जैविक (पादप, मानव, जन्तु एवं सूक्ष्म जीवं) तथा अजैविक घटक (स्थल, जल एवं वायु) तथा ऊर्जा से होती है। इन सभी तत्त्वों के घट-बढ़ जाने से जैक्मण्डल में असन्तुलने की स्थिति उत्पन्न हो जाती है।

प्रश्न 27, डीटटस पोषक क्या हैं?
उत्तर-उपभोक्ताओं का वह समूह जो महासागरीय जल अथवा मृत प्राणियों पर निर्भर हो, ड्रीट्टस पोषक कहलाता है।

प्रश्न 28. जीरोफाइट्स क्या हैं?
उत्तर-जो पौधे शुष्क जलवायु में भी रह सकते हैं, उन्हें जीरोफाइट्स कहते हैं।

प्रश्न 29. जैविक तत्त्वों के तीन वर्ग कौन-से हैं?
उत्तर-जैविक तत्त्वों के तीन वर्ग निम्नलिखित हैं

  • उत्पादक,
  • उपभोक्ती तथा
  • अपघटक।

प्रश्न 30. कार्बन चक्र क्या हैं?
उत्तर-कार्बन चक्र कार्बन डाइऑक्साइड का परिवर्तित रूप है।।

लघु उत्तरीय प्रश्न

प्रश्न 1. जैवमण्डल का अर्थ एवं उसके मुख्य तत्त्व बताइए।
या जैवमण्डल पर टिप्पणी लिखिए।
उत्तर-जैवमण्डल में पृथ्वी के निकट का वह कटिबन्ध सम्मिलित है जो किसी-न-किसी रूप में जैव विकास के लिए अनुकूल पड़ता है। इसका निर्माण स्थलमण्डल, जलमण्डल और वायुमण्डल तीनों के सम्पर्क क्षेत्र में होता है। इन तीनों के संयोग से ऐसा पर्यावरण बन जाता है जो वनस्पति जगत, जीव-जन्तु और मानव-शरीर के विकास के लिए अनुकूल दशाएँ प्रदान करता है। पृथ्वी तल के निकट स्थित यह क्षेत्र हो । जैवमण्डल (Biosphere) कहलाता है। विद्वानों ने जैवमण्डल को तीन पर्यावरणीय उपविभागों में बाँटा है–(i) महासागरीय, (ii) ताजे जल एवं (iii) स्थलीय जैवमण्डल। इनमें स्थलीय जैवमण्डल अधिक महत्त्वपूर्ण है।

जैवमण्डल के तत्त्व-जैवमण्डल के तीन प्रमुख तत्त्व हैं-1. वनस्पति के विविध प्रकार, 2. जन्तुओं के विविध प्रकार तथा 3. मानव समूह।। वनस्पति-जगत में समुद्री पेड़-पौधों से लेकर पर्वतों की उच्च श्रेणियों तक पाए जाने वाले वनस्पति के विविध प्रकार सम्मिलित हैं। जन्तु-जगत में समुद्रों में पाए जाने वाले विविध जीव, मिट्टियों को बनाने वाले बैक्टीरिया और स्थल पर पाए जाने वाले विविध जीव-जन्तु सम्मिलित हैं। जैवमण्डल के तत्त्व वायु, जल, सूर्य-प्रकाश और मिट्टियों पर प्रत्यक्ष व अप्रत्यक्ष रूप से निर्भर होते हैं। जैवमण्डल के तत्त्वों में परस्पर गहरा सम्बध होता है। किसी तत्त्व में कमी या अवरोध उत्पन्न होने पर जैवमण्डल पर बहुत गहरा प्रभाव पड़ता है।

प्रश्न 2. प्रथम एवं द्वितीय श्रेणी के उपभोक्ताओं में अन्तर बताइए।
उत्तर-प्राथमिक एवं द्वितीयंक श्रेणी के उपभोक्ताओं में निम्नलिखित अन्तर हैं
UP Board Solutions for Class 11 Geography Fundamentals of Physical Geography Chapter 15 Life on the Earth (पृथ्वी पर जीवन) img 1

प्रश्न 3. पारिस्थितिक सन्तुलन की महत्ता को स्पष्ट कीजिए।
उत्तर-पारिस्थितिक तन्त्र जीवन का आधार है। इसका मूल उद्देश्य मानव एवं प्रकृति के मध्य मधुर सम्बन्ध स्थापित करना है, परन्तु जनसंख्या में द्रुतगति से वृद्धि के कारण प्राकृतिक संसाधनों का तीव्र गति से विदोहन हुआ है तथा पारिस्थितिक असन्तुलन के कारण जीवन के लिए संकट की स्थिति उत्पन्न होने का खतरा बढ़ा है। समग्र रूप से पारिस्थितिक सन्तुलन की महत्ता को निम्नलिखित रूपों में व्यक्त किया जा सकता है ।

1. पारिस्थितिक सन्तुलन के कारण ही वायुमण्डल एवं जलमण्डल के परिसंचरण से जलवायु सन्तुलित रहती है तथा जल एवं वन संसाधनों का भण्डार सतत बना रहता है।

2. पारिस्थितिक सन्तुलन के कारण प्राकृतिक जैव एवं अजैव घटकों की स्वनिर्मित प्रक्रिया में कोई अवरोध उत्पन्न नहीं होता, जिसके कारण प्राकृतिक आपदाओं की सम्भावनाएँ न्यूनतम रहती हैं।

3. पर्यावरण सन्तुलन पृथ्वी पर जीव-जन्तु और पेड़-पौधों में एक निश्चितै अनुपात कायम रखता है,

अत: जीवन की सतत साम्यावस्था बनी रहती है। इस प्रकार पारिस्थितिक सन्तुलन को जीवन में व्यापक महत्त्व है। इसकी साम्यावस्था मानव-जीवन के लिए ही नहीं, बल्कि अन्य जीव-जन्तु, पेड़-पौधों और समस्त अजैव संसाधनों की दीर्घ अवधि तक उपभोग क्षमता में वृद्धि के लिए अत्यन्त आवश्यक है।

प्रश्न 4. पर्यावरण किस प्रकार सन्तुलित रह सकता है?
उत्तर-जैव समुदाय में वृद्धि, विकास एवं अस्तित्व के लिए पर्यावरण का सन्तुलित होना आवश्यक है। मानव के सभी क्रियाकलाप पर्यावरण से सम्बन्धित होते हैं तथा उसी से ही निर्धारित होते हैं। मानव का आवास इसी पृथ्वी तल पर है। वह पृथ्वी तल पर उत्पन्न होने वाली वनस्पति तथा जीव-जन्तुओं से अलग नहीं रह सकता है, क्योंकि अपने भोजन और अन्य आवश्यकताओं की आपूर्ति के लिए वह वनस्पति एवं जीव-जन्तुओं पर ही आश्रित है; अतः मानव के लिए इनकी सुरक्षा करना अति आवश्यक हो जाता है। पर्यावरण सन्तुलन के लिए आज पर्यावरण के प्रति जागरूक होना अथवा पर्यावरण का बोध होना अति आवश्यक है।

पारिस्थितिक सन्तुलन तभी बना रह सकता है जब प्रत्येक घटक सम्मिलित रूप से सभी क्रियाएँ करता रहे, जो वह पहले से करता आ रहा है; जैसे—यदि वन पर्याप्त मात्रा में बने रहते हैं तो इससे वातावरण एवं वायु में नमी बनी रहती है, जिससे वर्षा होती रहेगी, कृषि फसलों के लिए जल मिलता रहेगा, फलस्वरूप खाद्य-पदार्थों का उत्पादन होगा और भोजन की कमी नहीं रहेगी। वास्तव में पारिस्थितिक सन्तुलन के लिए इसके प्रत्येक घटक का सन्तुलित अवस्था में रहना आवश्यक है।

प्रश्न 5. पारिस्थितिक तन्त्र को परिभाषित कीजिए।
उत्तर-भौतिक पर्यावरण में पेड़-पौधे, जीव-जन्तु तथा अन्य सूक्ष्म जीवाणु सब एक साथ मिलकर पारितन्त्र की रचना करते हैं। “पारिस्थितिकी जीवविज्ञान का वह भाग है जिसके द्वारा हमें जीव तथा पर्यावरण की पारस्परिक प्रतिक्रियाओं का बोध होता है। इस प्रकार विभिन्न जीवों के पारस्परिक सम्बन्धों तथा उनका भौतिक पर्यावरण से सम्बन्धों का अध्ययन पारिस्थितिक विज्ञान (Ecology) के अन्तर्गत किया जाता है। हमारी पृथ्वी स्वयं में एक बहुत बड़ा पारिस्थितिक तन्त्र है, जिसमें समस्त जैव समुदाय सूर्य द्वारा प्राप्त ऊर्जा पर निर्भर है तथा वे स्थलमण्डल, जलमण्डल तथा वायुमण्डल से जीवनोपयोगी सभी तत्त्वों को प्राप्त करते हैं। जलवायु प्राणियों और पौधों की क्रियाओं को नियन्त्रित करती है। ये दोनों ही एक-दूसरे को तथा साथ ही अपने पर्यावरण को भी प्रभावित करते हैं। इस प्रकार पर्यावरण और उसमें निवास करने वाले जीवधारी परस्पर एक-दूसरे को प्रभावित करते हुए एक तन्त्र की रचना कर लेते हैं, जिसे ‘पारितन्त्र’ अथवा ‘पारिस्थितिक तन्त्र’ कहा जाता है।

प्रश्न 6. पारितन्त्र के प्रमुख घटकों का वर्णन कीजिए।
उत्तर-पारितन्त्र के निम्नलिखित दो प्रमुख घटक होते हैं
1. अजैव घटक-मृदा, जल और वायुमण्डल में विद्यमान अनेक रासायनिक पदार्थ अजैव घटक कहलाते हैं। इन रासायनिक पदार्थों में जल, ऑक्सीजन, हाइड्रोजन, नाइट्रोजन, कार्बन डाइ-ऑक्साइड, कैल्सियम, फॉस्फोरस तथा अन्य अनेक रासायनिक पदार्थ सम्मिलित किए जाते हैं। भौतिक पर्यावरण के अजैव घटक किसी क्षेत्र में निवास करने वाले जीव-जन्तुओं तथा वनस्पति की विभिन्न प्रजातियों को प्रभावित करते हैं। अजैव घटकों में जलवायु का महत्त्वपूर्ण स्थान है, जो सम्पूर्ण पारितन्त्र को प्रभावित करती है तथा उसमें अनेक परिवर्तन लाती है।

2. जैव घटक-स्थल, जल और वायुमण्डल में निवास करने वाले सभी प्रकार के जीव-जन्तु, जीवाणु, कीटाणु आदि तथा सभी प्रजातियों के पेड़-पौधे (वनस्पति) जैव घटक के अन्तर्गत सम्मिलित किए जाते हैं।

प्रश्न 7. जैव घटक के दो प्रमुख वर्ग कौन-कौन से हैं? वर्णन कीजिए।
उत्तर-जैव घटक के दो प्रमुख वर्ग निम्नलिखित हैं
1. उत्पादक-उत्पादक वे जीव हैं जो भौतिक पर्यावरण से अपना भोजन स्वयं लेते हैं। इन्हें स्वपोषित जीव भी कहते हैं। हरे पेड़-पौधे तथा सभी प्रकार की वनस्पति प्राथमिक उत्पादक हैं। महासागरीय जल में पादप प्लवक प्राामिक उत्पादक हैं, क्योंकि वे सौर ऊर्जा का उपयोग कर अपना भोजन स्वयं बना लेते हैं।

2. उपभोक्ता-उपभोक्ता अपने भोजन के लिए अन्य जीवों पर निर्भर रहते हैं। इन्हें परपोषी भी कहा जाता है। इनकी चार श्रेणियाँ हैं
(क) शाकाहारी या प्राथमिक उपभोक्ता हिरण एवं खरगोश।
(ख) मांसाहारी या गौण उपभोक्ता–शेर एवं चीता।
(ग) सर्वाहारी या सर्वभक्षी उपभोक्ता—मनुष्य।
(घ) अपघटक या अपरदभोजी उपभोक्ता–जीवाणु, कवक, दीमक, केंचुए एवं मैगट आदि।।

इस प्रकार अपघटक जीव, जैव पदार्थों को अजैव पदार्थों में परिणत कर देते हैं। पुनः इन अजैव पदार्थों को सौर ऊर्जा की सहायता से पेड़-पौधे अपना भोजन बना लेते हैं। हिरण एवं खरगोश पेड़-पौधों से अपना भोजन प्राप्त करते हैं, जबकि शेर एवं चीता, हिरण एवं खरगोश को खा जाते हैं। मनुष्य अपना भोजन पेड़-पौधों एवं गौण उपभोक्ताओं से प्राप्त करता है। इस प्रकार यह क्रम अबाध गति से चलता रहता है तथा चक्रीय प्रक्रिया पूर्ण हो जाती है।

प्रश्न 8. खाद्य-श्रृंखला या आहार-जाल किसे कहते हैं?
उत्तर-मानव सहित सभी जीव अपनी भोजन सम्बन्धी आवश्यकताओं की आपूर्ति के लिए एक-दूसरे पर निर्भर करते हैं तथा उनमें भोजन के लिए कड़ी प्रतिस्पर्धा रहती है। इस प्रकार पारिस्थितिक तन्त्र में एक जीव से दूसरे जीव में ऊर्जा का स्थानान्तरण’खाद्य-श्रृंखला’ कहलाता है। उदाहरण के लिए-खरगोश, हिरण, भेड़, बकरी आदि जीव घास (पौधों) से अपना भोजन प्राप्त करते हैं, परन्तु लोमड़ी खरगोश को खा जाती है। और शेर लोमड़ी को खा जाता है। परन्तु विघटक पौधों और जीवों के सड़े-गले अंश से ऊर्जा और पोषक तत्त्व प्राप्त करते हैं। ये जैव पदार्थों को अजैव पदार्थों में बदल देते हैं जिन्हें हरे पौधे ग्रहण कर लेते हैं। इस प्रकार खाद्य–श्रृंखला का चक्र पूर्ण हो जाता है। परन्तु अपघटक पौधों एवं मृत शरीरों के ऊतकों से ऊर्जा और पोषक तत्त्वों को ग्रहण करते हैं। अपने भोजन की प्रक्रिया में अपघटक जीव, जैव पदार्थों को अजैव पदार्थों में परिणत कर देते हैं। इस प्रकार प्रकृति में खाद्य-श्रृंखलाएँ जटिल बन जाती हैं तथा इनका एक जाल-सा बन जाता है। जीवों द्वारा पारस्परिक रूप से सम्बन्धित खाद्य-श्रृंखलाओं के जटिल समूह को आहार-जाल कहते हैं।

प्रश्न 9. पृथ्वी के पारितन्त्र को कितने भागों में विभाजित किया जाता है?
उत्तर-पृथ्वी के पारितन्त्र को. निम्नलिखित दो भागों में विभाजित किया जाता है
1. जलीय पारितन्त्र-जल में घुले विभिन्न लवणों के कारण जलीय जीवों की संख्या सीमित होती है। जलीय पारितन्त्र का उपविभाजन मीठे जल, ज्वारनदमुख तथा समुद्री पारितन्त्रों के रूप में किया जाता है। जल में घुली हुई ऑक्सीजन का संकेन्द्रण और जल में सूर्य के प्रकाश का प्रवेश तथा पोषण की उपलब्धि, जलीय जीवों को सीमित करने वाले प्रमुख कारक हैं।

2. स्थलीय पारितन्त्र-हम स्थलखण्ड पर निवास करते हैं; अत: स्थलीय पारितन्त्र से हमारा गहन सम्बन्ध है, क्योंकि हमारी भोजन तथा अन्य सभी आवश्यकताएँ इन्हीं से ही पूर्ण होती हैं। भू-पृष्ठ पर जलवायु की दशाओं के अनुसार विभिन्न प्रकार की वनस्पति पाई जाती है। एकसमान जलवायु-दशाओं वाले भागों में पौधों के समुदायों के पृथक्-पृथक् समूह मिलते हैं, जिन्हें ‘जीवोम’ कहते हैं। इस प्रकार स्थलीय पारितन्त्र का वर्गीकरण जलवायु-दशाओं के आधार पर किया जाता है। इनमें आर्द्रता, तापमान तथा मृदा महत्त्वपूर्ण कारक हैं।

प्रश्न 10. प्रकाश-संश्लेषण के महत्त्व की विवेचना कीजिए।
उत्तर-प्रकाश-संश्लेषण वह प्रक्रिया है जिसके द्वारा हरे पौधे सूर्य की ऊर्जा की सहायता से अजैव पदार्थों को जैव पदार्थों में परिवर्तित कर देते हैं। प्रकाश-संश्लेषण की प्रक्रिया में पौधे वायुमण्डल से कार्बन डाइऑक्साइड और मृदा से खनिज व जल लेकर, सूर्य की ऊर्जा द्वारा जैव पदार्थों को संश्लेषण करते हैं। पेड़-पौधों की पत्तियों में व्याप्त पर्णहरित (Chlorophyll) नामक हरे वर्णक द्वारा प्रकाश-संश्लेषण सम्भव होता है। महासागरीय जल में पादप प्लवक प्राथमिक उत्पादक हैं क्योंक वे सौर ऊर्जा का उपयोग कर अपना भोजन स्वयं बना लेते हैं।

प्रश्न 11. पारिस्थितिक पिरामिड को समझाइए।
उत्तर-जीव-जन्तुओं के प्रत्येक समूह का एक पोषण स्तर होता है। हरी घासें एवं अन्य वनस्पति प्रथम स्तर के पोषण के अन्तर्गत सम्मिलित की जाती हैं, जिन्हें प्राथमिक उत्पादक भी कहा जाता है। शाकाहारी जीव-जन्तु, जो इनका भक्षण करते हैं, द्वितीय स्तर के पोषण में सम्मिलित किए जाते हैं। वे मांसाहारी जीव-जन्तु, जो शाकाहारी जीव-जन्तुओं का शिकार करते हैं, तृतीय स्तर के पोषण में सम्मिलित होते हैं, जिन्हें द्वितीयक उपभोक्ता भी कहते हैं। चतुर्थ स्तर के पोषण में ऐसे मांसाहारी जीव सम्मिलित किए जाते हैं जो अपने से छोटे मांसाहारी जीवों का भक्षण करते हैं, इन्हें तृतीयक उपभोक्ता कहते हैं। मनुष्य तृतीयक उपभोक्ता है जो तीनों ही पोषण स्तरों का प्रत्यक्ष एवं परोक्ष रूप में उपभोग करता है, क्योंकि मनुष्य सर्वाहारी उपभोक्ता है। ऊर्जा की उपलब्धता के अनुसार सभी पोषण स्तर समान नहीं होते हैं, क्योंकि निम्न स्तर से उच्च स्तर पर ऊर्जा का एक अंश ही स्थानान्तरित होता है। इन पोषण स्तरों का प्रदर्शन एक पिरामिड की सहायता से किया जाता है, जिसे पारिस्थितिक पिरामिड कहा जाता है।

प्रश्न 12. पारिस्थितिक क्षमता का वर्णन कीजिए।
उत्तर-एक पोषी स्तर से दूसरे पोषी स्तर में स्थानान्तरित ऊर्जा के प्रतिशत को पारिस्थितिक क्षमता कहा जाता है। जीवों के एक समूह का एक पोषी स्तर होता है।

एक पोषी स्तर से दूसरे पोषी स्तर में ऊर्जा स्थानान्तरण की क्षमता भिन्न-भिन्न होती है। जीवों की जाति और पर्यावरणीय परिस्थितियों के अनुसार यह क्षमता 5% से लेकर 20% के मध्य हो सकती है। स्थलीय पारिस्थितिक तन्त्र में शाकाहारी जीवों द्वारा पादप पदार्थ के केवल 10% भाग का ही उपभोग किया जाता है। औसत रूप से केवल 10% ऊर्जा का स्थानान्तरण एक पोषी स्तर से दूसरे पोषी स्तर में होता है। इसका तात्पर्य यह है कि जीवों को 10 किग्रा मांस के उत्पादन के लिए 100 किग्रा खाद्यान्नों की आवश्यकता होती है। इस कम क्षमता का कारण यह है कि उच्च स्तर के उपभोक्ताओं को एक स्तर पर विद्यमान सभी जीव सुगमता से उपलब्ध नहीं हो पाते हैं। परभक्षी जीव उपलब्ध प्रत्येक शिकार को पकड़ नहीं पाते हैं। परभक्षियों के आक्रमण से जो जीव बच जाते हैं वे अन्ततोगत्वा काल-कवलित हो जाते हैं। तथा इन्हें विघटक खा जाते हैं। इस प्रकार उच्च पोषी स्तर के जीवों की निर्वाह करने की क्षमता भी सीमित होती है।

प्रश्न 13. पारितन्त्र में ऊर्जा और खनिज पदार्थों के प्रवाह पर प्रकाश डालिए।
उत्तर-सम्पूर्ण पारितन्त्र ऊर्जा के लिए सूर्यातप पर निर्भर करता है; अतः सभी प्रकार के पोषकों में ऊर्जा का प्रवाह सतत रूप में प्रतिपल होता रहता है। उत्पादकों को अपना भोजन बनाने के लिए सौर-विकिरण से ऊर्जा प्राप्त होती है। ऊर्जा का स्थानान्तरण उत्पांदकों से शाकाहारियों में और शाकाहारियों से मांसाहारियों में होता रहता है। इस प्रकार उत्पादकों शाकाहारियों तथा मांसाहारियों के निर्जीव या विघटित अवशेष, अपघटकों को ऊर्जा प्रदान करते हैं। अत: सूर्य से प्राप्त ऊर्जा का प्रवाह एक ही दिशा में होता रहता है तथा यह प्रवाहं तब तक जारी रहता है जब तक कि ऊर्जा विलीन नहीं हो जाती है। जीव-जन्तु भोजन से प्राप्त ऊर्जा का कुछ भाग तो पचा लेते हैं तथा शेष भाग श्वसन द्वारा ऊष्मा के रूप में बाहर निकल जाता है। मृदा से खनिज पदार्थों का पेड़-पौधों में प्रवाह उनकी वृद्धि एवं विकास में सहायक होता है। उपभोक्ता अपनी वृद्धि एवं विकास के लिए इन पोषकों का भरपूर उपयाग करते हैं। जब पेड़-पौधे और जीव-जन्तु । नष्ट अर्थात् काल-कवलित हो जाते हैं, तब जीवाणु और कवक जैसे अपघटक उन्हें अपना भोजन बना लेते हैं तथा उन्हें विघटित कर अजैव पोषकों में परिणत कर दते हैं। ये अजैव पोषक मृदा में विलीन होते रहते हैं तथा पेड़-पौधे पुन: उनका उपभोग करते हैं। इस प्रकार पारितन्त्र में खनिज पदार्थों की यह चक्रीय प्रक्रिया अबाध गति से चलती रहती है।

प्रश्न 14. ज्वारनदमुख पारितन्त्र का विवरण दीजिए।
उत्तर-नदी जब अपने मुहाने का निर्माण करती है तो उसका जल भू-सतह पर फैल जाता है। ज्वार-भाटा के समय सागरीय जलं नदी के जल को पीछे की ओर धकेल देता है। इस क्षेत्र को ज्वारनदमुख कहते हैं। इस प्रकार की नदियाँ डेल्टाओं का निर्माण नहीं करती हैं; अत: इस क्षेत्र में नदी के मृदुल जल तथा सागर के खारे जल का सम्मिश्रण होता रहता है। ज्वारनदमुख के उथला होने के कारण सूर्य भी अधःस्थल तक पहुँचता है। ज्वार-भाटा के समय इस क्षेत्र में जल का उतार-चढ़ाव होता रहता है, फलस्वरूप यहाँ पोषक तत्त्वों का मिश्रण हो जाता है। अतः इस क्षेत्र में पौधों का विकास तीव्रता से होता है, जिनसे जीवों को भोजन की प्राप्ति होती रहती है तथा यहाँ केकड़े, सीपियाँ, झींगे, मछलियाँ, जलचर एवं जलीय वनस्पति पर्याप्त मात्रा में विकसित होती हैं। कुछ विशिष्ट प्रकार की मछलियों के लिए ज्वारनदमुख सबसे सुरक्षित प्रजनन क्षेत्र होते हैं, क्योंकि जल की कम लवणता महासागरीय परभक्षियों के लिए बाधा उपस्थित करती है।

प्रश्न 15. स्थलीय पारितन्त्र को कौन-कौन से कारक प्रभावित करते हैं?
उत्तर-स्थलीय पारितन्त्र को निम्नलिखित कारकै प्रभावित करते हैं
1. आर्द्रता-पौधों की वृद्धि के लिए जल अति आवश्यक है क्योंकि पौधों की वृद्धि के लिए आवश्यक पोषक तत्त्व घुली हुई अवस्था में जड़ों के माध्यम से पत्तियों तक पहुँचते हैं। अतएव जल पौधों में पोषकों के प्रवाह का माध्यम है।

2. तापमान-प्रत्येक पौधे को अपने अंकुरण, वृद्धि, विकास, पुनरुत्पादन के लिए एक निश्चित तापमान की आवश्यकता होती है।

3. मृदा-स्थलीय पारितन्त्र में मृदा सबसे महत्त्वपूर्ण तत्त्व है, क्योंकि वह पौधों की वृद्धि का माध्यम है। मृदा की निर्माण प्रक्रिया बहुत मन्द गति से होती है तथा इस प्रक्रिया में भौतिक, रासायनिक और जैविक परिवर्तन होते हैं। मृदा की रचना में जलवायु सर्वप्रथम कारक है। जलवायु प्रदेश ही मृदा के प्रकारों का निर्धारण करते हैं।

प्रश्न 16. उत्पादक तथा उपभोक्ता में अन्तर स्पष्ट कीजिए।
उत्तर-उत्पादक तथा उपभोक्ता में अन्तर
UP Board Solutions for Class 11 Geography Fundamentals of Physical Geography Chapter 15 Life on the Earth (पृथ्वी पर जीवन) img 2

प्रश्न 17. कार्बन चक्र से आप क्या समझते हैं? सचित्र वर्णन कीजिए।
उत्तर-कार्बन चक्र कार्बन डाइऑक्साइड का परिवर्तित रूप है। परिवर्तन की यह प्रक्रिया पौधों में प्रकाश-संश्लेषण द्वारा कार्बन डाइऑक्साइड के यौगिकीकरण से आरम्भ होती है। इस प्रक्रिया से । कार्बोहाइड्रेट्स व ग्लूकोज बनता है जो कार्बनिक यौगिक; जैसे-स्टार्च, सेल्यूलोज, सुक्रोज आदि के रूप में पौधों में संचित हो जाता है। कार्बोहाइड्रेट्स का कुछ भाग सीधे पौधों की जैविक क्रिया में प्रयुक्त होता है। इस प्रक्रिया के अन्तर्गत पौधों के पत्तों व जड़ों के विघटन से कार्बन डाइऑक्साइड गैस मुक्त होती है तथा शेष कार्बोहाइड्रेट्स जो पौधों की जैविक क्रियाओं में प्रयुक्त नहीं होती वह पौधों के ऊतकों में एकत्र हो जाती है। ये पौधे या तो शाकाहारियों का भोजन बनते हैं या सूक्ष्म जीवों द्वारा विघटित हो जाते हैं। यही शाकाहारी जीव उपभोग किए गए कार्बोहाइड्रेट्स को कार्बन डाइऑक्साइड में परिवर्तित करते हैं और श्वसन क्रिया द्वारा वायुमण्डल में छोड़ते हैं। इसके अतिरिक्त सूक्ष्म जीवाणुओं द्वारा भी कार्बोहाइड्रेट्स ऑक्सीजन प्रक्रिया द्वारा कार्बन डाइऑक्साइड में परिवर्तित होकर पुनः वायुमण्डल में आ जाती है (चित्र 15.1)।
UP Board Solutions for Class 11 Geography Fundamentals of Physical Geography Chapter 15 Life on the Earth (पृथ्वी पर जीवन) img 2

दीर्घ उत्तरीय प्रश्न

प्रश्न 1. ऑक्सीजन चक्र अथवा नाइट्रोजन चक्र पर संक्षिप्त टिप्पणी लिखिए।
उत्तर-ऑक्सीजन चक्र
ऑक्सीजन प्रकाश-संश्लेषण क्रिया का प्रमुख सहपरिणाम है। यह कार्बोहाइड्रेट्स के ऑक्सीकरण में सम्मिलित है जिससे ऊर्जा, कार्बन डाइऑक्साइड व जल विमुक्त होते हैं।

ऑक्सीजन चक्र बहुत ही जटिल प्रक्रिया है। ऑक्सीजन बहुत-से रासायनिक तत्त्वों के सम्मिश्रण में पाई जाती है। ऑक्सीजन नाइट्रोजन के साथ मिलकर नाइट्रेट बनाती है तथा बहुत से अन्य खनिज तत्त्वों से मिलकर कई तरह के ऑक्साइड बनाती है; जैसे—आयरन ऑक्साइड, ऐलुमिनियम ऑक्साइड आदि। ऑक्सीजन की उत्पत्ति सूर्य प्रकाश-संश्लेषण प्रक्रिया के दौरान जल अणुओं के विघटन से होती है और पौधों की वाष्पोत्सर्जन प्रक्रिया के द्वारा वायुमण्डल में पहुँचती है।।

नाइट्रोजन चक्र

नाइयेजन वायुमण्डल की संरचना का प्रमुख घटक है। वायमुण्डलीय गैसों में नाइट्रोजन का योगदान सर्वाधिक (79%) है। वायु में स्वतन्त्र रूप से पाई जाने वाली नाइट्रोजन को अधिकांश जीव प्रत्यक्ष रूप से ग्रहण करने में असमर्थ होते हैं। इसे प्रत्यक्ष रूप से केवल कुछ विशिष्ट प्रकार के जीव ही गैसीय रूप में ग्रहण करते हैं जिसमें मृदा जीवाणु एवं ब्लू-ग्रीन एल्गी मुख्य हैं।

सामान्यतः नाइट्रोजन यौगिकीकरण द्वारा ही प्रयोग में लाई जाती है। वायुमण्डल में यह गैस मिट्टी के सूक्ष्म जीवाणुओं की क्रिया तथा सम्बन्धित पौधों की जड़ों व रन्ध्र वाली मृदा से वायु द्वारा पहुँचती है। वायुमण्डलीय नाइट्रोजन के इस तरह यौगिक रूप में उपलब्ध होने पर हरे पौधों में इसका स्वांगीकरण (Nitrogen assimilation) होता है (चित्र 15.2)। शाकाहारी जन्तुओं द्वारा इन पौधों के खाने पर नाइट्रोजन का कुछ भाग उनमें चला जाता है। फिर मृत पौधों व जानवरों के नाइट्रोजनी अपशिष्ट
(Excretion of Nitrogenous Wastes), मिट्टी में उपस्थित बैक्टीरिया द्वारा नाइट्राइट में परिवर्तित हो जाते हैं।
UP Board Solutions for Class 11 Geography Fundamentals of Physical Geography Chapter 15 Life on the Earth (पृथ्वी पर जीवन) img 3

प्रश्न 2. पारिस्थितिक-तन्त्र (Ecosystem) से आप क्या समझते हैं। ये कितने प्रकार के होते हैं?
या टिप्पणी लिखिए-पारिस्थितिकी-तन्त्र।
या पारिस्थितिक-तन्त्र की व्याख्या कीजिए।
उत्तर-पारिस्थितिकी
‘पारिस्थितिकी’ शब्द की व्युत्पत्ति ग्रीक भाषा के शब्द ‘OIKOs’ से हुई है, जिसका शाब्दिक अर्थ है-‘घर’ अथवा ‘आवास। अत: इस आधार पर पारिस्थितिकी का अर्थ हुआ—जीव को घर या
आवास। इस प्रकार जीव विज्ञान का वह भाग जिसके अन्तर्गत जीवों तथा उनके पर्यावरण की पारस्परिक क्रियाओं-प्रतिक्रियाओं का अध्ययन किया जाता है, पारिस्थितिकी विज्ञान कहलाता है। जीव और पर्यावरण के पारस्परिक सम्बन्धों के अध्ययन को वातावरणीय जीव विज्ञान (Environmental Biology) भी कहा जाता है। एच० रेटर (H. Reiter) ने ‘इकोलॉजी’ शब्द का प्रयोग सर्वप्रथम 1868 ई० में किया था। जीव और उसका पर्यावरण प्रकृति के जटिल एवं गतिशील घटक हैं। पर्यावरण अनेक घटकों का समूह है। ये घटक जीवों को पारस्परिक क्रियाओं द्वारा प्रभावित करते रहते हैं। पारिस्थितिकी को अनेक विद्वानों ने परिभाषित किया है, जिनमें से कुछ परिभाषाएँ निम्नलिखित हैं-

ओडम (Odum) के अनुसार, “इकोसिस्टम पारिस्थितिकी की वह आधारभूत इकाई है जिसमें जैविक और अजैविक वातावरण एक-दूसरे पर अपना प्रभाव डालते हुए पारस्परिक अनुक्रिया से ऊर्जा और रासायनिक पदार्थों के निरन्तर प्रवाह से तन्त्र की कार्यात्मक गतिशीलता बनाये रखते हैं।”

“पारिस्थितिकी प्रकृति की अर्थव्यवस्था तथा प्राणियों के अपने अजैविक तथा जैविक पर्यावरण के साथ समस्त सम्बन्धों का अध्ययन है।” -हैकल

“पारिस्थितिकी पर्यावरण के सन्दर्भ में जीवों के अध्ययन का विज्ञान है।” -वार्मिंग

इस प्रकार उपर्युक्त परिभाषाओं से निष्कर्ष निकलता है कि पारिस्थितिकी जैविक तथा पर्यावरण के पारस्परिक सम्बन्धों का अध्ययन है। वास्तव में पृथ्वीतल पर पाये जाने वाले प्राणियों तथा जैविक एवं अजैविक पर्यावरण की सम्मिलित क्रिया-प्रतिक्रिया पारिस्थितिक-तन्त्र कहलाती है।

पारिस्थितिक-तन्त्र.

‘इकोसिस्टम’ शब्द का प्रयोग सर्वप्रथम सन् 1935 में ए०जी० तांसले द्वारा किया गया था। उनके अनुसार, “पारिस्थितिक-तन्त्र पर्यावरण के सभी जीवित एवं निर्जीव कारकों के सम्पूर्ण सन्तुलन के परिणामस्वरूप बनी हुई प्रणाली है।”

विभिन्न विद्वानों द्वारा पारिस्थितिकी एवं पारिस्थितिक-तन्त्र (Ecosystem) के पर्याय शब्दों का प्रयोग
UP Board Solutions for Class 11 Geography Fundamentals of Physical Geography Chapter 15 Life on the Earth (पृथ्वी पर जीवन) img 4
कोई भी जैविक तत्त्व पर्यावरण के बिना जीवित नहीं रह सकता। उसका एक निश्चित पारिस्थितिक-तन्त्र होता है। स्वयं में हमारी पृथ्वी एक बहुत बड़ा पारिस्थितिक-तन्त्र है, जिसमें समस्त जीव समुदाय सूर्य से ऊर्जा प्राप्ति पर निर्भर करता है तथा भौतिक पर्यावरण जो भूतल पर पाया जाता है; अर्थात् स्थलमण्डल, वायुमण्डल एवं जलमण्डल में जीवनोपयोगी- समस्त तत्त्वों की प्राप्ति करता है। जलवायु जैविक तत्त्वों–प्राणी एवं पौधों के विचरण तथा उनकी क्रियाओं को नियन्त्रित करती है। प्राणी एवं पौधे स्वयं पारस्परिक रूप से पर्यावरण को भी प्रभावित करते हैं। इस प्रकार पर्यावरण और उसमें निवास करने वाले जीवधारी आपस में एक-दूसरे को प्रभावित करते हुए एक तन्त्र बना लेते हैं। यही तन्त्र ‘पारिस्थितिक-तन्त्र’ (Ecosystem) कहलाता है।

प्रकृति में कोई भी जीवधारी एवं उसका समुदाय अकेले रहकर अपनी क्रियाओं का सम्पादन नहीं कर सकता, बल्कि प्रकृति में पाये जाने वाले एवं विचरण करने वाले सम्पूर्ण जीव-जन्तु एवं पेड़-पौधे एक साथ मिलकर कार्य करते हैं तथा एक-दूसरे पर प्रभाव डालते हैं। इस प्रकार किसी क्षेत्र में कार्य करने वाले जैविक एवं अजैविक अंशों का सम्पूर्ण योग ही ‘पारिस्थितिक-तन्त्र’ कहलाता है। इस आधार पर कहा जा सकता है कि “पारिस्थितिक-तन्त्र प्रकृति की एक क्रियात्मक इकाई है।” इस प्रकार किसी क्षेत्र या प्रदेश विशेष में कार्यरत जैविक एवं अजैविक अंशों का पूर्ण योग ही पारिस्थितिक-तन्त्र कहलाता है। पारिस्थितिक-तन्त्र को वैज्ञानिकों ने निम्नलिखित प्रकार परिभाषित किया है

“पारिस्थितिक-तन्त्र पर्यावरण तथा समुदाय की क्रियात्मक समक्रिया है।” -क्लार्क

“पारिस्थितिक-तन्त्र मूल क्रियात्मक इकाई है, जिसमें जैविक तथा अजैविक पर्यावरण सम्मिलित हैं जो परस्पर प्रभावित करते हैं जिससे ऊर्जा प्रवाह-तन्त्र में निश्चित एवं स्पष्ट जैविक विविधता का चक्र बनता है।” -ओडम

इस प्रकार पारिस्थितिक-तन्त्र जैविक एवं पर्यावरण के सभी भागों में तथा उसके बीच पारस्परिक क्रिया का योग है। पारिस्थितिक-तन्त्र में जीवधारियों का समुदाय अनेक प्रकार के जीवों (पेड़-पौधे एवं जीव-जन्तु) से मिलकर बनता है। पारिस्थितिक-तन्त्र स्थायी अथवा अस्थायी दोनों प्रकार का हो सकता है।

पारिस्थितिक-तत्र का वर्गीकरण

पारिस्थितिक-तन्त्र को अनेक छोटी इकाइयों में वर्गीकृत किया जा सकता है जिससे उनको आकारिकी, कार्यिकी एवं गति सम्बन्धी ज्ञान प्राप्त हो सके। इस वर्गीकरण का आधार जलवायु, निवास स्थान एवं पौधों का समुदाय होता है। पारिस्थितिक-तन्त्र निम्नलिखित दो प्रकार का होता है
1. प्राकृतिक पारिस्थितिक-तन्त्र—यह तन्त्र प्रकृति द्वारा सम्पन्न किया जाता है। प्राकृतिक आवास निम्नलिखित दो प्रकार का होता है
(i) स्थलीय (Terrestrial)-वन, घास के मैदान, मरुस्थलीय आदि।
(ii) जलीय (Aquatic)-जलीय पारिस्थितिक-तन्त्र दो प्रकार का होता है

(अ) स्वच्छ जलीय (Fresh water)-इसके अन्तर्गत, नदी, झील एवं तालाब,आदि सम्मिलित किये जाते हैं।
(ब) समुद्र जलीय (Sea water)-इसके अन्तर्गत खारे जल के क्षेत्र अर्थात् महासागर, सागर. एवं खारे पानी की झीलें सम्मिलित की जाती हैं।

2. कृत्रिम अथवा मानव-निर्मित पारिस्थितिक-तन्त्र-यह तन्त्र मानव एवं उसकी क्रियाओं द्वारा सम्पन्न होता है। मानव अपने क्रिया-कलापों एवं तकनीकी-प्राविधिक ज्ञान द्वारा प्राकृतिक सन्तुलन में गड़बड़ कर देता है; अर्थात् असन्तुलन की स्थिति उत्पन्न हो जाती है। मानव बड़े-बड़े क्षेत्रों को काटकर कृषि-योग्य भूमि, औद्योगिक क्षेत्रों तथा बड़ी-बड़ी बस्तियों का निर्माण करता है। यह मानव द्वारा निर्मित भूदृश्य कहलाता है। मानव भौतिक पर्यावरण को नियन्त्रित करने का प्रयास करता है, परन्तु नियन्त्रण के स्थान पर इसमें असन्तुलन की स्थिति उत्पन्न हो जाती है। इसे कृत्रिम | अथवा अप्राकृतिक पारिस्थितिक-तन्त्र कहा जाता है।

प्रश्न 3. पारिस्थितिक-तन्त्र में असन्तुलन की समस्याओं का वर्णन कीजिए।
या पारिस्थितिक-तन्त्र के असन्तुलन की समस्या की विवेचना कीजिए तथा उसके निराकरण के उपायों को प्रस्तावित कीजिए। |
या टिप्पणी लिखिए-पारिस्थितिक असन्तुलन।
उत्तर-पारिस्थितिकीय असन्तुलन की समस्या
मानव द्वारा पारिस्थितिक-तन्त्र का शोषण किया जाता है। इनमें प्राकृतिक वनस्पति, वन्य जीव-जन्तु, मत्स्य आदि प्रमुख हैं। अधिकतम खाद्यान्नों की प्राप्ति के लिए पारिस्थितिक-तन्त्र में अनेक परिवर्तन हुए। हैं जिससे सामान्य पारिस्थितिक-तन्त्र का विकास हुआ है। पारिस्थितिकीय असन्तुलन को पर्यावरण-प्रदूषण भी कहा जा सकता है। मानव प्राकृतिक संसाधनों का शोषण करता है; जैसे-खानों से खनिजों का शोषण, भूगर्भ से खनिज तेल, वनों से लकड़ी की कटाई कर तथा अपने मनोरंजन के लिए। वन्य जीवों का आखेट कर पारिस्थितिकीय सन्तुलन को बिगाड़ता रहता है। पालतू पशुओं से दूध, मांस, ऊन तथा अन्य पदार्थ प्राप्त होते हैं जिससे उसकी भोजन-श्रृंखला छोटी हो गयी है। इससे इन पालतू पशुओं की संख्या में भी कमी होने लगी है तथा पारिस्थितिकीय असन्तुलन की समस्या ने जन्म ले लिया है।

“पारिस्थितिक-तन्त्र के किसी भी घटक को वांछित एवं आवश्यक मात्रा से कम हो जाना अथवा अधिक हो जाना ही ‘पारिस्थितिकीय असन्तुलन’ कहलाता है तथा प्रत्येक घटक का उस अनुपात में रहना जिससे इस तन्त्र के अन्य घटकों पर कोई हानिकारक प्रभाव न पड़े ‘पारिस्थितिक सन्तुलन कहलाता है।” पारिस्थितिक-तन्त्र में असन्तुलन की स्थिति तभी उत्पन्न होती है जब किसी सम्पूर्ण पोषण-स्तर का विनाश हो जाता है। यह स्थिति जीवों की कमी के कारण अथवा वैकल्पिक साधनों की कमी के कारण अथवा प्रदूषण के कारण हो सकती है।

1. प्रदूषण की समस्या-कृषि उत्पादन की सफलता फसलों द्वारा अपने पारिस्थितिक-तन्त्र के अनुकूलन पर निर्भर करती है। स्थानीय जलवायु दशाएँ फसलों का निर्धारण करती हैं। रासायनिक उर्वरकों द्वारा पोषक तत्वों में वृद्धि कर उत्पादन में भी वृद्धि के प्रयास किये गये हैं तथा अधिक उत्पादन देने वाली फसलें खोज ली गयी हैं। ‘हरित क्रान्ति’ ने खाद्यान्न उत्पादन में तो वृद्धि की है,.. परन्तु उर्वरकों के उत्पादन से प्रदूषण की समस्या उत्पन्न हो गयी है। मानव ने फसलों के क्षेत्रफल में वृद्धि की है जिसके कारण प्रेयरी, टैगा एवं स्टेप्स घास के मैदानों में प्राकृतिक वनस्पति में कमी हो गयी है। पादपों को कीटनाशकों से बचाव के लिए कीटनाशक दवाओं का अधिकाधिक उपयोग : किया जाने लगा है, परन्तु इनका अधिक उपयोग मानव के लिए हानिकारक है। इनसे मानव को । दूषित खाद्य सामग्री प्राप्त होती है। इन कीटनाशकों के उत्पादन काल में प्रदूषण की अनेक समस्याएँ जन्म लेती हैं। भोपाल गैस त्रासदी इसका एक मुख्य उदाहरण है जिससे मानवता में। अपंगता ने जन्म लिया है।

2. जैव-प्रदूषण की समस्या-आदि काल से लेकर आज तक वनों का बड़ी निर्ममता से शोषण किया जाता रहा है। इससे वन-क्षेत्रों का ह्रास हुआ है। प्रारम्भ से ही विश्व के अनेक भागों में स्थानान्तरित अथवा शूमिंग कृषि पद्धति प्रचलित है। इस पद्धति के अन्तर्गत उर्वर भूमि को प्राप्त करने के लिए मानव वन-क्षेत्रों का शोषण कर उस पर कृषि करता है। जब इस भूमि की उर्वरता समाप्त हो जाती है तो इसे परती छोड़ दिया जाता है। इस प्रकार वनों के निर्दयतापूर्वक शोषण से पर्यावरण प्रदूषण की समस्या उत्पन्न हुई है तथा पारिस्थितिक-तन्त्र भी असन्तुलित हुआ है। वनों की कमी के कारण भू-अपरदने, अनावृष्टि, बाढ़ आदि समस्याएँ उत्पन्न हो गयी हैं। अतः आज मानव के समक्ष अनेक प्रदूषण सम्बन्धी समस्याएँ विकराल रूप धारण कर गयी हैं। इसी कारण विश्व में वन-संरक्षण के प्रयास किये जा रहे हैं।

जलीय जीवों के प्राणों की रक्षा करना भी अति आवश्यक है। मत्स्य व्यवसाय पर ध्यान दिया जाना अति आवश्यक है। मछली से मानव को प्रोटीन की प्राप्ति होती है। यदि मत्स्य व्यवसाय का विकास सुचारु रूप से नहीं हुआ तो खाद्यान्न में प्रोटीन की कमी हो जाएगी। विश्व में 3% मानव का भोजन पूर्ण रूप से मछली पर निर्भर करता है, जबकि नॉर्वे, न्यूफाउण्डलैण्ड एवं जापान सदृश देशों में 10% मानव मछली के ऊपर ही निर्भर करते हैं। इनकी कमी से खाद्य समस्या उत्पन्न हो सकती है। अत: इस ओर ध्यान दिये जाने की नितान्त आवश्यकता है।

3. जनाधिक्य की समस्या संयुक्त राष्ट्र संघ के अनुसार-सन् 2010 में विश्व की जनसंख्या 6.9 अरब हो गयी है। सन् 2050 तक इसके 9.10 अरब हो जाने का अनुमान है। जनसंख्या की इस अतिशय वृद्धि के कारण वन क्षेत्रों एवं चरागाहों की कमी होती जा रही है। भोजन की समस्या के समाधान के लिए कृषि-क्षेत्रों का विस्तार किया जा रहा है जिस कारण वन एवं घास क्षेत्रों का विनाश किया जा रहा है जिससे पारिस्थितिक-तन्त्र में अन्तर उपस्थित हुआ है। इसके साथ ही आवास समस्या उत्पन्न हो गयी है। बस्तियों के विकास के लिए उत्पादक भूमि का अधिग्रहण होता
जा रहा है।

इस प्रकार जनसंख्या में उत्तरोत्तर वृद्धि के कारण अनेक समस्याएँ विकराल रूप धारण कर चुकी हैं। भोजन, वस्त्र एवं आवास जैसी प्राथमिक आवश्यकताओं का अभाव होता जा रहा है। इससे प्रदूषण की अनेक समस्याओं ने जन्म लिया है। वायु, जल व मृदा प्रदूषण वर्तमान युग की सबसे बड़ी समस्याएँ हैं। यह पारिस्थितिकीय असन्तुलन की स्थिति है। इससे आज मानव समुदाय को अनेक विषमताओं का सामना करना पड़ता है।

पारिस्थितिकीय असन्तुलन की समस्या का निवारण

पारिस्थितिक-तन्त्र में असन्तुलन की समस्या के निवारण के लिए निम्नलिखित उपाय किये जाने चाहिए
1. जनाधिक्य पर नियन्त्रण-आधुनिक युग में विश्व की जनसंख्या में तीव्र गति से वृद्धि होती जा | रही है जिस पर नियन्त्रण किया जाना अति आवश्यक है। यदि जनसंख्या-वृद्धि उपलब्ध संसाधनों के अनुसार हो तो अधिक उपयुक्त रहेगा।

2. वन क्षेत्रफल में वृद्धि तथा उनका संरक्षण-प्रदूषण से बचाव के लिए वन क्षेत्रफल में वृद्धि किया जाना अति आवश्यक है। भारत में सामाजिक वानिकी तथा वन महोत्सव आदि कार्यक्रमों द्वारा वन क्षेत्रफल में वृद्धि के प्रयास किये जा रहे हैं। उत्तराखण्ड में श्री सुन्दर लाल बहुगुणा का ‘चिपको आन्दोलन’ भी इस दिशा में एक महत्त्वपूर्ण प्रयास है।

वन एक महत्त्वपूर्ण प्राकृतिक स्रोत है। यह वन्य जीवन के लिए अति आवश्यक है। पर्यावरणीय सन्तुलन को बनाये रखने के लिए वनों का महत्त्वपूर्ण योगदान है। पौधे पर्यावरण से कार्बन
डाइ-ऑक्साइड लेकर ऑक्सीजन निःसृत करते हैं जिसका उपयोग जन्तुओं द्वारा श्वसन क्रिया में किया जाता है।

3. जल संसाधनों में वृद्धि एवं उनका संरक्षण-जल संसाधनों में वृद्धि किया जाना अति आवश्यक है। जब जल में अनेक प्रकार के खनिज पदार्थ, कार्बनिक एवं अकार्बनिक पदार्थों तथा गैसों के एक निश्चित अनुपात में अधिक अथवा कम अनावश्यक एवं हानिकारक पदार्थ घुले होते हैं, तब जल का प्रदूषण हो जाता है। प्रदूषित जल का उपयोग करने से प्राणी समुदाय में अनेक रोग उत्पन्न हो जाते हैं; अत: ऐसे प्रयास किये जाने चाहिए कि जल का प्रदूषण न होने पाये।

जल संसाधनों में वृद्धि के लिए मत्स्य उत्पादक क्षेत्रों में जल प्रदूषणरहित होना चाहिए। जल-क्षेत्रों में जिन मछलियों की प्रजाति की कमी है, उनके पकड़ने पर रोक लगा देनी चाहिए; जैसे—सील, ह्वेल आदि। ऐसे प्रयास किये जाने चाहिए कि मछलियों की भोज्य-सामग्री वनस्पति एवं प्लैंकटन पर्याप्त मात्रा में पनपती रहे। इसके लिए इन क्षेत्रों में शुद्ध जल की प्राप्ति होना अति आवश्यक है। यह जल प्रदूषणरहित होना चाहिए। ऐसे क्षेत्र मछलियों के अक्षय भण्डार हो सकते हैं। मछली पकड़ने की विकसित तकनीक होनी चाहिए तथा उन्हें नियमित रूप से ही पकड़ा जाना चाहिए।

4. वन्य-प्राणियों का संरक्षण-वन्य जीवों में लगभग 350 जातियाँ स्तनधारियों की तथा 2,100 पक्षियों की होने के साथ-साथ लगभग 20,000 प्रजातियों कीटों की वन्य अवस्था में पायी जाती हैं। अतः पारिस्थितिक-तन्त्र में सन्तुलन बनाये रखने के लिए वन्य प्राणियों का संरक्षण अति आवश्यक है। वन्य जीवों के संरक्षण के लिए निम्नलिखित उपायों को कार्यरूप में परिणत किया जाना चाहिए
(i) वन्य प्राणियों के संरक्षण के लिए अभयारण्यों की स्थापना की जानी चाहिए।
(ii) वन्य जीवों के आवासीय स्थलों को संरक्षण दिया जाना चाहिए।
(iii) वन्य जीवों की विभिन्न जातियों को संरक्षण दिया जाना चाहिए।
(iv) नवीन वन्य जातियों को वन-क्षेत्रों में पालन-पोषण के प्रयास किये जाने चाहिए।
(v) सभी देशों में वन्य जीवों के आखेट पर प्रतिबन्ध लगा दिये जाएँ तथा इनके अनुपालन के | लिए कठोर नियम एवं कानून होने चाहिए। इनका कठोरता से अनुपालन किया जाना
चाहिए।

इस प्रकार उपर्युक्त तथ्यों से निष्कर्ष निकलता है कि प्राणी-समुदाय के कल्याण एवं उसके पर्यावरण को अक्षुण्ण बनाये रखने के लिए पारिस्थितिक-तन्त्र को सन्तुलित बनाये रखना अति आवश्यक है।

प्रश्न 4. पारितन्त्र की कार्यप्रणाली एवं संरचना पर प्रकाश डालिए।
उत्तर-पारितन्त्र की कार्यप्रणाली एवं संरचना
पारिस्थितिकी की क्रियाशील इकाई पारिस्थितिक-तन्त्र होती है जिसमें जैव तथा अजैव पर्यावरण एक-दूसरे के गुणों से प्रभावित होते हुए महत्त्वपूर्ण भूमिका का निर्वाह करते हैं। यह दोनों ही धरातल पर जीवन बनाए रखने के लिए आवश्यक हैं। कोई भी जैव, अजैव पर्यावरण के बिना जीवित नहीं है। उसके जीवन का निश्चित तन्त्र पारिस्थितिक-तन्त्र द्वारा ही निर्धारित होता है।

संरचना की दृष्टि से पारितन्त्र में जैविक एवं अजैविक कारकों की सक्रिय भूमिका होती है। जैविक कारकों में उत्पादक-प्राथमिक, द्वितीयक व तृतीयक, उपभोक्ता तथा अपघटक सम्मिलित हैं। अजैविक कारकों में तापमान, वर्षा, सूर्य का प्रकाश, आर्द्रता, मृदा की स्थिति एवं अकार्बनिक तत्त्व (कार्बन डाइ-ऑक्साइड, जल, नाइट्रोजन, कैल्सियम, फॉस्फोरस, पोटैशियम आदि) सम्मिलित हैं।
UP Board Solutions for Class 11 Geography Fundamentals of Physical Geography Chapter 15 Life on the Earth (पृथ्वी पर जीवन) img 6
ओडम के अनुसार, “पारिस्थितिक-तन्त्र, पारिस्थितिकी की वह आधारभूत इकाई है जिसमें जैविक और अजैविक वातावरण एक-दूसरे पर अपना प्रभाव डालते हुए पारस्परिक अनुक्रिया एवं ऊर्जा व रासायनिक पदार्थों के निरन्तर प्रवाह से तन्त्र की कार्यात्मक गतिशीलता बनाए रखते हैं।” (चित्र 15.3)। समुदाय के जैविक सदस्यों तथा उनके अजैविक वातावरण में ऊर्जा प्रवाह और खनिज पदार्थों के चक्र को पूरा करने के लिए निरन्तर रचनात्मक और कार्यात्मक पारस्परिक अनुक्रियाएँ होती रहती हैं, इन्हीं अनुक्रियाओं एवं कार्य-प्रणाली से पारितन्त्र संचालित होकर जीवों के विकास में महत्त्वपूर्ण भूमिका निभाता है।

We hope the UP Board Solutions for Class 11 Geography: Fundamentals of Physical Geography Chapter 15 Life on the Earth (पृथ्वी पर जीवन) help you. If you have any query regarding UP Board Solutions for Class 11 Geography: Fundamentals of Physical Geography Chapter 15 Life on the Earth (पृथ्वी पर जीवन), drop a comment below and we will get back to you at the earliest.

 

UP Board Solutions for Class 11 Geography: Fundamentals of Physical Geography Chapter 14 Movements of Ocean Water

UP Board Solutions for Class 11 Geography: Fundamentals of Physical Geography Chapter 14 Movements of Ocean Water (महासागरीय जल संचलन)

These Solutions are part of UP Board Solutions for Class 11 Geography. Here we have given UP Board Solutions for Class 11 Geography: Fundamentals of Physical Geography Chapter 14 Movements of Ocean Water (महासागरीय जल संचलन)

पाठ्य-पुस्तक के प्रश्नोत्तर

1. बहुवैकल्पिक प्रश्न
प्रश्न (i) महासागरीय जल की ऊपर व नीचे की गति किससे सम्बन्धित है?
(क) ज्वार ।
(ख) तरंग
(ग) धाराएँ।
(घ) इनमें से कोई नहीं
उत्तर-(क) ज्वार।

प्रश्न (ii) वृहत ज्वार आने को क्या कारण है?
(क) सूर्य और चन्द्रमा का पृथ्वी पर एक ही दिशा में गुरुत्वाकर्षण बल
(ख) सूर्य और चन्द्रमा द्वारा एक-दूसरे की विपरीत दिशा से पृथ्वी पर गुरुत्वाकर्षण बल
(ग) तट रेखा का दन्तुरित होना |
(घ) उपर्युक्त में से कोई नहीं ।
उत्तर-(क) सूर्य और चन्द्रमा का पृथ्वी पर एक ही दिशा में गुरुत्वाकर्षण बल।।

प्रश्न (iii) पृथ्वी तथा चन्द्रमा की न्यूनतम दूरी कब होती है?
(क) अपसौर
(ख) उपसौर
(ग) उपभू ।
(घ) अपभू
उत्तर-(ग) उपभू।

प्रश्न (iv) पृथ्वी उपसौर की स्थिति कब होती है?
(क) अक्टूबर
(ख) जुलाई ।
(ग) सितम्बर
(घ) जनवरी
उत्तर-(घ) जनवरी।।

2. निम्नलिखित प्रश्नों के उत्तर लगभग 30 शब्दों में दीजिए
प्रश्न (i) तरंगें क्या हैं?
उत्तर-तरंगें वास्तव में ऊर्जा हैं जल नहीं, जो महासागरीय सतह के आर-पार गति करती हैं। समुद्र का जल पवनों के चलने से ऊपर उठता एवं गिरता हुआ प्रतीत होता है। हवाओं के प्रभाव से यह जल लहरदार आकृतियों में दिखाई देता है। इसलिए इनको तरंगें या लहरें (waves) कहते हैं।

प्रश्न (ii) महासागरीय तरंगें ऊर्जा कहाँ से प्राप्त करती हैं?
उत्तर-तरंगें ऊर्जा वायु से प्राप्त करती हैं। वायु के कारण ही तरंगें महासागर में गति करती हैं तथा ऊर्जा को तट रेखा पर निर्मुक्त करती हैं।

प्रश्न (iii) ज्वार-भाटा क्या है?
उत्तर-चन्द्रमा एवं सूर्य के गुरुत्वाकर्षण के प्रभाव से समुद्रतल में निश्चित समय पर आने वाले परिवर्तन को ‘ज्वार-भाटा’ कहा जाता है। ज्वार’ सागर के जल के ऊपर उठने की प्रक्रिया है अर्थात् जब सागरीय जल सूर्य एवं चन्द्रमा की आकर्षण शक्ति द्वारा एकत्रित होकर तीव्रता से ऊपर उठता है तो उसे ज्वार’ कहते हैं। जिन स्थानों से जल खिंचकर आता है वहाँ जल का तल नीचा हो जाता है, जिसे ‘भटा’ कहा जाता है।

प्रश्न (iv) ज्वार-भाटा उत्पन्न होने के क्या कारण हैं? .
उत्तर-ज्वार-भाटा, चन्द्रमा एवं सूर्य की गुरुत्वाकर्षण शक्ति का परिणाम है। इसके अतिरिक्त ज्वार-भाटा की उत्पत्ति के लिए अपकेन्द्रीय बल भी उत्तरदायी है जो गुरुत्वाकर्षण शक्ति को संतुलित करता है। अतः गुरुत्वाकर्षण बल और अपकेन्द्रीय बल दोनों मिलकर पृथ्वी पर दो महत्त्वपूर्ण ज्वार-भाटाओं को उत्पन्न करते हैं। चन्द्रमा की ओर वाले पृथ्वी के भाग पर एक ज्वार-भाटा उत्पन्न होता है, जब विपरीत भौग पर चन्द्रमा का गुरुत्वीय आकर्षण बल उसकी दूरी के कारण कम होता है, तब अपकेन्द्रीय बल दूसरी तरफ ज्वार उत्पन्न करता है (चित्र 14.1)।
UP Board Solutions for Class 11 Geography Fundamentals of Physical Geography Chapter 14 Movements of Ocean Water (महासागरीय जल संचलन) img 1

प्रश्न (v) ज्वार-भाटा नौसंचालन से कैसे सम्बन्धित है? ।
उत्तर-ज्वार-भाटा नौसंचालन एवं मछुआरों को उनके कार्य में सहयोग प्रदान करता है। नौसंचालन में ज्वारीय प्रवाह अत्यधिक सहयोगी होता है। विशेषकर ज्वारनदमुख के भीतर जहाँ प्रवेशद्वार पर छिछले रोधिका होते हैं वहाँ पर ज्वार-भाटा से जल की आपूर्ति हो जाने पर नौका संचालन अत्यन्त सरल हो जाता है। अत: उथले समुद्रों में दीर्घ ज्वार से जहाज बन्दरगाह तक आ जाते हैं और भाटे के समय चले जाते हैं।

3. निम्नलिखित प्रश्नों के उत्तर लगभग 150 शब्दों में दीजिए
प्रश्न (1) जलधाराएँ तापमान को कैसे प्रभावित करती हैं? उत्तर-पश्चिमी यूरोप के तटीय क्षेत्रों के तापमान को ये किस प्रकार प्रभावित करती हैं?
उत्तर-जलधाराएँ किसी प्रदेश की जलवायु एवं विशेषकर तापमान को बहुत प्रभावित करती हैं। जिस प्रकार की जलवायु होगी वैसा ही प्रभाव उस क्षेत्र पर भी पड़ता है। गर्म जलधाराएँ जिस क्षेत्र में प्रभावित होती हैं उस क्षेत्र के तापमान में वृद्धि हो जाती है और ठण्डी जलधाराएँ उस क्षेत्र के तापमान को कम कर देती हैं। यह प्रभाव पश्चिमी यूरोप तथा उत्तरी-पूर्वी एशिया में विशेष रूप से देखा जा सकता है जहाँ क्रमशः गल्फस्ट्रीम एवं क्यूरोसिवो जलधाराएँ तापमान में परिवर्तन उत्पन्न करने के लिए उत्तरदायी मानी जाती हैं। उत्तरी अन्ध महासागर में यूरोप का पश्चिमी तट उत्तर अटलाण्टिक ड्रिफ्ट के गर्म जल के प्रभाव से जाड़ों में यूरेशिया के भीतरी भागों व कनाडा के पूर्वी तट की अपेक्षा लगभग 10° से 15° सेल्सियस तर्क अधिक गर्म रहता है। इसी कारण नॉर्वे का तट व्यापार के लिए जाड़ों में भी खुला रहता है, जबकि उन्हीं अक्षांशों में स्थित साइबेरिया का तट हिम से जम जाता है।

प्रश्न (ii) जलधाराएँ कैसे उत्पन्न होती हैं?
या जलधाराओं की उत्पत्ति के क्या कारण हैं? वर्णन कीजिए।
उत्तर-महासागरीय जलधाराओं की उत्पत्ति के निम्नलिखित कारण हैं
1. तापमान की भिन्नता-सागरीय जल के तापमान में क्षैतिज एवं लम्बवत् भिन्नताएँ पाई जाती हैं। जल निम्न तापमान के कारण ठण्डा होकर नीचे बैठ जाता है, जिस कारण विषुवत रेखीय क्षेत्रों से जल ध्रुवों की ओर प्रवाहित होने लगता है। उत्तरी एवं दक्षिणी विषुवत्रेखीय जलधाराएँ इसी प्रकार की हैं।

2. लवणता की भिन्नता-महासागरीय जल की लवणता में पर्याप्त भिन्नता पाई जाती है। लवणता की भिन्नता से सागरीय जल का घनत्व भी परिवर्तित हो जाता है। उत्तरी एवं दक्षिणी ध्रुवों से जल कम घनत्व या कम लवणता वाले भागों से विषुवत् रेखा की ओर प्रवाहित होने लगता है। इस प्रकार सागरीय जल में लवणता के घनत्व में भिन्नता के कारण जलधाराओं की उत्पत्ति हो जाती है। हिन्द महासागर के जल को लाल सागर की ओर प्रवाह इसका उत्तम उदाहरण है।

3. प्रचलित पवनों का प्रभाव-प्रचलित पवनें वर्षेभर नियमित रूप से प्रवाहित होती हैं और ये अपने मार्ग में पड़ने वाली जलराशि को पवन की दिशा के अनुकूल धकेलती हुई चलती हैं जिससे जलराशि प्रवाहित होने लगती है। उदाहरण के लिए-पछुआ पवनों के प्रभाव से क्यूरोसिवो व गल्फस्ट्रीम की धाराओं को गति व दिशा मिलती है (चित्र 14.2)।

4. वाष्पीकरण व वर्षा-पृथ्वी तल पर वाष्पीकरण व वर्षा में पर्याप्त भिन्नता पाई जाती है। जहाँ वाष्पीकरण अधिक होता है वहाँ सागर तल नीचा हो जाता है; अत: उच्च-तल के क्षेत्रों से सागरीय जल निम्न जल-तल की ओर प्रवाहित होने लगता है जिससे जलधाराओं की उत्पत्ति हो जाती है। ठीक इसी प्रकार अधिक वर्षा वाले क्षेत्रों में सागरीय जल-तल में वृद्धि हो जाती है। ऐसे क्षेत्रों से जल निम्न वर्षा तथा निम्न जल-तल वाले भागों की ओर एक धारा के रूप में प्रवाहित होने लगता है।

5. पृथ्वी की दैनिक गति-पृथ्वी अपने अक्ष पर तीव्र गति से घूमती हुई सूर्य के सम्मुख लगभग 24 घण्टे में एक चक्कर पूरा कर लेती है। पृथ्वी की घूर्णन गति के कारण सागरीय जल उत्तरी गोलार्द्ध में दाईं ओर तथा दक्षिणी गोलार्द्ध में बाईं ओर घूम जाता है। पृथ्वी की घूर्णन गति का प्रभाव जलधाराओं के प्रवाह एवं उनकी गति पर भी पड़ता है।

इसके अतिरिक्त वायुभारे की भिन्नता, ऋतु परिवर्तन, समुद्रतटीय आकृति आदि ऐसे कारक हैं जो महासागरों में जलधाराओं की उत्पत्ति में सहायक होते हैं।

परीक्षोपयोगी प्रश्नोत्तर

बहुविकल्पीय प्रश्न
प्रश्न 1. निम्नलिखित में से कौन-सी समुद्री धारा गर्म धारा है?
(क) कनारी
(ख) पेरू
(ग) क्यूराइल
(घ) क्यूरोसिवो
उत्तर-(घ) क्यूरोसिवो।

प्रश्न 2. निम्नलिखित में से कौन-सी समुद्री धारा गर्म धारा नहीं है?
(क) ब्राजील
(ख) गल्फस्ट्रीम
(ग) कनारी
(घ) क्यूरोसिवो
उत्तर-(ग) कनारी।

प्रश्न 3. निम्नलिखित में से कौन-सी महासागरीय ठण्डी धारा है?
(क) कैलीफोर्नियन
(ख) क्यूरोसिवो ।
(ग) क्यूराइल
(घ) गल्फस्ट्रीम
उत्तर-(ग) क्यूराइल।

प्रश्न 4. निम्नलिखित में से कौन एक उत्तरी अंध महासागर की धारा है?
(क) बेंगुएला धारा
(ख) फाकलैण्ड धारा
(ग) लैब्रेडोर धारा ।
(घ) अगुलहास धारा
उत्तर-(ग) लैब्रेडोर धारा।

प्रश्न 5. निम्नलिखित में से कौन गर्म समुद्री धारा है?
(क) लैब्रेडोर धारा
(ख) ब्राजील धारा
(ग) कैलीफोर्निया धारा ।
(घ) पश्चिम ऑस्ट्रेलिया धारा
उत्तर-(घ) पश्चिम ऑस्ट्रेलिया धारा।।

प्रश्न 6. निम्नलिखित में से कौन-सी समुद्री धारा ठण्डी धारा है?
(क) ब्राजील धारा
(ख) हम्बोल्ट (पेरू) धारा
(ग) क्यूरोसिवो धारा
(घ) गल्फस्ट्रीम
उत्तर-(ख) हम्बोल्ट (पेरू) धारा।

प्रश्न 7. निम्नलिखित में से कौन-सी एक गर्म महासागरीय धारा है।
(क) कैलीफोर्निया
(ख) कनारी
(ग) ब्राजील ।
(घ) बंगुएला
उत्तर-(ग) ब्राजील।

प्रश्न 8. निम्नलिखित में से कौन एक ठण्डी धारा है?
(क) ब्राजील ।
(ख) क्यूराइल
(ग) गल्फस्ट्रीम
(घ) क्यूरोसिवो
उत्तर-(ख)क्यूराइल।

अतिलघु उत्तरीय प्रश्न

प्रश्न 1. महासागरीय मग्नतट से आप क्या समझते हैं?
उत्तर-यह समुद्र के नितल का अति मन्द ढालयुक्त भाग है, जो महाद्वीप के चारों ओर फैला हुआ है।

प्रश्न 2. महासागरीय जल की लवणता को समझाइए।
उत्तर-सागरीय जल में लवणों की उपस्थिति से उत्पन्न खारेपन को महासागरीय जल की लवणता कहा जाता है।

प्रश्न 3. महासागरीय जलधाराएँ क्या हैं?
उत्तर-जब सागरों एवं महासागरों का जल नियमित रूप से एक निश्चित दिशा में प्रवाहित होने लगता है तो उसे महासागरीय जलधारा कहते हैं। जलधाराएँ सागरों में उसी प्रकार प्रवाहित होती हैं, जैसे स्थलीय भागों में नदियाँ एवं नाले।

प्रश्न 4. महासागरीय जलधारा के दो प्रभाव लिखिए।
उत्तर-महासागरीय जलधारा के दो प्रभाव निम्नलिखित हैं

  • महासागरीय जलधाराओं का निकटवर्ती क्षेत्र की जलवायु पर व्यापक प्रभाव पड़ना।
  • ठण्डी एवं गर्म जलधाराओं के सम्मिश्रण से मत्स्य क्षेत्रों का विकास।

प्रश्न 5. उत्तरी अन्ध महासागर की धाराओं के नाम लिखिए।
उत्तर-उत्तरी अन्ध महासागर की धाराओं में उत्तरी विषुवतीय धारा, गल्फस्ट्रीम धारा, उत्तरी अटलाण्टिक ड्रिफ्ट, लैब्रेडोर धारा और कनारी धारा प्रमुख हैं।

प्रश्न 6. सारंगैसो सागर कहाँ है? इसका यह नाम क्यों पड़ा है?
उत्तर-सारगैसो सागर उत्तरी अन्ध महासागर में स्थित है। इस क्षेत्र की धाराएँ एक चक्र के रूप में घूमती हैं। इस स्थिर व शान्त जल क्षेत्र में अनेक प्रकार की घास-फूस भी एकत्र हो जाती है जिसमें सारगैसो घास की अधिकता होने के कारण इसे सारगैसो सागर कहा जाता है।

प्रश्न 7. हिन्द महासागर की धाराओं के नाम लिखिए।
उत्तर-1. दक्षिणी भूमध्यरेखीय जलधारा, 2. मौजाम्बिक जलधारा, 3. अगुलहास जलधारा, 4. पश्चिमी ऑस्ट्रेलिया की धारा, 5. मेष्मकालीन मानसून प्रवाह एवं उत्तर-पूर्वी मानसून प्रवाह आदि हिन्द महासागर की धाराएँ हैं।

प्रश्न 8. प्रशान्त महासागर तथा अन्ध महासागर की एक-एक ठण्डी एवं गर्म जलधारा का आर्थिक महत्त्व बतलाइए।
उत्तर-न्यूफाउण्डलैण्ड के समीप अन्ध महासागर की गर्म गल्फस्ट्रीम तथा लैब्रेडोर की ठण्डी धारा एवं जापाने तट पर प्रशान्त महासागर की क्यूरोसिवो गर्म और क्यूराइल ठण्डी धाराओं के मिलने से घना कुहरा उत्पन्न होता है। ये क्षेत्र संसार के सबसे महत्त्वपूर्ण मत्स्य आखेट क्षेत्र के रूप में विकसित हैं।

प्रश्न 9. पृथ्वी के सागर तल पर प्रतिदिन दो बार ज्वार-भाटा क्यों आता है।
उत्तर-पृथ्वी के सागर तल पर प्रतिदिन दो बार ज्वार-भाटा आने का प्रमुख कारण मुरुत्वाकर्षण शक्ति एवं पृथ्वी के अपकेन्द्रीय बल का प्रभाव है।

प्रश्न 10. दीर्घ ज्वार क्या है? यह पूर्णिमा और अमावस्या को ही क्यों आता है?
या वृहत् ज्वार की व्याख्या कीजिए।
उत्तर-जब सागरीय ज्वार की ऊँचाई औसत ज्वार से 20% अधिक होती है जो उसे दीर्घ ज्वार कहते हैं। यह पूर्णिमा और अमावस्या को इसलिए आता है क्योंकि इन तिथियों में सूर्य व चन्द्रमा की संयुक्त शक्ति पृथ्वी पर अपना सर्वाधिक प्रभाव डालती है।

प्रश्न 11. अर्द्ध-दैनिक ज्वार-भाटा किसे कहते हैं?
उत्तर-जब कहीं दिन में दो बार ज्वार-भाटा आता है तो उसे अर्द्ध-दैनिक ज्वार-भाटा कहते हैं। यह प्रति 12 घण्टे 26 मिनट पश्चात् आता है।

प्रश्न 12. तरंग की गति कैसे निर्धारित होती है?
उत्तर-तरंग की गति उसके दैर्घ्य तथा आवर्तकाल से सम्बन्धित है और निम्नलिखित सूत्र से ज्ञात की जा सकती है
UP Board Solutions for Class 11 Geography Fundamentals of Physical Geography Chapter 14 Movements of Ocean Water (महासागरीय जल संचलन) img 3

प्रश्न 13. ऊँचाई एवं आवृत्ति के आधार पर ज्वार-भाटा का वर्गीकरण कीजिए।
उत्तर-ऊँचाई के आधार पर-1. वृहत् ज्वार, 2. निम्न ज्वार।।
आवृत्ति के आधार पर—

  1. अर्द्ध-दैनिक ज्वार,
  2. दैनिक ज्वार,
  3. मिश्रित ज्वार।

प्रश्न 14. उन तीन आकर्षण शक्तियों को बताइए जो ज्वार के निर्माण में योगदान करती हैं?
उत्तर-ज्वार के निर्माण में योगदान देने वाली तीन आकर्षण शक्तियाँ निम्नलिखित हैं1. चन्द्रमा की आकर्षण शक्ति, 2. सूर्य की आकर्षण शक्ति, 3. पृथ्वी की आकर्षण शक्ति।

प्रश्न 15. महासागरीय जल की तीन गतियाँ कौन-सी हैं?
उत्तर-महासागरीय जल की तीन गतियाँ निम्नलिखित हैं

  • तरंग,
  • धाराएँ,
  • ज्वार-भाटा।

लघु उत्तरीय प्रश्न

प्रश्न 1. महासागरीय धाराओं के तटवर्ती भागों पर क्या प्रभाव होते हैं?
उत्तर-महासागरीय धाराएँ अपने स्वभाव के अनुसार तटवर्ती भागों की जलवायु मत्स्याखेट तथा व्यापार को प्रभावित करती हैं-

1. जलवायु पर प्रभाव- ठण्डी धाराएँ तटवर्ती भागों की जलवायु को शुष्क बना देती हैं, क्योंकि इनके ऊपर से बहकर आने वाली पवनें शुष्क तथा शीतल होती हैं, जो वर्षा नहीं करा पातीं। यही कारण है कि बेग्युला की धारा, पीरू की धारा, कनारी की धारा, पश्चिमी ऑस्ट्रेलिया की धारा तथा कैलीफोर्निया की धारा के तटवर्ती भागों की जलवायु शुष्क मरुस्थली होती है। विश्व के विशाल
मरुस्थल इन्हीं धाराओं के कारण उष्ण तथा शुष्क हैं।

2. मत्स्याखेट पर प्रभाव-जहाँ गर्म तथा ठण्डी धाराएँ परस्पर मिलती हैं वहाँ सघन कोहरा पैदा होता है। यह दशा मछलियों के विकास के लिए उत्तम होती है तथा वहाँ मछली पकड़ने का कार्य बड़े पैमाने पर होता है। जापान के निकट गर्म क्यूरोसिवो तथा ठण्डी ओयाशियो की धारा और न्यूफाउण्डलैण्ड के निकट गर्म गल्फस्ट्रीम और ठण्डी लैब्रेडोर धाराओं के मिलने से यही दशा पैदा
होती है। ये क्षेत्र मत्स्याखेट के लिए उत्तम हैं।

3. व्यापार पर प्रभाव-उत्तरी गोलार्द्ध में उच्च अक्षांशों में जहाँ तटों के निकट गर्म धाराएँ बहती हैं, वे तट शीतकाल में भी हिमाच्छादित नहीं होते; अत: व्यापार के लिए खुले रहते हैं। उदाहरणार्थ-पश्चिमी यूरोप के तट गर्म उत्तरी अटलांटिक ड्रिफ्ट के कारण व्यापार के लिए वर्ष-भर खुले रहते हैं, जबकि उन्हीं अक्षांशों में स्थित साइबेरिया (रूस) के पूर्वी तट शीतकाल में जम जाते हैं।

प्रश्न 2. महासागरीय जलधारा एवं ज्वार-भाटे में अन्तर बताइए।
उत्तर-महासागरीय जलधाराएँ एवं ज्वार-भाटे दोनों ही महासागरीय गतियाँ हैं, किन्तु ज्वार-भाटा एक स्थानीय तथा अल्पकालिक गति है जिसमें सागर या महासागर तटों पर निश्चिंत समय पर दिन में दो बार सागरीय जल चढ़ता एवं उतरता रहता है। इनकी उत्पत्ति पृथ्वी पर चन्द्रमा तथा सूर्य की आकर्षण शक्ति के कारण होती है।

ज्वार-भाटों के विपरीत महासागरीय जलधाराओं में जल का संचलन (गतिं) निरन्तर होता रहता है। इनमें जले का प्रवाह एक निश्चित दिशा में होता है। इनकी उत्पत्ति पृथ्वी के घूर्णन (परिभ्रमण), प्रचलित पवनों, तापमानों एवं लवणता की भिन्नताओं तथा तटरेखा की आकृति के कारण होती है।

प्रश्न 3. किसी स्थान पर प्रतिदिन निश्चित समय पर ज्वार-भाटा क्यों नहीं आते हैं?
उत्तर-ज्वार प्रत्येक स्थान पर 24 घण्टे में दो बार आता है। लेकिन ज्वार आने का समय नियमित रूप से एक ही नहीं रहता है। इसका मुख्य कारण यह है कि पृथ्वी 24 घण्टे में अपनी कक्षा पर एक चक्कर पूरा करती है। पृथ्वी अपना यह चक्कर पश्चिम से पूर्व की ओर लगाती है। चन्द्रमा भी अपनी धुरी पर घूमते हुए पृथ्वी को चक्कर लगाता है; अत: चन्द्रमा अगले एक दिन में अपने निश्चित ज्वार केन्द्र से कुछ आगे बढ़ जाता है। इस कारण ज्वार केन्द्र को चन्द्रमा के इस नवीन केन्द्र के ठीक नीचे तक या चन्द्रमा के सामने पहुँचने में 52 मिनट का समय अधिक लगता है। इस प्रकार प्रति अगले दिन ज्वार केन्द्र को चन्द्रमा के सामने आने में कुल 24 घण्टे 52 मिनट लगते हैं। इसी कारण अगला ज्वार ठीक 12 घण्टे बाद न आकर 12 घण्टे 26 मिनट बादं आता है। यह 26 मिनट की देरी प्रतिदिन होती रहती है, जो कि ज्वार आने के प्रत्येक समय में परिवर्तन सिद्ध करती है। यही स्थिति ज्वार केन्द्र के विपरीत भाग में रहती है, जहाँ ज्वार 12 घण्टे 26 मिनट पश्चात् आता है।

प्रश्न 4. लघु तथा दीर्घ ज्वार में अन्तर स्पष्ट कीजिए।
उत्तर-लघु ज्वार-यह ज्वार कृष्ण पक्ष एवं शुक्ल पक्ष की सप्तमी और अष्टमी तिथियों में सूर्य और चन्द्रमा के पृथ्वी के साथ समकोणीय स्थिति में आने के कारण आता है। इस समकोणीय स्थिति के द्वारा सूर्य और चन्द्रमा पृथ्वी के महासागरीय जल को अनेक दिशाओं में आकर्षित करते हैं। इस कारण महासागरों में ज्वार की ऊँचाई अन्य तिथियों की अपेक्षा कम रह जाती है। दीर्घ

ज्वार-यह ज्वार पूर्णिमा एवं अमावस्या के दिन सूर्य, पृथ्वी एवं चन्द्रमा के एक सीध में स्थित होने के कारण आता है। इस स्थिति के कारण ज्वार की ऊँचाई अन्य दिनों की अपेक्षा बीस प्रतिशत अधिक हो जाती है।

प्रश्न 5. महासागरीय धाराएँ कितने प्रकार की होती हैं?
उत्तर-महासागरीय धाराएँ गति तथा तापमान के आधार पर दो वर्गों में विभाजित की जाती हैं
1. गति के आधार पर-गति के आधार पर महासागरीय धाराएँ दो प्रकार की होती हैं
(i) स्ट्रीम-तीव्र गति से प्रवाहित होने वाली धारा स्ट्रीम कहलाती है। उत्तरी अन्ध महासागर की गल्फस्ट्रीम इसका मुख्य उदाहरण है।
(ii) इिफ्ट-वे महासागरीय धाराएँ जो धीमी गति से प्रवाहित होती हैं उन्हें ड्रिफ्ट या अपवाह कहते हैं। ऑस्ट्रेलिया के दक्षिण में पश्चिमी अपवाह इसका एक अच्छा उदाहरण है।

2. तापमान के आधार पर-तापमान के आधार पर महासागरीय धाराएँ दो प्रकार की होती हैं
(i) गर्म धाराएँ-वे महासागरीय धाराएँ जिनका जल गर्म होता है, उन्हें गर्म जलधाराएँ कहते हैं। गल्फस्ट्रीम तथा क्यूरोसिवो आदि इसी प्रकार की जलधाराएँ हैं।
(ii) ठण्डी जलधाराएँ-वे महासागरीय जलधाराएँ जिनका जल ठण्डा होता है उनको ठण्डी | जलधारा कहते हैं। लैब्रेडोर, क्यूराइल आदि इसी प्रकार की धाराएँ हैं।

प्रश्न 6. ज्वार प्रतिदिन 50 मिनट विलम्ब से क्यों आता है?
या दो ज्वारों के बीच ठीक बारह घण्टे का अन्तर क्यों नहीं होता है?
उत्तर-ज्वार 24 घण्टे में प्रत्येक स्थान पर प्राय: दो बार आता है, परन्तु यह निश्चित रूप से एक ही समय पर नहीं आता है। इसका प्रमुख कारण यह है कि पृथ्वी 24 घण्टे में अपना पूरा एक चक्कर पश्चिम से पूरब दिशा में घूर्णन करते हुए लगाती है। चन्द्रमा भी अपने अक्ष पर घूर्णन करते हुए पृथ्वी का चक्कर लगाता है। पृथ्वी की इस परिभ्रमण गति से ज्वार पश्चिम से पूरब की ओर बढ़ता है। इस ज्वार केन्द्र के एक पूरे चक्कर के बाद भी चन्द्रमा अपनी गति से कुछ आगे निकल जाता है जिसका प्रमुख कारण पृथ्वी का परिक्रमण है। इस प्रकार ज्वार केन्द्र को चन्द्रमा के केन्द्र तक या चन्द्रमा के समक्ष पहुँचने में 52 मिनट का समय और लग जाता है। इस ज्वार केन्द्र को पुनः चन्द्रमा के समक्ष आने में कुल 24 घण्टे 52 मिनट लगते हैं। इसी कारण ज्वार ठीक 12 घण्टे बाद न आकर 12 घण्टे 26 मिनट के बाद आता है तथा यह 26 मिनट की देरी प्रतिदिन होती रहती है, जो कि ज्वार आने के प्रत्येक समय में परिवर्तन सिद्ध करती है। ऐसी ही स्थिति ज्वार केन्द्र के विपरीत भागों में रहती है।

प्रश्न 7. ज्वार-भाटा के लाभ तथा तटीय क्षेत्रों पर इसका प्रभाव बतलाइए।
उत्तर-ज्वार-भाटा का प्रभाव नौका-परिवहन पर अत्यधिक पड़ता है। ज्वार द्वारा कुछ नदियाँ बड़े जलयानों के चलाने योग्य बन जाती हैं। हुगली तथा टेम्स नदियाँ ज्वारीय धाराओं के कारण ही नाव योग्य हो सकी हैं तथा कोलकाता व लन्दन महत्त्वपूर्ण पत्तन बन सके हैं। इसी कारण ये दोनों पतन नदी पत्तन कुहलाते हैं। ज्वारीय ऊर्जा को भी आज महत्त्वपूर्ण समझा जाने लगा है। ज्वर के द्वारा समुद्रतटीय नगरों के कूड़ा-करकट व गन्दगी के ढेर प्रतिदिन बहकर समुद्र में चले जाते हैं। ज्वार-भाटा का तटीय भागों में प्रभाव व्यापक स्तर पर दिखाई देता है। ज्वार के समय मछलियाँ व अनेक मूल्यवान पदार्थ समुद्री किनारे पर आ जाते हैं, जहाँ किनारे उथले हैं वहाँ नौकाएँ तट पर आसानी से पहुँच जाती हैं। इस प्रकार ज्वार-भाटा का समीपवर्ती जन-जीवन पर पर्याप्त प्रभाव पड़ता है।

प्रश्न 8. उत्तर-पश्चिी यूरोप के तटीय क्षेत्रों को उत्तरी अटलाण्टिक प्रवाह से प्राप्त दो प्रमुख लाभों का उल्लेख कीजिए।
उत्तर-1. उत्तरी अटलाण्टिके प्रवाह के परिणामस्वरूप गर्म जल प्रवाहमान रहता है तथा तटीय भाग अधिक गर्म बने रहते हैं। अत: उत्तर-पश्चिमी यूरोप का तटीय क्षेत्र जलयानों के लिए वर्षभर खुला रहता है।

2. गर्म तथा ठण्डी जलधाराओं के मिलन-स्थल पर कुहरा अधिक पड़ता है। ऐसे स्थान मत्स्य उत्पादन के लिए उर्वर होते हैं। उत्तरी अटलाण्टिक प्रवाह के कारण उत्तर-पश्चिमी यूरोप के तटीय क्षेत्र मत्स्य उत्पादन में विश्व में प्रमुख स्थान बनाए हुए हैं।

प्रश्न 9. ज्वार-भाटा क्या है। इसके समय और ऊँचाई में विभिन्नता की विवेचना कीजिए।
उत्तर-सागर एवं महासागरीय जल की गतियों में ज्वार-भाटा अत्यन्त महत्त्वपूर्ण गति है, क्योंकि चन्द्रमा एवं सूर्य के आकर्षण से उत्पन्न ज्वारीय तरंगें नियमित रूप से ऊपर उठतीं तथा नीचे गिरती हैं, इससे सागर का जल आगे-पीछे होता रहता है। सागरीय जल सूर्य और चन्द्रमा की आकर्षण शक्ति द्वारा एकत्रित होकर जब तीव्रता से ऊपर उठता है तो उसे ज्वार कहते हैं। जिन स्थानों से जल खिंचकर आता है वहाँ जल का तल नीचा हो जाता है, जो भाटा कहलाता है।

ज्वार एवं भाटी के मध्य सागरीय जल सतह का अन्तर ज्वार-परिसर कहलाता है। खुले सागरों में जल की गति अधिक होने पर यह मात्र एक या दो फुट होता है, परन्तु उथले सागरों एवं खाड़ियों में स्थल के घर्षण के कारण ज्वार का अन्तर अधिक पाया जाता है। ब्रिटेन की टेम्स नदी के मुहाने पर 23 फुट का अन्तर देखा गया है, जबकि कनाडा की फण्डी खाड़ी में 70 फुट का अन्तर पाया गया है। इसके अतिरिक्त विभिन्न स्थानों पर ज्वार-भाटा की ऊँचाई में भी भिन्नता पाई जाती है। ज्वार की ऊँचाई पर तट रेखा का प्रभाव पड़ता है। उदाहरण के लिए झीलों में ज्वार की ऊँचाई केवल कुछ इंच ही होती है। भूमध्य सागर एवं बाल्टिक सागर में भी केवल 2 फुट ऊँचे ज्वार आते हैं। यही नहीं, विभिन्न स्थानों पर ज्वार-भाटा आने का समय भी भिन्न-भिन्न होता है। इसी कारण ज्वार की अवधि भी प्रत्येक स्थान पर भिन्न-भिन्न हो जाती है।

प्रश्न 10. महासागरीय जलधाराओं के मानव-जीवन पर पड़ने वाले किन्हीं दो प्रभावों का वर्णन कीजिए।
उत्तर-महासागरीय जलधाराओं का निकटवर्ती क्षेत्रों के मानव-जीवन पर गहरा प्रभाव पड़ता है। इसके दो प्रभाव निम्नलिखित हैं—

1. ठण्डी एवं गर्म जलधाराओं के मिलने से धुन्ध एवं कुहरा उत्पन्न होता है जो मत्स्य व्यवसाय के लिए आदर्श भौगोलिक सुविधाएँ प्रदान करता है। इस प्रकार तटवर्ती क्षेत्रों में मानव के आर्थिक व्यवसाय का विकास होता है। उदाहरण के लिए–गल्फस्ट्रीम की गर्म जलधारा तथा लैब्रेडोर की ठण्डी जलधाराओं से न्यूफाउण्डलैण्ड के समीप ग्राण्ड बैंक मछली पकड़ने के प्रमुख केन्द्र के रूप
में विकसित हुआ है।

2. महासागरीय धाराओं का निकटवर्ती क्षेत्रों की जलवायु, वर्षा, व्यापार एवं कृषि पर गहरा एवं व्यापक प्रभाव पड़ता है। उदाहरण के लिए-जापान के पश्चिमी तट पर प्रवाहित गर्म क्यूरोसिवों धारा की सुशिमा शाखा के कारण तापमान पूर्वी तट की अपेक्षा अधिक रहता है तथा यहाँ कृषि कार्य के लिए उपयुक्त जलवायु-दशाएँ उपलब्ध हो जाती हैं।

प्रश्न 11. तरंगों की मुख्य विशेषताएँ बतलाइए।
या तरंगों के सम्बन्ध में प्रमुख तथ्यों का विवरण दीजिए।
उत्तर-तरंगों के सम्बन्ध में निम्नलिखित तथ्य या विशेषताएँ महत्त्वपूर्ण हैं

  1. तरंग-शिखर एवं गर्त-एक तरंग के उच्चतम एवं निम्नतम बिन्दुओं को क्रमशः तरंग का शिखर एवं गर्त कहा जाता है।
  2. तरंग की ऊँचाई-तरंग की गति के अधःस्थल से शिखर के ऊपरी भाग तक की ऊर्ध्वाधर दूरी तरंग की ऊँचाई होती है (चित्र 14.3)।
  3. तरंग आयाम-तरंग की ऊँचाई का आधा उसका आयाम कहलाता है।
  4. तरंग काल-तरंग काल एक निश्चित बिन्दु से गुजरने वाले दो लगातार शिखरों या गर्तों के बीच का समयान्तराल है।
  5. तरंगदैर्ध्य-दो लगातार शिखरों या गर्ती के बीच की क्षैतिज दूरी तरंगदैर्ध्य कहलाती है।
  6. तरंग गति-जल के माध्यम से तरंग की गति करने की दर को तरंग गति कहते हैं। इसे नॉट में मापा जाता है।
  7. तरंग आकृति-एक सेकण्ड के समय अन्तराल में दिए गए बिन्दु से गुजरने वाली तरंगों की संख्या तरंग आवृत्ति कहलाती है।
    UP Board Solutions for Class 11 Geography Fundamentals of Physical Geography Chapter 14 Movements of Ocean Water (महासागरीय जल संचलन) img 4

प्रश्न 12. तरंगें कैसे निर्मित होती हैं? इनके मुख्य प्रकार बताईए।
उत्तर-तरंगें महासागरीय जल में दोलन गति की प्रतीके हैं। इनसे समुद्री धरातल निरन्तर ऊपर उठता और नीचे धंसता रहता है। मुख्य रूप से तरंगों का निर्माण पवनों से होता है। समुद्र की सतह पर जब पवन दाब और घर्षण के रूप में अपनी ऊर्जा का प्रयोग करता है तबै तरंगें उत्पन्न होती हैं। पवन द्वारा निर्मित ये तरंगें निम्नलिखित तीन प्रकार की होती हैं

  • सी-ये जटिल और परिवर्तनशील स्वभाव की तरंगें हैं। ये अस्त-व्यस्त तरंगी रूपरेखा का निर्माण करती हैं।
  • स्वेल-ये नियमित तरंगें हैं जो समान ऊँचाई और आवर्तकाल के साथ एक निश्चित रूप में | प्रवाहित होती हैं।
  • सर्फ-तटीय क्षेत्रों में इन टूटती हुई तरंगों को सर्फ (फोनिल लहर) कहते हैं।

प्रश्न 13. महासागरीय धाराएँ क्या हैं? इनकी मुख्य विशेषताएँ बताइए।
उत्तर-जब सागरों एवं महासागरों का जल नियमित रूप से एक निश्चित दिशा में प्रवाहित होने लगता है तो उसे महासागरीय जलधारा कहते हैं। जलधाराएँ सागरों में उसी प्रकार प्रवाहित होती हैं जैसे स्थलीय भागों में नदियाँ एवं नाले। जलधाराओं को सागरीय नदियाँ भी कह सकते हैं। सामान्यत: धाराओं की गति 2 से 10 किमी प्रति घण्टा तक ऑकी गई है। गति एवं विस्तार आदि की दृष्टि से जलधाराओं को ड्रिफ्ट, धारा एव स्ट्रीम आदि नामों से पुकारा जाता है। जलधाराएँ ठण्डी एवं गर्म दोनों प्रकार की ह्येती हैं अर्थात् जो जलधाराएँ। ध्रुवों की ओर से प्रवाहित होती हैं, वे ठण्डी जलधाराएँ कहलाती हैं, जबकि विषुवत् रेखा से प्रवाहित होने वाली जलधाराएँ गर्म जल की धाराएँ कहलाती हैं।

प्रश्न 14. दक्षिणी प्रशान्त महासागर की प्रमुख जलधाराओं का वर्णन कीजिए।
उत्तर-दक्षिणी प्रशान्त महासागर की जलधाराएँ
दक्षिणी प्रशान्त महासागर की प्रमुख जलधाराएँ निम्नलिखित हैं
1. पीरू जलधारा-अण्टार्कटिक प्रवाह दक्षिणी अमेरिका के दक्षिणी भाग से टकराकर केपहार्न अन्तरीप के निकट उततर की ओर को मुड़ जाता है। पीरू के तट पर पहुँचने के कारण इसे पीरू की धारा के नाम से पुकारते हैं। यह धारा उत्तर की ओर दक्षिणी विषुवत् धारा से जा मिलती है। यह एक शीतल जलधारा है जिसे सर्वप्रथम हम्बोल्ट नाम के नाविक ने देखा था; अतः उनके नाम
पर इसे हम्बोल्ट की धारा भी कहा जाता है।

2. दक्षिणी विषुवत्रखीय धारा-यह एक उष्ण जलधारा है जिसकी उत्पत्ति दक्षिणी गोलार्द्ध की दक्षिणी-पूर्व सन्मार्गी पवनों के कारण होती है। यह विषुवत् रेखा के दक्षिण में 3° से 10° अक्षांशों के मध्य प्रवाहित होती है। यह धारा दक्षिणी अमेरिका के पश्चिमी तटों से होती हुई ऑस्ट्रेलिया तक पहुँचती है। न्यूगिनी तट के निकट यह दो शाखाओं में बँट जाती है। यह धारा 1,300 किमी की दूरी तक प्रवाहित होती है।

3. पूर्वी ऑस्ट्रेलियन धारा-न्यूगिनी तट के निकट दक्षिणी विषुवत्रेखीय धारा दो शाखाओं में विभक्त हो जाती है। इसकी जो शाखा ऑस्ट्रेलिया के पूर्वी तट पर प्रवाहित होती है उसे पूर्वी ऑस्ट्रेलियन गर्म जलधारा अथवा न्यूसाउथ की धारा के नाम से पुकारते हैं।

4. अण्टार्कटिक-प्रवाह-यह एक ठण्डी जलधारा है जिसकी उत्पत्ति अण्टार्कटिक महासागरीय भागों में होती है। यह धारा पछुवा पवनों के प्रभाव से पश्चिम से पूर्व की ओर प्रवाहित होती है। यह धारा बहुत मन्द गति से प्रवाहित होती है।

प्रश्न 15. अन्ध महासागर एवं प्रशान्त महासागर की जलधाराओं का समीपवर्ती क्षेत्र की जलवायु पर प्रभाव बताइए।
उत्तर- अन्ध महासागर की धाराओं का समीपवर्ती क्षेत्रों की जलवायु पर प्रभाव
उत्तरी अन्ध महासागर में यूरोप का पश्चिमी तट उत्तर अटलाण्टिक ड्रिफ्ट के गर्म जल के प्रभाव से जाड़ों में यूरेशिया के भीतरी भागों व कनाडा के पूर्वी तट की अपेक्षा लगभग 10° से 15° सेल्सियस तक अधिक गर्म रहता है। इसी कारण नॉर्वे का तट व्यापार के लिए जाड़ों में भी खुला रहता है, जबकि उन्हीं अक्षांशों में स्थित साइबेरिया का तट हिम से जम जाता है। कनारी (उत्तर) और बेंगुला (दक्षिण) की ठण्डी धाराओं के कारण पश्चिमी अफ्रीका के उष्ण मरुस्थलीय तटों पर भीतरी भागों की अपेक्षा अधिक आर्द्रता रहँती है। जहाँ कहीं ठण्डी या गर्म जलधाराएँ मिलती हैं, वहाँ कुहरा अधिक पड़ता है। ऐसे क्षेत्र मत्स्य उत्पादन के लिए उत्तम होते हैं, क्योंकि इस प्रकार की जलवायु में मछलियों का भोज्य पदार्थ-प्लेंकटन अधिक मात्रा में उत्पन्न होता है।

समीपवर्ती क्षेत्रों की जलवायु पर प्रशान्त महासागर की धाराओं का प्रभाव ।
प्रशान्त महासागर की सभी धाराएँ अपने निकटवर्ती क्षेत्रों की जलवायु पर गहरा प्रभाव डालती हैं। जापान के तट पर ठण्डी एवं गर्म जलधाराओं के मिलने से धुन्ध एवं कुहरा उत्पन्न हो जाता है जिससे यहाँ भारी मात्रा में मछलियाँ पकड़ी जाती हैं। इसके अतिरिक्त इन जलधाराओं का निकटवर्ती क्षेत्रों की जलवायु, वर्षा, व्यापार, कृषि एवं सागरीय जीव-जन्तुओं पर भी गहरा एवं व्यापक प्रभाव पड़ता है। उदाहरण के लिए-जापान के पश्चिमी तट पर प्रवाहित गर्म क्यूरोसिवो धारा की सुशिमा शाखा के कारण तापमान पूर्वी तट की अपेक्षा अधिक रहता है जबकि पश्चिमी तट साइबेरिया से आने वाली अति ठण्डी पवनों के लिए भी खुला रहता है। गर्म अलास्का धारा के कारण अलास्को का तट भीलरी भागों की अपेक्षा जाड़ों में अधिक उष्ण रहता है। ठण्डी कैलीफोर्निया धारा भी गर्मियों में उत्तरी अमेरिका के पश्चिमतटीय भागों की जलवायु को शीतल कर देती है, जबकि भीतरी मरुस्थलीय भागों में उच्च ताप पाया जाता है। ||

दीर्घ उत्तरीय प्रश्न

प्रश्न 1. सागरीय धाराएँ किन्हें कहते हैं? प्रशान्त महासागर की धाराओं का वर्णन कीजिए।
या महासागरीय जलधाराएँ क्या हैं? इनकी उत्पत्ति के कारणों की सोदाहरण विवेचना कीजिए।
या उत्तरी प्रशान्त महासागर की प्रमुख जलधाराओं का वर्णन कीजिए तथा उनमें से किसी एक धारा के समीपवर्ती क्षेत्र की जलवायु पर प्रभाव बताइए।
या टिप्पणी लिखिए—(क) उत्तरी प्रशान्त महासागर की प्रमुख धाराएँ। (ख) क्यूरोसिवो जलधारा।।
उत्तर-सागरीय धाराएँ
जिस प्रकार धरातल पर नदियाँ प्रवाहित होती हैं, उसी प्रकार सागरों एवं महासागरों में धाराएँ एक विशाल जलराशि के साथ एक दिशा में प्रवाहित होती हैं। ये धाराएँ अधिक शक्तिशाली एवं प्रभावशाली होती हैं। ये धाराएँ महासागरों के ऊपरी तल पर ही नहीं, अपितु सागरों की निश्चित गहराई में भी एक निश्चित दिशा में प्रवाहित होती हैं। कुछ धाराएँ महाद्वीपीय तट के किनारे-किनारे प्रवाहित होती हैं, जबकि कुछ सागरों के बीचों-बीच प्रवाहित होती हैं, जिन्हें सागरीय धाराओं के नाम से जाना जाता है।

धाराओं की उत्पत्ति के कारण
1. तापमान की भिन्नता-सागरीय जल के तापमान में क्षैतिज एवं लम्बवत् भिन्नता पायी जाती है। भूमध्यरेखीय क्षेत्रों में अधिक गर्मी के कारण समुद्री जल गर्म हो जाता है, जबकि उच्च एवं ध्रुवीय क्षेत्रों में जल निम्न तापमान के कारण ठण्डा होकर नीचे बैठ जाता है, जिस कारण भूमध्यरेखीय क्षेत्रों का जल ध्रुवों की तरफ प्रवाहित होने लगता है। उत्तरी एवं दक्षिणी भूमध्यरेखीय धाराएँ इसी प्रकार की धाराएँ हैं।

2. लवणता की भिन्नता-महासागरीय जल में लवणता में पर्याप्त भिन्नता पायी जाती है। लवणता की भिन्नता से सागरीय जल का घनत्व परिवर्तित हो जाता है। उत्तरी कम घनत्व वाले भागों या कम लवणता वाले भागों से जल अधिक लवणता वाले जलीय भागों की ओर प्रवाहित होने लगता है, जिससे जलधाराओं की उत्पत्ति हो जाती है। हिन्द महासागर में लाल सागर की तरफ इसका प्रवाह उत्तम उदाहरण है।

3. प्रचलित पवनों का प्रभाव-प्रचलित पवनें वर्ष-भर नियमित रूप से प्रवाहित होती हैं और ये पवनें अपने मार्ग में पड़ने वाली जलराशि को पवन की दिशा के अनुकूल धकेलती हैं और जलराशि प्रवाहित होने लगती है। पछुवा हवाओं के प्रभाव से क्यूरोसिवो व गल्फस्ट्रीम की धाराओं को गति |व दिशा मिलती है।

4. वाष्पीकरण व वर्षा-पृथ्वीतल पर वाष्पीकरण वे वर्षा में पर्याप्त भिन्नता पायी जाती है। जहाँ वाष्पीकरण अधिक होता है वहाँ सागरतल नीचा हो जाता है; अत: उच्च तल के क्षेत्रों से सागरीय जल प्रवाहित होने लगता है और जलधाराओं का निर्माण हो जाता है। ठीक इसी प्रकार अधिक वर्षा के क्षेत्रों में सागर जल-तल में वृद्धि हो जाती है और ऐसे क्षेत्रों में जल निम्न वर्षा वाले तथा निम्न जल-तल वाले भागों की तरफ एक धारा के रूप में बहने लगता है।

5. पृथ्वी की दैनिक गति-पृथ्वी अपनी कीली पर तीव्र गति से घूमती हुई 24 घण्टे में एक पूरा चक्कर लगा लेती है। पृथ्वी की घूर्णन गति के कारण ही सागरीय जल उत्तरी गोलार्द्ध में दाहिनी ओर तथा दक्षिणी गोलार्द्ध में बायीं ओर घूम जाता है। पृथ्वी की घूर्णन गति को प्रभाव जलधाराओं के प्रवाह एवं गति पर पड़ता है।।

इसके अलावा वायुभार की भिन्नता, ऋतु-परिवर्तन, समुद्रतटीय आकृति आदि ऐसे कारक हैं जो महासागरों में जलधाराओं की उत्पत्ति में सहायक होते हैं।

प्रशान्त महासागर की धाराएँ

प्रशान्त महासागर की प्राकृतिक संरचना तथा जल-तल की स्थिति में परिवर्तन के कारण इन धाराओं की दिशा एवं गति में भी परिवर्तन होता रहता है। प्रशान्त महासागर की धाराओं का विवरण निम्नलिखित है

1. उत्तरी भूमध्यरेखीय गर्म धारा-इस जलधारा की उत्पत्ति उत्तरी एवं दक्षिणी अमेरिका महाद्वीपों के मिलन-स्थल पर पश्चिमी दिशा से होती है। यह जलधारा इस तट के पश्चिम की ओर चलकर सम्पूर्ण महासागर से होती हुई फिलीपाइन द्वीप समूह से टकराती है तथा उत्तर की ओर मुड़ जाती है। यहाँ पर इसकी दो शाखाएँ हो जाती हैं–पहली शाखा दक्षिण की ओर मुड़कर अपनी दिशा पूर्व की ओर मोड़ लेती है, जिसे प्रति-भूमध्यरेखीय धारा के नाम से पुकारते हैं। दूसरी शाखा उत्तर की ओर ताईवान द्वीप पर पहुँचती है तथा यही शाखा आगे की ओर क्यूरोसिवो की धारा बन जाती है। शीत ऋतु में इसकी दक्षिणी सीमा 5° उत्तरी अक्षांश तथा ग्रीष्म ऋतु में 10° उत्तरी अक्षांश पर स्थिर रहती है।

2. दक्षिणी भूमध्यरेखीय गर्म धारा-दक्षिणी अमेरिका महाद्वीप के पश्चिमी भाग में लगभग 3°से 10° दक्षिणी अक्षांशों के मध्य यह धारा पश्चिम की ओर प्रवाहित होती है। इसकी गति तीव्र एवं मन्द होती रहती है। इस धारा की औसत गति 30 किमी प्रतिदिन है, परन्तु कभी-कभी यह 150 किमी प्रतिदिन की गति से भी प्रवाहित होती है। पश्चिम की ओर प्रवाहित होती हुई यह जलधारा ऑस्ट्रेलिया के उत्तर में न्यूगिनी तट से टकराती है, तो दो भागों में विभाजित हो जाती है। पहली शाखा उत्तर की ओर प्रति-भूमध्यरेखीय धारा से मिल जाती है तथा इसकी दिशा में पूर्व की ओर परिवर्तन हो जाता है। दूसरी शाखा ऑस्ट्रेलिया के पूर्वी भाग में दक्षिण की ओर प्रवाहित होती हुई पुनः अपनी दिशा पूरब की ओर मोड़ लेती है।

3. प्रति-भूमध्यरेखीय धारा-जिस प्रकार उत्तरी तथा दक्षिणी हिन्द महासागर में प्रति- भूमध्यरेखीय धारा की स्थिति है, उसी प्रकार उत्तरी तथा दक्षिणी प्रशान्त महासागर के मध्य प्रति- भूमध्यरेखीय धारा प्रवाहित होती है। यह धारा दो कारणों से चलती है—पहला कारण पूरब से पश्चिम की ओर बहती हुई उत्तरी तथा दक्षिणी भूमध्यरेखीय धारा। जब यह फिलीपाइन द्वीप समूह तथा न्यूगिनी द्वीप से टकराती है तो अपने दायीं-बायीं ओर घूम जाती है। अन्दर की ओर दोनों जलधाराओं का जल प्रति- भूमध्यरेखीय धारा को जन्म देता है। यह धारा पश्चिम से पूरब की ओर प्रवाहित होती है। इस धारा के प्रवाह का दूसरा कारण पश्चिम से पूरब की ओर प्रवाहित होने वाली व्यापारिक हवाओं द्वारा एकत्रित जल को पूरब की ओर सामान्य ढाल से प्रवाहित कराना है तथा इसका पनामा की खाड़ी में प्रवेश कर जाना है।।

4. क्यूरोसिवो धारा-गर्म जल की यह धारा अपने विकास तथा गति में एक परिपक्व धारा है। इसमें अनेक धाराएँ मिली हुई हैं। इसका प्रवाह क्षेत्र ताईवाने द्वीप से लेकर बेरिंग जलडमरूमध्य तक है। इसकी निम्नलिखित शाखाएँ हैं

(i) क्यूरोसिवो की गर्म जलधारा-यह उत्तरी भूमध्यरेखीय गर्म धारा का ही सिलसिला है। जो उत्तर की ओर मुड़ते ही क्यूरोसिवो धारा के नाम से जानी जाती है। इसका प्रवाह क्षेत्र | ताईवान द्वीप से रियूक्यू द्वीप तक है।

(ii) क्यूरोसिव प्रसरण-जापान के पूर्वी तट पर 35° उत्तरी अक्षांश के समीप क्यूरोसिवो धारा अपने जल का दो भागों में प्रसरण कर देती है। इसका कारण पृथ्वी की परिभ्रमण गति तथा पछुवी हवाओं का प्रवाहित होना है। इसकी पहली शाखा पूरब की ओर चली जाती है। तथा दूसरी उत्तर-पूर्व की ओर तट का अनुसरण करते हुए आगे की ओर बढ़ जाती है। लगभग 55° उत्तरी अक्षांश पर जापान के होकैडो द्वीप के पूर्वी तट पर, दूसरी शाखा उत्तर की ओर से आने वाली क्यूराइल की ठण्डी धारा से टकराकर पूर्व की ओर मुड़ जाती है। ठण्डी एवं गर्म जलधाराओं के मिलन से यहाँ पर भारी कोहरा पड़ता रहता है।

(iii) उत्तरी प्रशान्त महासागरीय प्रवाह-पछुवा पवनों के प्रभाव एवं क्यूरोसिवो धारा के बहने के साथ यह धारा पूर्व की ओर प्रवाहित होती है तथा 150° पूर्वी देशान्तर के समीप अपनी गति के कारण इसके जल का कुछ भाग दक्षिण की ओर मुड़ जाता है तथा इससे एक धारा निर्मित होती है। यह धारा उत्तरी अमेरिका महाद्वीप के पश्चिमी तट से टकराकर दो भागों में बँट जाती है—पहली शाखा दक्षिण की ओर कैलीफोर्निया की धारा के नौम से जानी जाती है। इसकी दूसरी शाखा पुनः दो शाखाओं में बँटकर बेरिंग सागर तथा अलास्का की खाड़ी में विलीन हो जाती है।

(iv) सुशिमा धारा-क्यूरोसिवो की गर्म जलधारा की यह उपशाखा जापान के दक्षिण में शिकोकू द्वीप से टकराती है जिससे इसका कुछ जल दक्षिण में स्थित जापान सागर में प्रवेश कर जाता है, जिसे सुशिमा धारा के नाम से पुकारा जाता है।

(v) प्रति-क्यूरोसिवो धारा-यह एक छोटी उपधारा है, जो क्यूरोसिवो के उत्तर-पूर्व से पश्चिम की ओर प्रवाहित होती है। इसकी स्थिति उत्तरी अमेरिका तथा हवाई द्वीप के मध्य में है।

5. क्यूराइल की ठण्डी जलधारा-ठण्डे जल की यह धारा ओयाशियो के नाम से भी जानी जाती है। आर्कटिक महासागर से बहता हुआ जलबेरिंग सागर से प्रवाहित होकर दक्षिण की ओर जाता है। जापान के उत्तर में 50° उत्तरी अक्षांश के समीप दक्षिण से प्रवाहित गर्म क्यूरोसिवो की धारा से टकराकर इसका प्रवाह पूरब की ओर हो जाता है तथा कुछ जल जापान तट के सहारे-सहारे दक्षिण की ओर प्रवाहित हो जाता है।

6. कैलीफोर्निया धारा-ठण्डे जल की यह धारा कैलीफोर्निया खाड़ी की धारा के नाम से भी जानी जाती है। यह उत्तरी प्रशान्त महासागरीय धारा का अग्रभाग है। इस जलधारा पर उत्तर-पश्चिम तथा दक्षिण की ओर से प्रवाहित व्यापारिक पवनों का अधिक प्रभाव पड़ता है। कैलीफोर्निया खाड़ी के सहारे- सहारे यह उत्तर-दक्षिण प्रवाहित होती है तथा दक्षिण की ओर उत्तरी भूमध्यरेखीय धारा में मिल जाती है।

7. अलास्का की धारा-उत्तरी प्रशान्त महासागरीय प्रवाह की एक शाखा उत्तरी अमेरिकी पश्चिमी तट पर प्रवाहित होती हुई अलास्का की खाड़ी में चली जाती है, जिसे अलास्का की ठण्डी जलधारा के नाम से जाना जाता है।

8. पीरू धारा-यह धारा दक्षिण प्रशान्त महासागर में दक्षिणी अमेरिका के पश्चिमी तट पर दक्षिण से उत्तर की ओर प्रवाहित होने वाली ठण्डे जल की मुख्य धारा है। पीरू तट से प्रवाहित होने के कारण इसका नाम पीरू धारा पड़ा। इसे हम्बोल्ट धारा भी कहते हैं। इसकी औसत गति 27 किमी प्रतिदिन है। यह 150 किमी की चौड़ाई में प्रवाहित होती है।

9. पूर्वी ऑस्ट्रेलियन धारा-यह दक्षिणी भूमध्यरेखीय धारा का अग्रभाग है, जो ऑस्ट्रेलिया के पूर्वी तट पर उत्तर से दक्षिण की ओर प्रवाहित होती है। यह एक गर्म जल की धारा है जो यहाँ के तापमान को ऊँचा बनाये रखने में सहायक है। 40° दक्षिणी अक्षांश पर पछुवा पवनों तथा पृथ्वी के परिभ्रमण गति के कारण यह धारा पूरब की ओर मुड़ जाती है जो आगे चलकर दक्षिणी अमेरिका के पश्चिमी तट तक पहुँच जाती है।

10. पछुवा-पवन-प्रवाह-दक्षिणी प्रशान्त महासागर में पछुवा हवाओं के प्रभाव से यह धारा पश्चिम से पूरब की ओर अधिक विस्तार के साथ गरजती हुई प्रवाहित होती है, जिसे पछुवा-पवन-प्रवाह के नाम से जाना जाता है। लगभग 45° दक्षिणी अक्षांश पूरब की ओर इसकी दो शाखाएँ हो जाती हैं—पहली हार्न अन्तरीप से होकर अन्ध महासागर में प्रवेश कर जाती है तथा दूसरी उत्तर की ओर प्रवाहित होती हुई पीरू की ठण्डी जलधारा से मिल जाती है।

जलवायु पर प्रभाव–प्रशान्त महासागर की सभी धाराएँ अपने निकटवर्ती क्षेत्रों की जलवायु पर गहरा प्रभाव डालती हैं। जापान के तट पर ठण्डी एवं गर्म जलधाराओं के मिलने से धुन्ध एवं कोहरा उत्पन्न होता है, जिससे यहाँ पर भारी मात्रा में मछलियाँ पकड़ी जाती हैं। इसके लिए जलधाराएँ ही उत्तरदायी हैं। इसके अतिरिक्त इन जलधाराओं का निकटवर्ती क्षेत्रों की जलवायु, वर्षा, व्यापार, कृषि एवं सांगरीय जीव-जन्तुओं पर भी गहरा एवं व्यापक प्रभाव पड़ता है। उदाहरण के लिए, जापान के पश्चिमी तट पर बहने वाली गर्म क्यूरोसिवो धारा की सुशिमा शाखा द्वारा ताप पूर्वी तट की अपेक्षा ऊँचे रहते हैं, जबकि पश्चिमी तट साइबेरिया से आने वाली अति ठण्डी हवाओं के लिए भी खुला है। गर्म अलास्का धौरा के कारण अलास्का का तट भीतरी भागों की अपेक्षा जाड़ों में गर्म रहता है। ठण्डी कैलीफोर्निया धारा भी गर्मियों में तटीय भागों की जलवायु को शीतले कर देती है, जबकि भीतरी मरुस्थलीय भाग में उच्च ताप पाया जाता है।

प्रश्न 2. अन्ध महासागर की प्रमुख जलधाराओं का वर्णन कीजिए तथानिकटवर्ती क्षेत्र पर उनका प्रभाव स्पष्ट कीजिए।
या दक्षिणी अन्ध महासागरीय धाराओं का वर्णन कीजिए तथा तटवर्ती जलवायु पर उनके प्रभाव की विवेचना कीजिए।
या उत्तरीय प्रशान्त महासागर की धाराओं का वर्णन कीजिए।
या टिप्पणी लिखिए–गल्फस्ट्रीम। ।
या अटलाण्टिक महासागर की धाराओं का वर्णन कीजिए।
या उत्तरीय अन्ध महासागर की किन्हीं दो धाराओं का तटीय जलवायु पर प्रभाव का वर्णन कीजिए।
उत्तर-अन्ध महासागर की प्रमुख जलधाराएँ
अन्ध महासागर का जल एक निश्चित प्रणाली के अनुरूप प्रवाहित होता है जो निश्चित क्रम वाली जलधाराओं को जन्म देता है। अन्ध महासागर में उत्तरी तथा दक्षिणी जलधाराओं का पृथक्-पृथक् क्रम पाया जाता है। अतः अन्ध महासागर की प्रमुख जलधाराओं का वर्गीकरण निम्नलिखित प्रकार से किया जा सकता है

(अ) उत्तरी अन्ध महासागर की जलधाराएँ
उत्तरी अन्ध महासागर की प्रमुख जलधाराएँ निम्नलिखित हैं

1. उत्तरी विषुवतरेखीय धारा-विषुवत् रेखा के उत्तरी भाग में उत्तरी-पूर्वी व्यापारिक पवनों के कारण जो गर्म जलधारा पूर्व से पश्चिम की ओर बहती है, उत्तरी विषुवत्रेखीय जलधारा कहलाती है। यह जल-धारा 0° अक्षांश से 10° अक्षांशों के मध्य प्रवाहित होती है। पश्चिमी द्वीप समूह के निकट यह धारा तट से टकराकर दो शाखाओं में बँट जाती है। इसकी एक शाखा उत्तर की ओर अमेरिका के तट के सहारे प्रवाहित होने लगती है जिसे एण्टिलीज धारा कहा जाता है। इसकी दूसरी शाखा कैरेबियन सागर में प्रवेश करके आगे जाकर यूकाटन चैनल में सम्मिलित हो जाती है।

2. गल्फस्ट्रीम-मल्फस्ट्रीम नामक गर्म जलधारा की उत्पत्ति मैक्सिको की खाड़ी से होती है। अतः इसे खाड़ी धारा या गल्फस्ट्रीम की संज्ञा दी गयी है। यह धारा 20° उत्तरी अक्षांश से 60° उत्तरी अक्षांशों के मध्य बहती है। इस धारा का तापमान 28° सेग्रे तथा गति 5 किमी प्रति घण्टा है। यह 1 किमी गहरी तथा 48 किमी चौड़ी है। यह खाड़ी से आगे उत्तरी अमेरिका के पूर्वी तट के सहारे-सहारे बहकर फ्लोरिडा तथा एण्टिलीज की धाराओं का सहयोग लेकर आगे बढ़ती है। न्यूफाउण्डलैण्ड तट के निकट इसका संगम लैब्रेडोर की ठण्डी जलधारा से होने के कारण घना कोहरा उत्पन्न होता है। पछुवा पवनों के प्रभाव के कारण यह धारा कई उपशाखाओं में बँट जाती है। इस धारा की एक शाखा पूर्व की ओर बहकर सारगैसो सागर में विलीन हो जाती है। इसका कुछ भाग उत्तरी-पश्चिमी यूरोप से टकरा कर कनारी की धारा में जुड़ जाता है। उत्तरी अटलाण्टिक ड्रिफ्ट इसका ही अगला भाग़ है। आइसलैण्ड एवं इंग्लैण्ड के मध्य पहुँचकर गल्फस्ट्रीम का रूप समाप्त हो जाता है।

3. कनारी की धारा-यह एक ठण्डी ज्ञलधारा है जो अफ्रीका के तट के सहारे मेडीरा से केपवर्डे तक प्रवाहित होती है। व्यापारिक पवनें इस धारा को दिशा और गति प्रदान करती हैं। ये पवनें इस धारा का रुख सारगैसो सागर की ओर को कर देती हैं। सागर की तली को शीतल जल ऊपर आकर कनारी की धारा के रूप में बहने लगता है।

4. लैब्रेडोर की धारा-यह भी शीतल जलधारा है जो ग्रीनलैण्ड के उत्तर-पश्चिम में बैफीन की खाड़ी एवं डेविस जलडमरूमध्य से दक्षिण की ओर लैब्रेडोर के तट के सहारे-सहारे बहती है। न्यूफाउण्डलैण्ड के तट के निकट यह गल्फस्ट्रीम धारा से मिल जाती है और घना कोहरा उत्पन्न करती है।

5. पूर्वी ग्रीनलैण्ड धारा-यह एक ठण्डी जलधारा है जो ग्रीनलैण्ड के उत्तरी-पूर्वी तट पर ध्रुवीय पवनों के प्रवाह से उत्तर-पूर्व दिशा की ओर बहती है। उत्तरी अटलाण्टिक ड्रिफ्ट इसमें आकर मिल
जाती है।

6. सारगैसो सागर-उत्तरी अटलाण्टिक महासागर के मध्य में जलधाराओं का एक गोलाकार क्रम बन जाता है, जिसके मध्य में शान्त जल पाया जाता है। शान्त जल का यह क्षेत्र सारगैसो नाम से पुकारा जाता है। इस भाग में सारगैसो नामक घास एकत्र हो जाने के फलस्वरूप भी इसे सारगैसो का नाम दिया गया है। इस सागरीय क्षेत्र का विस्तार 11,000 वर्ग किमी है। इस प्रकार का विचित्र सागर विश्व में अन्यत्र कहीं नहीं पाया जाता।

(ब) दक्षिणी अन्ध ( अटलाण्टिक) महासागर की जलधाराएँ
दक्षिणी अन्ध (अटलाण्टिक) महासागर की प्रमुख जलधाराएँ निम्नलिखित हैं

1. दक्षिणी विषुवतरेखीय धारा-उत्तरी विषुवत्रेखीय जलधारा के समानान्तर 0° दक्षिण से 20° दक्षिणी अक्षांशों के मध्य दक्षिणी-पूर्वी पवनों के कारण जो धारा जन्म लेती है, उसे दक्षिणी विषुवत्रेखीय धारा कहा जाता है। यह धारा विषुवत् रेखा के सहारे पूर्व से पश्चिम की ओर बहती है। दक्षिणी अमेरिका के सैनॉक अन्तरीप के निकट इस रेखा के दो भाग हो जाते हैं। इनमें से एक शाखा उत्तरी विषुवत्रेखीय धारा में मिल जाती है, जबकि दूसरी शाखा दक्षिण की ओर ब्राजील तट के सहारे बह निकलती है।

2. विपरीत विषुवत्रखीय धारा-उत्तरी एवं दक्षिणी विषुवत्रेखीय धाराओं के बीच विपरीत दिशा में जो धारा प्रवाहित होती है, उसे विपरीत विषुवत्रेखीय धारा कहते हैं।

3. ब्राजील की धारा-यह एक गर्म जलधारा है जो सैनरॉक अन्तरीप के निकट दक्षिण में ब्राजील तट के सहारे 40°दक्षिणी अक्षांशों तक प्रवाहित होती है। आगे चलकर यह पछुवा हवाओं के प्रभाव से पूर्व की ओर को मुड़ जाती है। यहाँ यह फाकलैण्ड की ठण्डी जलधारा से मिलकर कोहरा उत्पन्न करती है।

4. फाकलैण्ड धारा-फाकलैण्ड के उत्तर में बहने वाली ठण्डी धारा फाकलैण्ड धारा कहलाती है। यह धारा अपने साथ ध्रुवीय क्षेत्र से हिमखण्ड एवं हिमशिलाएँ बहाकर लाती है। बाद में यह ब्राजील धारा में मिलकर कोहरा उत्पन्न करती है।

5. दक्षिणी अटलाण्टिक ड्रिफ्ट-यह एक शीतल जलधारा है जो पछुवा पवनों से प्रभावित होकर दक्षिणी अफ्रीका के पश्चिमी तट की ओर अग्रसर होती है।

6. अण्टार्कटिक ड्रिफ्ट या पश्चिमी पवन प्रवाह-पृथ्वी की घूर्णन गति के कारण पछुवा पवनों के | प्रभाव से जो ठण्डी धारा जन्म लेती है उसे अण्टार्कटिक ड्रिफ्ट कहते हैं। यह शीतल जलधारा सम्पूर्ण भूमण्डल का परिभ्रमण करती है।

7. बेंग्युला धारा-दक्षिणी अटलाण्टिक ड्रिफ्ट पूर्व की ओर बहकर अफ्रीका के दक्षिणी- पश्चिमी तट से टकराकर उत्तर की ओर तट के सहारे बहकर बेंग्युला धारा बन जाती है, उत्तर में गिनी की खाड़ी से बहती हुई यह धारी दक्षिणी विषुवत्रेखीय धारा से मिलकर आगे बढ़ जाती है।

जलवायु पर प्रभाव-अन्ध महासागर की धाराएँ अपने निकटवर्ती क्षेत्रों की जलवायु पर विशेष प्रभाव डालती हैं। ये गल्फस्ट्रीम न्यूफाउण्डलैण्ड के निकट कोहरा उत्पन्न करती हैं। इसकी उष्णता के प्रभाव से ही उत्तरी-पश्चिमी यूरोप के तटवर्ती भागों में आकर्षक समशीतोष्ण जलवायु रहती है तथा यह तट वर्ष-भर जलयानों के आवागमन के लिए खुला रहता है, जबकि उन्हीं अक्षांशों में स्थित साइबेरिया का तट शीतलता के कारण जम जाता है। ब्राजील की गर्म जलधारा और फाकलैण्ड की शीतल जलधाराएँ। मिलकर कोहरा उत्पन्न कर देती हैं।

प्रश्न 3. ज्वार-भाटा किसे कहते हैं? ज्वार-भाटे के प्रकार बताइए।
या महासागरीय ज्वार-भाटा के कारणों की विवेचना कीजिए तथा तटीय क्षेत्रों के आर्थिक जीवन पर उनके प्रभावों का विश्लेषण कीजिए।
या टिप्पणी लिखिए-लघु ज्वार।।
या दीर्घ ज्वार आने का कारण बताइए। या ज्वार-भाटा क्या है तथा उसकी उत्पत्ति कैसे होती है? उसके प्रमुख प्रकारों का वर्णन कीजिए।
या ज्वार की उत्पत्ति की विवेचना कीजिए तथा मानव-जीवन पर इसके पड़ने वाले प्रभाव का वर्णन कीजिए।
उत्तर-ज्वार-भाटा (Tides)–महासागरीय जल की अस्थिर गतियों में ज्वार-भाटा का विशेष महत्त्व है। सागरीय जल के उभार या उठाव को ज्वार (tide) तथा जल के नीचे होने को भाटा (ebb) कहते हैं। इससे समुद्र-तल निरन्तर घटता-बढ़ता अथवा उद्वेलित होता रहता है और जल आगे-पीछे होता रहता है। इन सागरीय गतियों के कारण समुद्र-तल में सदैव परिवर्तन होता रहता है। इसके अतिरिक्त सागरीय तरंगें, लहरें, धाराएँ आदि भी ज्वार-भाटे में सहायक होती हैं। ज्वार-भाटा की मुख्य विशेषता है कि सूर्य एवं चन्द्रमा की आकर्षण शक्ति द्वारा सागर का जल ऊपर उठता है एवं नीचे गिरता है। सागरीय जल का तरंगों के रूप में आगे बढ़ना ज्वार कहलाता है। यदि इन तरंगों द्वारा उठे जलं का तल सर्वाधिक होता है तो उसे उच्च ज्वार कहते हैं। इसके उपरान्त वही तरंगें पीछे की ओर हटकर निम्न जल-तल का निर्माण करती हैं, इसे भाटा अथवा निम्न ज्वार कहते हैं। इस प्रकार सूर्य एवं चन्द्रमा की आकर्षण शक्तियाँ जल-तल को नित्य-प्रति क्रमश: ऊपर-नीचे करती रहती हैं, जिसे ज्वार-भाटा कहते हैं।

ज्वार-भाटा प्रतिदिन दो बार आता है, अर्थात् दो बार सागर का जल ऊपर उठता है एवं दो बार नीचे उतरता है। ज्वार-भाटे के समय नदियों के जल-तल में भी परिवर्तन आता रहता है। जब सागर में ज्वार का समय होता है तो नदियों की जल-धारा का तल ऊपर उठ जाता है। इसके विपरीत भाटा के समय नदियों की धारा का जल-तल नीचे की ओर होकर बड़ी तीव्र गति से बहने लगता है। समुद्रों एवं नदियों के जल-तल के ऊपर चढ़ने एवं नीचे गिरने के मध्य समय में बहुत कम अन्तर होता है। अतः सागरीय ज्वार-भाटा तथा इससे सम्बन्धित सभी क्रियाओं का सम्बन्ध सूर्य तथा चन्द्रमा की पारस्परिक आकर्षण शक्ति ही है।

ज्वार-भाटे की उत्पत्ति के कारण
हम जानते हैं कि सौर-परिवार के सदस्य परस्पर आकर्षण में बँधे हुए हैं। पृथ्वीतल पर सूर्य एवं चन्द्रमा दोनों की ही आकर्षण शक्ति का प्रभाव पड़ता है। चन्द्रमा की अपेक्षा सूर्य पृथ्वी से बहुत दूर है। अत: पृथ्वीतल पर उसकी आकर्षण शक्ति चन्द्रमा की आकर्षण शक्ति की लगभग आधी (5/11) है। अतः स्पष्ट है कि ज्वार-भाटे की उत्पत्ति में सूर्य की अपेक्षा चन्द्रमा की आकर्षण शक्ति का विशेष प्रभाव पड़ता है। परन्तु दोनों का संयुक्त प्रभाव ज्वार को विशालता प्रदान करता है। ज्वार-भाटा उत्पन्न होने की प्रक्रिया को समझने के लिए हमें निम्नलिखित तथ्यों की ओर ध्यान देना चाहिए

  1. ज्वार-भाटा उत्पन्न करने वाले दो बल गुरुत्वाकर्षण बल और अपकेन्द्र बल हैं।
  2. गुरुत्वाकर्षण बल चन्द्रमा, सूर्य और पृथ्वी की पारस्परिक क्रिया है। चन्द्रमा और सूर्य का गुरुत्वाकर्षण पृथ्वी की निकट दिशा में सबसे अधिक, पृथ्वी के केन्द्र पर उससे कम और पृथ्वी के । दूसरी ओर अर्थात् अधिकतम दूरी पर सबसे कम होता है।
  3. अपकेन्द्र बल पृथ्वी के घूर्णन से उत्पन्न होता है जो पृथ्वी के सभी भागों में समान होता है और | गुरुत्वाकर्षण बल की विपरीत दिशा में कार्य करता है। यह बल पृथ्वी के केन्द्र पर गुरुत्वाकर्षण बल के बराबर ही होता है।
  4. पृथ्वी की निकट दिशा पर गुरुत्वाकर्षण बल की अपकेन्द्र बल से अधिकता के कारण ज्वार उत्पन्न | होता है तथा पृथ्वी के दूसरी ओर उसी समय ज्वार की उत्पत्ति अपकेन्द्र बल की गुरुत्वाकर्षण बल से अधिकता के कारण होती है।
  5. इस प्रकार पृथ्वी के प्रत्येक भाग में प्रतिदिन दो बार ज्वार उठते हैं। जिन दो स्थानों से जल क्षैतिज गति करता हुआ ज्वार के स्थानों को चला जाता है उन स्थानों पर भाटे आते हैं। अत: प्रत्येक स्थान पर प्रतिदिन दो ज्वार उठते हैं और दो भाटे आते हैं।

ज्वार-भाटा के प्रकार

चन्द्रमा तथा पृथ्वी की गतियों के कारण सागरों तथा महासागरों में ज्वार-भाटा आता है। इन पर महासागरों के विशाल आकार का भी प्रभाव पड़ता है। विषुवत् रेखा तथा उसके आस-पास के सागरीय क्षेत्रों में ज्वार-भाटा का प्रभाव अधिक रहता है। यहाँ दो बार उच्च ज्वार तथा दो बार निम्न ज्वार का क्रम जारी रहता है, जब कि ध्रुवों के समीपवर्ती भागों में एक बार उच्च ज्वार तथा एक बार निम्न ज्वार ही उठता है। अध्ययन की सुविधा हेतु ज्वार-भाटी को निम्नलिखित भागों में बाँटा जा सकता है

1. वृहत् अथवा दीर्घ ज्वार- ज्वार-भाटा की उत्पत्ति में चन्द्रमा की विशेष भूमिका रहती है, परन्तु सूर्य का भी विशेष प्रभाव अंकित किया जाता है। सूर्य, पृथ्वी एवं चन्द्रमा एक-दूसरे को अपनी ओर आकर्षित करते हैं। इसीलिए चन्द्रमा के आकर्षण के अतिरिक्त सूर्य भी सम्पूर्ण महासागरीय जल को अपनी ओर आकर्षित करता है। सूर्य द्वारा पृथ्वी के आकर्षण का प्रभाव इसलिए भी कम दिखाई पड़ता है, क्योंकि सूर्य चन्द्रमा की अपेक्षा पृथ्वी से अधिक दूर स्थित है। सूर्य की आकर्षण शक्ति को प्रभाव उस समय अधिक दिखाई पड़ता है जब उसके सहयोग से चन्द्रमा महासागरों में उच्च ज्वार उत्पन्न करता है।
UP Board Solutions for Class 11 Geography Fundamentals of Physical Geography Chapter 14 Movements of Ocean Water (महासागरीय जल संचलन) img 5

उच्च ज्वार का निर्माण उस समय होता है जब सूर्य, पृथ्वी एवं चन्द्रमा तीनों एक सीध में स्थित होते हैं। यह स्थिति पूर्णिमा एवं अमावस्या के दिन होती है। पूर्णिमा तथा अमावस्या के दिन वृहत् ज्वार की ऊँचाई अन्य दिनों की अपेक्षा 20 प्रतिशत अधिक होती है। इस प्रकार की दशा माह में दो बार उत्पन्न होती है। इसे ही वृहत् अथवा दीर्घ-ज्वार कहते हैं।

2. लघु ज्वार-यह ज्वार पूर्णिमा तथा अमावस्या के अतिरिक्त तिथियों में आता है। सूर्य से पृथ्वी और चन्द्रमा की स्थिति सदैव परिवर्तित होती रहती है। कृष्णपक्ष एवं शुक्लपक्ष की सप्तमी और अष्टमी तिथियों में सूर्य और चन्द्रमा पृथ्वी के साथ समकोणीय स्थिति उत्पन्न करते हैं। इस समकोणीय स्थिति के द्वारा सूर्य और चन्द्रमा पृथ्वी के महासागरीय जल को अनेक दशाओं में आकर्षित करते हैं। इसीलिए महासागरों में ज्वार की ऊँचाई अन्य तिथियों की अपेक्षा कम रह जाती है, जिसे लघु ज्वार कहते हैं। औसत रूप से ज्वार अपनी ऊँचाई में इन्हीं तिथियों में 20 प्रतिशत कम रहता है।
UP Board Solutions for Class 11 Geography Fundamentals of Physical Geography Chapter 14 Movements of Ocean Water (महासागरीय जल संचलन) img 6

3. उच्च तथा निम्न ज्वार-चन्द्रमा अपने अण्डाकार पथ पर पृथ्वी की परिक्रमा करता है। इस अण्डाकार पथ पर परिभ्रमण करता हुआ चन्द्रमा पृथ्वी से निकटतम दूरी अर्थात् 3,56,000 किमी पर आ जाता है तो इस स्थिति को निम्न ज्वार कहते हैं। इस समय चन्द्रमा की ज्वार उत्पादक शक्ति औसत रूप में सर्वाधिक होती है जो उच्च ज्वार उत्पन्न करती है। इसके विपरीत, जब चन्द्रमा पृथ्वी से अधिकतम दूरी अर्थात् 4,07,000 किमी पर स्थित होता है। तो वह उच्च ज्वार कहलाता है। चन्द्रमा की अधिकतम दूरी होने पर इसका ज्वार उत्पादक बल निम्नतम रह जाता है। इससे ज्वार की स्थिति भी न्यूनतम रह जाती है। इस ज्वार की ऊँचाई औसत ज्वार की अपेक्षा 20 प्रतिशत कम रह जाती है।

4. अयनवृत्तीय या भूमध्यरेखीय ज्वार-जिस प्रकार सूर्य अपनी परिभ्रमण गति में उत्तरायण एवं दक्षिणायण होता है, उसी प्रकार चन्द्रमा भी अपनी इस गति के कारण उत्तरायण एवं दक्षिणायण होता रहता है, परन्तु चन्द्रमा की यह स्थिति 294 दिन में पूर्ण होती है। जब चन्द्रमा का झुकाव उत्तरायण में होता है तो चन्द्र-किरणें कर्क रेखा के समीप सीधी तथा लम्बवत् पड़ती हैं जो उच्च ज्वार में वृद्धि कर देती हैं। यह ज्वार पश्चिम दिशा की ओर अधिक बढ़ता है। यही स्थिति चन्द्र-किरणों के दक्षिणायण होने पर मकर रेखा पर होती है। इस प्रकार कर्क एवं मकर रेखाओं पर उत्पन्न होने वाले ज्वार को अयनवृत्तीय ज्वार कहते हैं। चन्द्रमा की किरणें विषुवत् रेखा पर प्रत्येक माह लम्बवत् होने के कारण ज्वार की दैनिक असमानता समाप्त हो जाती है। इसी कारण दो उच्च ज्वार तथा दो निम्न ज्वार की ऊँचाइयाँ बराबर हो जाती हैं, जिसे भूमध्यरेखीय ज्वार कहते हैं।

5. दैनिक ज्वार-भाटा-दैनिक ज्वार-भाटा प्रत्येक दिन एक ही स्थान पर एक ज्वार एवं एक भाटा के रूप में आता है। प्रत्येक ज्वार में सदैव 52 मिनट का अन्तर रहता है। दैनिक ज्वार-भाटा को सूर्य, पृथ्वी तथा चन्द्रमा की स्थिर गतियाँ समयानुसार प्रभावित करती रहती हैं। दैनिक ज्वार-भाटा 24 घण्टे बाद आता है।

6. अर्द्ध-दैनिक ज्वार-भाटा-एक दिन में एक ही स्थान पर दो बार आने वाले ज्वार-भाटा को अर्द्ध-दैनिक ज्वार-भाटा कहते हैं। यह ज्वार प्रत्येक दिन 24 घण्टे 26 मिनट बाद आता है। इस ज्वार-भाटा में जल का उठना एवं गिरना क्रमशः समान रहता है।

7. मिश्रित ज्वार-भाटा-इस ज्वार में अर्द्ध-दैनिक ज्वार-भाटा की समान ऊँचाइयों में अन्तर आ जाता है। कुछ विद्वानों ने दैनिक तथा अर्द्ध-दैनिक ज्वार-भाटा की ऊँचाइयों में अन्तर को मिश्रित ज्वार-भाटा कहा है।

ज्वार-भाटे के प्रभाव

ज्वार-भाटे का प्रभाव नौका-परिवहन पर अत्यधिकं पड़ता है। ज्वार द्वारा कुछ नदियाँ बड़े जलयानों के चलाने योग्य बन जाती हैं। हुगली तथा टेम्स नदियाँ ज्वारीय धाराओं के कारण ही नाव्य हो सकी हैं तथा कोलकाता व लन्दन महत्त्वपूर्ण पत्तन बन सके हैं। ज्वारीय ऊर्जा को भी आज महत्त्वपूर्ण समझा जाने लगा है। ज्वार के द्वारा समुद्रतटीय नगरों के कूड़े व गन्दगी के ढेर बहकर समुद्र में चले जाते हैं।

We hope the UP Board Solutions for Class 11 Geography: Fundamentals of Physical Geography Chapter 14 Movements of Ocean Water (महासागरीय जल संचलन) help you. If you have any query regarding UP Board Solutions for Class 11 Geography: Fundamentals of Physical Geography Chapter 14 Movements of Ocean Water (महासागरीय जल संचलन), drop a comment below and we will get back to you at the earliest.

UP Board Solutions for Class 11 Geography: Fundamentals of Physical Geography Chapter 9 Solar Radiation, Heat Balance and Temperature

UP Board Solutions for Class 11 Geography: Fundamentals of Physical Geography Chapter 9 Solar Radiation, Heat Balance and Temperature (सौर विकिरण, ऊष्मा संतुलन एवं तापमान)

These Solutions are part of UP Board Solutions for Class 11 Geography. Here we have given UP Board Solutions for Class 11 Geography: Fundamentals of Physical Geography Chapter 9 Solar Radiation, Heat Balance and Temperature (सौर विकिरण, ऊष्मा संतुलन एवं तापमान)

पाठ्य-पुस्तक के प्रश्नोत्तर

1. बहुवैकल्पिक प्रश्न
प्रश्न (i) निम्न में से किस अक्षांश पर 21 जून की दोपहर सूर्य की किरणें सीधी पड़ती हैं?
(क) विषुवत् वृत्त पर
(ख) 23.5° उ०
(ग) 66.5° द०
(घ)66.5° उ०
उत्तर-(ख) 23.5° उ०।

प्रश्न (ii) निम्न में से किस शहर में दिन ज्यादा लम्बा होता है?
(क) तिरुवनन्तपुरम
(ख) हैदराबाद
(ग) चण्डीगढ़
(घ) नागपुर
उत्तर-(ग) चण्डीगढ़।

प्रश्न (iii) निम्नलिखित में से किस प्रक्रिया द्वारा वायुमण्डल मुख्यतः गर्म होता है?
(क) लघु तरंगदैर्घ्य वाले सौर विकिरण से
(ख) लम्बी तरंगदैर्घ्य वाले स्थलीय विकिरण से
(ग) परावर्तित सौर विकिरण से।
(घ) प्रकीर्णित सौर विकिरण से।
उत्तर-(ख) लम्बी तरंगदैर्घ्य वाले स्थलीय विकिरण से।।

प्रश्न (iv) निम्न पदों को उसके उचित विवरण के साथ मिलाएँ
1. सूर्यातप (अ) सबसे कोष्ण और सबसे शीत महीनों के मध्य तापमान का अन्तर
2. एल्बिडो (ब) समान तापमान वाले स्थानों को जोड़ने वाली रेखा।
3. समताप रेखा (स) आने वाला सौर विकिरण ।
4. वार्षिक तापान्तर (द) किसी वस्तु के द्वारा परावर्तित दृश्य प्रकाश का प्रतिशत।
उत्तर-(1) (स), 2. (द) 3. (ब), 4. (अ)।।

प्रश्न (v) पृथ्वी के विषुवत वृत्तीय क्षेत्रों की अपेक्षा उत्तरी गोलार्द्ध के उपोष्ण कटिबन्धीय क्षेत्रों का तापमान अधिकतम होता है, इसका मुख्य कारण है
(क) विषुवतीय क्षेत्रों की अपेक्षा उपोष्ण कटिबन्धीय क्षेत्रों में कम बादल होते हैं।
(ख) उपोष्ण कटिबन्धीय क्षेत्रों में गर्मी के दिनों की लम्बाई विषुवत्तीय क्षेत्रों से ज्यादा होती है।
(ग) उपोष्ण कटिबन्धीय क्षेत्रों में ग्रीनहाउस प्रभाव’ विषुवतीय क्षेत्रों की अपेक्षा ज्यादा होता है।
(घ) उपोष्ण कटिबन्धीय क्षेत्र विषुवतीय क्षेत्रों की अपेक्षा महासागरीय क्षेत्रों के ज्यादा करीब है।
उत्तर—(क) विषुवतीय क्षेत्रों की अपेक्षा उपोष्ण कटिबन्धीय क्षेत्रों में कम बादल होते हैं।

2. निम्नलिखित प्रश्नों के उत्तर लगभग 30 शब्दों में दीजिए
प्रश्न (i) पृथ्वी पर तापमान का असमान वितरण किस प्रकार जलवायु और मौसम को प्रभावित करता है?
उत्तर-पृथ्वी पर कुछ स्थान शीत जलवायु वाले हैं तथा कुछ स्थान उष्ण मरुस्थलीय हैं जबकि कुछ शीतोष्ण जलवायु के अन्तर्गत सम्मिलित हैं। वास्तव में इसका मुख्य कारण पृथ्वी पर तापमान का असमान वितरण है। इतना ही नहीं पृथ्वी के उच्च पर्वतीय भागों में हिम के रूप में वर्षा होती है और मैदानी भागों में जल के रूप में। इसका कारण भी यही है क्योंकि उच्च पर्वतीय भागों पर तापमान कम और मैदानी भागों में तापमान अधिक रहता है। अत: पृथ्वी पर तापमान का असमान वितरण जलवायु और मौसम को सबसे अधिक प्रभावित करता है। जहाँ तापमान अधिक पाया जाता है वहाँ उष्ण जलवायु मिलती है किन्तु जहाँ तापमान निम्न रहता है वहाँ शीत जलवायु रहती है। वस्तुत: तापमान जलवायु व मौसम का निर्धारक तत्त्व है, इसीलिए पृथ्वी के सभी स्थान तापमान भिन्नता के कारण असमान जलवायु प्रदेशों के रूप में पहचाने जाते हैं।

प्रश्न (ii) वे कौन-से कारक हैं जो पृथ्वी पर तापमान के वितरण को प्रभावित करते हैं?
उत्तर-पृथ्वी पर तापमान वितरण में पर्याप्त असमानताएँ मिलती हैं। इस असमानता के प्रमुख कारक निम्नलिखित हैं। 1. अक्षांश, 2. समुद्रतल से ऊँचाई, 3. समुद्रतल से दूरी, 4. पवनों की दिशा, 5. स्थानीय कारक।

प्रश्न (iii) भारत में मई में तापमाने सर्वाधिक होता है, लेकिन उत्तर अयनांत के बाद तापमान अधिकतम नहीं होता। क्यों?
उत्तर-भारत में मुख्य रूप से मार्च के बाद गर्मी ऋतु प्रारम्भ हो जाती है। मई एवं जून देश में प्रचण्ड गर्मी के महीने होते हैं, इनमें भी मई मौसम में अधिक तापमान अंकित किया जाता है। किन्तु उत्तर अयनांत के बाद तापमान में गिरावट आरम्भ हो जाती है, क्योंकि दिन की अवधि में परिवर्तन आ जाता है।

प्रश्न (iv) साइबेरिया के मैदान में वार्षिक तापान्तर सर्वाधिक होता है। क्यों?
उत्तर-साइबेरिया के मैदान में वार्षिक तापान्तर सर्वाधिक होता है। इसका मुख्य कारण यहाँ जलवायु पर समुद्री धाराओं का विशेष प्रभाव है। इस भाग में कोष्ण महासागरीय धारा गल्फस्ट्रीम तथा उत्तरी अटलांटिक महासागरीय ड्रिफ्ट की उपस्थिति से उत्तरी अन्ध महासागर अधिक गर्म हो जाता है जो यूरेशिया के मैदानी भागों के तापमान में परिवर्तन कर देता है। इसलिए साइबेरिया के मैदानी भाग जो अन्ध महासागर के अधिक निकट हैं वहाँ वार्षिक तापान्तर अधिक पाया जाता है।

3. निम्नलिखित प्रश्नों के उत्तर लगभग 150 शब्दों में दीजिए
प्रश्न (1) अक्षांश और पृथ्वी के अक्ष का झुकाव किस प्रकार पृथ्वी की सतह पर प्राप्त होने वाली विकिरण की मात्रा को प्रभावित करते हैं?
उत्तर-पृथ्वी पर सूर्यातप की प्राप्ति अक्षांश और पृथ्वी के अक्ष के झुकाव द्वारा निर्धारित होती है। पृथ्वी का अक्ष सूर्य के चारों ओर परिक्रमण की समतल कक्षा से [latex s=2]66\frac { { 1 }^{ o } }{ 2 } [/latex] का कोण बनाता है, इसके कारण सभी अक्षांशों पर सूर्य की किरणों का नति कोण समान नहीं होता है। यह कोण उच्च अक्षांशों अर्थात् ध्रुवों पर कम होता है तथा निम्न अक्षांशों अर्थात् भूमध्य रेखा पर अधिक होता है। यही कारण है कि भूमध्य रेखा से ध्रुवों की ओर उत्तरी एवं दक्षिणी अक्षांशों पर तापमान कम होता जाता है। चित्र 9.1 को देखने से स्पष्ट होता है कि ध्रुवों पर सूर्य की किरणें तिरछी पड़ने से वह अधिक क्षेत्र परे फैलती हैं; अतः इन किरणों को पृथ्वी का अधिक स्थान घेरना पड़ता है, इसलिए कम ताप की प्राप्ति होती है जबकि भूमध्यरेखा पर सूर्य की किरणें सीधी पड़ती हैं। इन किरणों का घनत्व कम क्षेत्र को अधिक तापमान प्रदान करता है। अतः पृथ्वी का गोलाकार स्वरूप, उसका अपने अक्ष पर झुकाव और अक्षांश ऐसे तथ्य हैं जो पृथ्वी की सतह पर प्राप्त होने वाली विकिरणों की मात्रा से प्रभावित होते हैं और पृथ्वी के तापमान वितरण में असमानता उत्पन्न करते हैं।

UP Board Solutions for Class 11 Geography Fundamentals of Physical Geography Chapter 9 Solar Radiation, Heat Balance and Temperature (सौर विimg 1

प्रश्न (ii) पृथ्वी और वायुमण्डल किस प्रकार ताप को सन्तुलित करते हैं? इसकी व्याख्या करें।
उत्तर–वास्तव में पृथ्वी तापमान (ऊष्मा) का न तो संचय करती है न ही ह्रास, बल्कि यह अपने तापमान को स्थिर रखती है। ऐसा तभी सम्भव है, जब सूर्य विकिरण द्वारा सूर्यातप के रूप में प्राप्त उष्मा एवं पार्थिव विकिरण द्वारा अन्तरिक्ष में संचलित ताप बराबर हो। चित्र 9.2 से स्पष्ट है कि यदि यह मान लें कि वायुमण्डल की ऊपरी सतह पर प्राप्त सूर्यातप 100 प्रतिशत है तो 100 इकाइयों में से 35 इकाइयाँ पृथ्वी के धरातल पर पहुँचने से पहले ही अन्तरिक्ष में परावर्तित हो जाती हैं। शेष 65 इकाइयाँ अवशोषित होती हैं। इनमें 14 वायुमण्डल में तथा 51 पृथ्वी के धरातल को प्राप्त होती हैं। पृथ्वी द्वारा अवशोषित ये 51 इकाइयाँ पुनः पार्थिव विकिरण के रूप में लौटा दी जाती हैं। इनमें से 17 इकाइयाँ तो सीधे अन्तरिक्ष में चली जाती हैं और 34 इकाइयाँ वायुमण्डल द्वारा अवशोषित होती हैं। (देखिए चित्र 9.2 अ) (6 इकाइयाँ स्वयं वायुमण्डल द्वारा, 9 इकाइयाँ संवहन द्वारा और 19 इकाइयाँ संघनन की गुप्त उष्मा के रूप में चित्र 9.2 ब) वायुमण्डल द्वारा 48 इकाइयों का अवशोषण होता है। इनमें 14 इकाइयाँ सूर्यापत की ओर 34 इकाइयाँ पार्थिव विकिरण की होती हैं। वायुमण्डल विकिरण द्वारा इनको भी अन्तरिक्ष में वापस लौटा देता है। अतः पृथ्वी के धरातल तथा वायुमण्डल से अन्तरिक्ष में वापस लौटने वाली विकिरण की इकाइयाँ क्रमश: 17 और 48 हैं, जिनका योग 65 होता है (चित्र 9.2 ब)। वापस लौटने वाली ये इकाइयाँ उन 65 इकाइयों को सन्तुलन कर देती हैं जो सूर्य से प्राप्त होती हैं। यही पृथ्वी का 100 इकाइयों का ऊष्मा बजट है।
UP Board Solutions for Class 11 Geography Fundamentals of Physical Geography Chapter 9 Solar Radiation, Heat Balance and Temperature (सौर विimg 2
यही कारण है कि पृथ्वी पर ऊष्मा के इतने बड़े स्थानान्तरण के होते हुए भी उष्मा सन्तुलन बना रहता है। इसीलिए पृथ्वी न तो बहुत गर्म होती है न ही अधिक ठण्डी, बल्कि मानव एवं जीव-जंतुओं के अनुकूल तापमान रखती है।

प्रश्न (iii) जनवरी में पृथ्वी के उत्तरी और दक्षिणी गोलार्द्ध के बीच तापमान के विश्वव्यापी वितरण की तुलना करें।
उत्तर-जनवरी में पृथ्वी के उत्तरी और दक्षिणी गोलार्द्ध के बीच तापमान के विश्वव्यापी वितरण की तुलना जनवरी के भू-पृष्ठीय वायु तापक्रम समदाब रेखा मानचित्र 9.3 द्वारा स्पष्टत: समझी जा सकती है। इस समय उत्तरी गोलार्द्ध में समदाब रेखाएं महासागरों की ओर तथा दक्षिणी गोलार्द्ध में महाद्वीपों की ओर विचलित हो जाती हैं। मानचित्र से स्पष्ट है कि भूमध्य रेखा से उत्तरी गोलार्द्ध की ओर तापमान तेजी से घटते हुए उत्तरी अटलाण्टिक महासागर में शून्य तक तथा इसके बाद उत्तर की ओर -25° तक पहुँच जाता है। इसका मुख्य कारण कोष्ण महासागरीय धारा गल्फस्ट्रीम तथा उत्तरी अटलाण्टिक महासागरीय ड्रिफ्ट की उपस्थिति है, इसके कारण उत्तरी अन्ध महासागर गर्म हो जाता है। अत: यहाँ तापमान उत्तरी ध्रुव की अपेक्षा अधिक ही रहता है, जबकि इसके उत्तर की ओर जाने पर तापमान -25° से भी अधिक गिर जाता है।
UP Board Solutions for Class 11 Geography Fundamentals of Physical Geography Chapter 9 Solar Radiation, Heat Balance and Temperature (सौर विimg 3
दक्षिणी गोलार्द्ध इस समय उत्तरी गोलार्द्ध की अपेक्षा अधिक गर्म रहता है। मानचित्र 9.3 पर अंकित समदाब रेखाओं से स्पष्ट है कि यहाँ दक्षिणी अमेरिका, अफ्रीका तथा ऑस्ट्रेलिया महाद्वीप के मध्य भाग पर 20° एवं 30° समदाब रेखाएँ प्रवाहित होती हैं। केवल दक्षिण की ओर जाने पर इस गोलार्द्ध में तापमान गिरता है। फिर भी यहाँ दक्षिणी भाग का तापमान 10°C के आस-पास ही रहता है। जबकि उत्तरी गोलार्द्ध में तापमान हिमांक बिन्दु से भी नीचे पहुँच जाता है।

परीक्षोपयोगी प्रश्नोत्तर

बहुविकल्पीय प्रश्न
प्रश्न 1. विषुवत रेखा पर सूर्य की किरणें पड़ती हैं|
(क) तिरछी
(ख) लम्बवत्
(ग) समानान्तर
(घ) इनमें से किसी प्रकार की नहीं
उत्तर-(ख) लम्बवत्।

प्रश्न 2. ध्रुवीय प्रदेशों में तापमान बहुत ही कम होता है, इसका कारण है
(क) सूर्य की किरणों का लम्बवत् पड़ना ।
(ख) सूर्योदय न होना
(ग) सूर्य की किरणों का तिरछा पड़ना
(घ) इनमें से कोई नहीं
उत्तर-(ग) सूर्य की किरणों का तिरछा पड़ना।

प्रश्न 3. पृथ्वी की सूर्य से लगभग दूरी है
(क) 24.96 करोड़ किमी
(ख) 19.46 करोड़ किमी
(ग) 14.69 करोड़ किमी
(घ) 14.96 करोड़ किमी
उत्तर-(घ) 14.96 करोड़ किमी।

प्रश्न 4. तापमान को नापते हैं
(क) फारेनहाइट से
(ख) सेण्टीग्रेड से
(ग) रियूमर से
(घ) इन सभी से
उत्तर-(घ) इन सभी से।।

प्रश्न 5. उत्तरी तथा दक्षिणी गोलार्द्ध में तापमान में अन्तर पाया जाता है, क्योंकि
(क) वर्षा में अन्तर है।
(ख) जल तथा स्थल के वितरण में असमानता है।
(ग), वाष्पीकरण की दर में अन्तर
(घ) सूर्यातप का वितरण असमान है।
उत्तर-(घ) सूर्यातप का वितरण असमान है।

अतिलघु उत्तरीय प्रश्न

प्रश्न 1. सूर्यामिताप या सूर्यातप से क्या तात्पर्य है?
उत्तर-पृथ्वी को प्राप्त होने वाले सूर्य के ताप को सूर्यातप कहते हैं।

प्रश्न 2. पृथ्वी तक सूर्य का प्रकाश पहुँचने में कितना समय लगता है?
उत्तर-सूर्य से पृथ्वीतल तक प्रकाश पहुँचने में 8 मिनट 22 सेकण्ड का समय लगता है।

प्रश्न 3. सूर्यातप को प्रभावित करने वाले दो प्रमुख कारकों का उल्लेख कीजिए।
उत्तर-1. सूर्य की किरणों का सापेक्ष तिरछापन तथा 2. सूर्य में प्रकाश की अवधि।

प्रश्न 4. धरातल पर तापमानों के क्षैतिज वितरण को प्रभावित करने वाले दो कारकों का उल्लेख कीजिए।
उत्तर-

  • अक्षांशीय स्थिति तथा
  • समुद्रतल से ऊँचाई।

प्रश्न 5. भूमण्डल पर तापीय कटिबन्धों का उल्लेख कीजिए।
उत्तर-

  • उष्ण कटिबन्ध,
  • शीतोष्ण कटिबन्ध एवं
  • शीत कटिबन्ध।

प्रश्न 6. तापमान की सामान्य ह्रास दर से आप क्या समझते हैं?
उत्तर-औसत रूप से वायुमण्डल में प्रति 165 मीटर की ऊँचाई पर 1°C तापमान कम हो जाता है, जिसे तापमान की सामान्य ह्रास दर कहते हैं।

प्रश्न 7. तापमान विलोम क्या है?
या तापमान की विलोमता से आप क्या समझते हैं?
उत्तर-ऐसी अवस्था जिसमें धरातल के निकट कम तापमान तथा ऊपर अधिक तापमान हो, उसे ताप की विलोमता कहते हैं।

प्रश्न 8. ताप की विलोमता के लिए दो आवश्यक दशाओं का उल्लेख कीजिए।
उत्तर-

  • लम्बी रातें तथा
  • मेघरहित आकाश।

प्रश्न 9. तापमान विलोमता के लिए प्रमुख अनुकूल दशा क्या होनी चाहिए?
उत्तर-तापमान विलोमता मुख्यतः अन्तरापर्वतीय घाटियों में उत्पन्न होती है, क्योंकि यहाँ जाड़े की ठण्डी । रातों में आकाश साफ तथा वायु शुष्क एवं शान्त होती है।

प्रश्न 10. समताप रेखाओं से क्या अभिप्राय है? ।
उत्तर-समताप रेखाएँ वे कल्पित रेखाएँ हैं जो मानचित्र पर समान तापमान वाले स्थानों को मिलाकर खींची जाती हैं।

प्रश्न 11. पार्थिव विकिरण का क्या अर्थ है?
उत्तर-सूर्यातप का पृथ्वी की सतह से होने वाले विकिरण को पार्थिव विकिरण कहते हैं। यह विकिरण लम्बी तरंगों के रूप में होता है। इसी विकिरण प्रक्रिया द्वारा वायुमण्डल नीचे से ऊपर की ओर गर्म होता है।

प्रश्न 12. सूर्य की तिरछी किरणों का पृथ्वी पर क्या परिणाम होता है?
उत्तर-पृथ्वी पर सूर्य की तिरछी किरणें शीत और शीतोष्ण कटिबन्ध पर पड़ती हैं। इन किरणों के कारण दोनों कटिबन्धों पर ग्रीष्म एवं शीत ऋतु के दौरान ताप की अधिक भिन्नताएँ पाई जाती हैं अर्थात् कम ताप प्राप्त होता है।

प्रश्न 13. पृथ्वी पर तापमान वितरण में भूमि के ढाल का प्रभाव बताइए।
उत्तर-स्थलाकृतियों के जो ढाल सूर्य की किरणों के सम्मुख पड़ते हैं, उन पर ताप अपेक्षाकृत अधिक पाया जाता है। इसी कारण उत्तरी गोलार्द्ध में पर्वतों के दक्षिणी ढालों पर, उत्तरी ढालों की अपेक्षा अधिक तापमान पाया जाता है।

प्रश्न 14, पृथ्वी के तापमान को प्रभावित करने वाली वायुदाब घटनाओं का उल्लेख कीजिए।
उत्तर-तूफान, आँधी, बादल, वर्षा, ओस, कुहरा, तुषार व हिमपात आदि सभी परोक्ष एवं अपरोक्ष रूप से तापमान को प्रभावित करते हैं। मेघों का आवरण सूर्य की किरणों को धरातल तक पहुँचने एवं पार्थिव ऊष्मा के पुनः विकिरण में अवरोध पैदा करता है जिससे तापमान प्रभावित होता है।

प्रश्न 15. तापान्तर क्या है?
उत्तर-अधिकतम तथा न्यूनतम तापमान के अन्तर को ‘तापान्तर’ कहते हैं। यह दो प्रकार का होता है—(1) दैनिक तापान्तर तथा (2) वार्षिक तापान्तर।

प्रश्न 16. पृथ्वी का एल्बिडो किसे कहते हैं?
उत्तर-पृथ्वी के धरातल पर पहुँचने से पहले ही विकिरण की परावर्तित मात्रा को पृथ्वी का एल्बिडो कहते हैं। यह परावर्तित मात्रा 49 इकाई के रूप में होती है।

प्रश्न 17, क्या कारण है कि तापमान उत्तुंगता बढ़ने के साथ घटता है?
उत्तर-उत्तुंगता या ऊँचाई (Altitude) बढ़ने के साथ तापमान वास्तव में घटता है। हम जानते हैं कि वायुमण्डल पार्थिव विकिरण द्वारा नीचे की परतों में पहले गर्म होता है। यही कारण है कि समुद्र तल के स्थानों पर तापमान अधिक तथा ऊँचे भाग में स्थित स्थानों पर तापमान कम होता है।

प्रश्न 18. समताप रेखाओं की दिशा अधिकतर पूर्व-पश्चिम क्यों रहती है?
उत्तर-जनवरी तथा जुलाई माह के समताप मानचित्रों के अध्ययन से ज्ञात होता है कि समताप रेखाओं की दिशा मुख्यतः पूर्व-पश्चिम होती है, क्योंकि समान अक्षांशों पर स्थित अनेक स्थानों पर तापमान समान पाया जाता है। अतः जब इन समान तापमान वाले स्थानों को मिलाकर रेखाएँ खींची जाती हैं तो उन रेखाओं की दिशा पूर्व-पश्चिम की ओर होती है।

प्रश्न 19. निम्नलिखित के लिए एक उपयुक्त शब्द चुनिए
(i) समान तापमान वाले स्थानों को जोड़ने वाली रेखा,
(ii) लघु तरंगों में सूर्य से आने वाला विकिरण,
(iii) ऊष्मा का आने वाला एवं जाने वाला तापान्तर,
(iv) सूर्य की किरणों का कोण जब वे पृथ्वी पर आती हैं।
उत्तर-(i) समताप रेखा, (i) सूर्यातप, (ii) ऊष्मा सन्तुलन, (iv) आपतन कोण।

लघु उत्तरीय प्रश्न

प्रश्न 1. सूर्यातप एवं भौमिक विकिरण में अन्तर स्पष्ट कीजिए।
उत्तर-सूर्यातप–सूर्य से पृथ्वी तक पहुँचने वाली विकिरण ऊर्जा को सूर्यातप कहते हैं। यह ऊर्जा लघु तरंगों के रूप में सूर्य से पृथ्वी पर पहुँचती है। यह पृथ्वी पर ऊर्जा का मुख्य स्रोत है। यहाँ होने वाली अधिकांश भौतिक एवं जैविक घटनाएँ इसी ऊर्जा से नियन्त्रित होती हैं।

भौमिक विकिरण-पृथ्वी द्वारा विकिरित ऊर्जा को भौमिक या पार्थिव विकिरण कहते हैं। वायुमण्डल भौमिक विकिरण द्वारा ही गर्म होता है। यह प्रक्रिया लम्बी तरंगों द्वारा पूरी होती है।

प्रश्न 2. अभिवहन तथा संवहन की तुलनात्मक व्याख्या कीजिए।
उत्तर-अभिवहन–अभिवहन ऊष्मा का क्षैतिज दिशा में स्थानान्तरण है। इस प्रक्रिया में जब ठण्डे प्रदेशों में गर्म वायुराशि जाती है तो उनको गर्म कर देती है। इससे ऊष्मा का संचार निम्न अक्षांशीय क्षेत्रों से उच्च अक्षांशीय क्षेत्रों में भी होता है। इसके अतिरिक्त वायु द्वारा संचालित समुद्री धाराएँ भी उष्ण कटिबन्धों से ध्रुवीय क्षेत्रों में ऊष्मा का संचार करती हैं।

संवहन-जब पृथ्वी को स्पर्श करके वायु के कणगर्म होते हैं तो वह हल्के होकर ऊपर उठते हैं और फिर ऊपर से ठण्डे होकर नीचे आते हैं। इस प्रक्रिया से संवहन धाराएँ उत्पन्न हो जाती हैं। इन धाराओं की प्रकृति गैसीय या तरल पदार्थों में जाने की होती है। इससे ऊष्मा का संचार होता है जो संवहन कहलाता है।

प्रश्न 3. यान्त्रिक प्रतिलोमन क्या है? इसकी उत्पत्ति के क्या कारण हैं?
उत्तर-यह प्रतिलोमन धरातल से ऊपर वायुमण्डल में होता है। इसकी उत्पत्ति मूलतः वायुमण्डल में वायु के ऊपर-नीचे (लम्बवत्) गतिशील होने से होती है। इसकी उत्पत्ति में निम्न दो प्रक्रियाएँ महत्त्वपूर्ण हैं—(अ) कभी-कभी नीचे की गर्म वायु तीव्र गति से ऊपर उठ जाती है तथा ऊपर की ठण्डी व भारी वायु नीचे धरातल की ओर आ जाती है और प्रतिलोमन की दशा पैदा करी देती है। (ब) मध्य अक्षांशों चक्रवातों के कारण भी यान्त्रिक प्रतिलोमन की स्थिति उत्पन्न हो जाती है।

प्रश्न 4. लम्बवत तापमान या तापमान विलोमता की क्या आर्थिक उपयोगिता है?
उत्तर-वस्तुतः तापमान का लम्बवत् वितरण एवं तापमान की विलोम दशाएँ वायुमण्डलीय देशाओं और आर्थिक दृष्टि से क्शेिष महत्त्व रखती हैं। मेघों का स्वरूप, वर्षा की मात्रा, वायुमण्डल की दृश्यता आदि पर तापमान विलोमता का स्पष्ट प्रभाव पड़ता है। पर्वतीय ढालों पर कृषि तथा मानव बसाव की आदर्श दशाएँ भी तापमान विलोमता के कारण ही उत्पन्न होती हैं। तापमान विलोमती के कारण पर्वतीय क्षेत्रों में एक ही ऋतु में विभिन्न प्रकार की कृषि उपजें उत्पन्न होती हैं। जो वर्तमान में आर्थिक दृष्टि से विशेष महत्त्वपूर्ण है। अतः लम्बवत् तापमान की असमानताएँ प्राकृतिक और आर्थिक दोनों दृष्टियों से उपयोगी हैं।

प्रश्न 5. सूर्यातप पर टिप्पणी लिखिए।
उत्तर-‘सूर्यातप’ अंग्रेजी भाषा के Insolation शब्द का हिन्दी रूपान्तरण है। Insolation से तात्पर्य In Coming Solar Radiation है। वास्तव में, सूर्य से निरन्तर तरंगों के रूप में ताप शक्ति प्रसारित होती रहती है। यही ताप शक्ति धरातल तक पहुँचकर उसे ऊष्मा प्रदान करती है। सूर्य से प्राप्त होने वाली यही ऊर्जा सूर्यातप कहलाती है। सूर्यातप विद्युत चुम्बकीय तरंगों द्वारा वायुमण्डल की 32,000 किमी मोटी परत को पार कर लघु तरंगों के रूप में धरातल पर आती हैं, जिसे सौर विकिरण (Solar Radiation) की प्रक्रिया कहते हैं। अत: वायुमण्डल अधिकतर सीधे सूर्य की किरणों से ऊष्मा प्राप्त नहीं कर पाता है। वह इन किरणों से केवल 19% ताप ही जल-वाष्प एवं धूलकणों के माध्यम से प्राप्त कर पाता है। वस्तुत: गर्म होती हुई पृथ्वी को स्पर्श करके ही वायुमण्डल गर्म हो जाता है, जिसे वायुमण्डल का परोक्ष रूप से गर्म होना कहते हैं।

प्रश्न 6. पृथ्वी पर तापमान वितरण को प्रदर्शित करने के लिए अक्षांश रेखाओं की भाँति समदाब रेखाओं का उपयोग क्यों किया जाता है?
उत्तर-वायुमण्डल एवं भूपटल पर ताप एवं ऊर्जा का प्रधान स्रोत सूर्य है। पृथ्वी पर तापमान की मात्रा सर्वत्र एक-समान नहीं पाई जाती है। भूमण्डल पर ताप का क्षैतिज वितरण प्रदर्शित करने के लिए विषुवत् रेखा को आधार माना जाता है। धरातल पर तापमान का क्षैतिज वितरण समताप रेखाओं (Isotherms) द्वारा प्रकट किया जाता है। समताप रेखाएँ वे काल्पनिक रेखाएँ होती हैं जो मानचित्रों में समान ताप वाले स्थानों को मिलाते हुए खींची जाती हैं। इन रेखाओं के सिरों पर तापमान सेल्सियस या फारेनहाइट में अंकित कर दिया जाता है। ये रेखाएँ किसी स्थान-विशेष पर केवल औसत तापमान को ही प्रकट करती हैं। भूपटल पर तापमान का क्षैतिज वितरण दिखाने के लिए इन्हीं रेखाओं के आधार पर मानचित्र बनाए जाते हैं। हम जानते हैं कि पृथ्वी अपने क्षैतिज अक्ष पर लम्बवत् अक्ष से [latex s=2]23\frac { 1^{ o } }{ 2 } [/latex] झुकी हुई है। इसी झुकाव तथा दैनिक गति के कारण पृथ्वी की सूर्य से सापेक्ष दूरियाँ परिवर्तित होती रहती हैं। अतः भूमध्य रेखा से उत्तर एवं दक्षिण की ओर की दूरी बढ़ने के साथ-साथ तापमान में उत्तरोत्तर कमी आती जाती है। यही कारण है कि ताप कटिबन्यों को अक्षांश रेखाओं की भाँति निर्धारित किया जाता है।

प्रश्न 7. दक्षिणी गोलार्द्ध की अपेक्षा उत्तरी गोलार्द्ध में समताप रेखाएँ अधिक अनियमित क्यों होती
उत्तर-हम जानते हैं कि उत्तरी गोलार्द्ध में महाद्वीप अधिक हैं तथा दक्षिणी गोलार्द्ध में महासागर अधिक है। महाद्वीप या स्थलखण्ड शीघ्र गर्म होते हैं तथा शीघ्र ही ठण्डे हो जाते हैं। इसके विपरीत महासागर देर से गर्म होते हैं और देर में ही ठण्ड़े होते हैं। इसलिए समताप रेखाएँ महाद्वीपों की अपेक्षा महासागरों की ओर अधिक नियमित रहती हैं। यही कारण है कि उत्तरी गोलार्द्ध की अपेक्षा दक्षिणी गोलार्द्ध में तापमान परिवर्तन बहुत कम रहता है और उत्तरी गोलार्द्ध में समताप रेखाओं की अनियमितता बनी रहती है।

प्रश्न 8. पृथ्वी की सतह पर कुल ऊष्मा बजट में भिन्नता क्यों होती है?
या पृथ्वी के कुछ क्षेत्रों में ताप अतिरेक क्यों पाया जाता है?
उत्तर-पृथ्वी की सतह पर प्राप्त विकिरण की मात्रा में भिन्नता पाई जाती है। वास्तव में पृथ्वी के कुछ भागों में विकिरण सन्तुलन में अधिशेष (Surplus) पाया जाता है, जबकि कुछ भागों में ऋणात्मक सन्तुलन होता है। चित्र 9.4 से स्पष्ट है कि शुद्ध विकिरण में अधिशेष 40° उत्तरी एवं दक्षिणी अक्षांशों में अधिक है, परन्तु ध्रुवों के पास इसमें कमी (Deficit) है। ऐसा उष्ण कटिबन्ध ताप संचयन के कारण बहुत अधिक गर्म नहीं होता और न के कारण है। फलस्वरूप उष्ण कटिबन्ध ताप संचयन के कारण बहुत अधिक गर्म नहीं होता और न ही उच्च अक्षांश ताप की अत्यधिक कमी के कारण पूरी तरह जमे हुए हैं। इस प्रकार पृथ्वी के उष्ण कटिबन्ध एवं शीत कटिबन्ध में मानव एवं जीव-जन्तु के लिए तापमान लगभग अनुकूल बना रहता है।
UP Board Solutions for Class 11 Geography Fundamentals of Physical Geography Chapter 9 Solar Radiation, Heat Balance and Temperature (सौर विimg 4

प्रश्न 9. तापमान के ऊध्र्वाधर वितरण से आप क्या समझते हैं?
या तापमान की सामान्य ह्रास दर का क्या अर्थ है?
उत्तर-तापमान का ऊर्ध्वाधर अध्ययन एक महत्त्वपूर्ण विषय है। सामान्यतः ऊँचाई के साथ-साथ तापमान कम होता जाता है। तापमान में इस गिरावट की दर एक डिग्री सेल्सियस प्रति 165 मीटर है। इसे सामान्य ह्रास दर कहते हैं। इस प्रकार ऊध्र्वाधर तापमान की सबसे बड़ी विशेषता है ऊँचाई के साथ-साथ तापमान में होने वाली कमी। किन्तु इसमें ऊँचाई के साथ तापमान घटने की कोई समान गति नहीं होती है। यह मौसम, स्थिति एवं दिन की अवधि के अनुसार कम व अधिक होता रहता है, परन्तु औसतन प्रति किमी की ऊँचाई पर 6.5 सेल्सियस कम होता है। सन् 1899 ई० में टेसेराइन व 1902 ई० में आसमन (Asman) नामक वैज्ञानिकों ने बताया है कि लगभग 12 किमी की ऊँचाई पर तापमान कम होना प्रायः रुक जाता है। फिर भी तापमान के ऊध्र्वाधर वितरण में यह एक मान्य तथ्य है कि वायुमण्डल की सबसे निचली परत (क्षोभमण्डल) जो धरातले के सम्पर्क में रहती हैं, सबसे अधिक गर्म होती है। यहीं से वायु की अन्य परतें गर्म होनी शुरू होती हैं। अतः हम जैसे-जैसे वायुमण्डल में ऊपर आते हैं। तापमान क्रमशः घटता जाता है।

प्रश्न 10. तापमान विलोमता के क्या कारण हैं? वर्णन कीजिए।
या तापमान विलोमता उत्पन्न करने वाली दशाओं का उल्लेख कीजिए।
उत्तर-तापमान की विलोमता में निम्नलिखित करण/दशाएँ सहायक होती हैं
1. लम्बी रात्रि-रात्रि के समय वायुमण्डल से ताप का विकिरण होता है तथा धीरे-धीरे वायुमण्डल ठण्डा होता रहता है। रात्रि जितनी लम्बी होगी, विकिरण भी उतना ही अधिक होगा। रात्रि के अन्तिम पहर में पृथ्वी तल का तापमान वायुमण्डल की अपेक्षा कम हो जाता है। इस प्रकार ऊपरी वायुमण्डल में ताप अधिक तथा धरातल के निकटवर्ती भागों मे ताप कम हो जाता है। फलतः विलोमता की स्थिति उत्पन्न हो जाती है।

2. स्वच्छ वायुमण्डल-रात्रि के समय जब आकाश अपेक्षाकृत अधिक स्वच्छ रहता है तो धरातल से विकिरित तापमान को वह नहीं रोक पाता जिससे विकिरित ताप शीघ्र ही वायुमण्डल में नष्ट हो जाता है। इस प्रकार लगातार विकिरण से धरातल ठण्डा होता जाता है, जबकि वायुमण्डल के उच्च भागों में अधिक तापमान पाया जाता है।

3. शीतल एवं शुष्क वायु-शीतल एवं शुष्क वायु में पृथ्वी से विकिरित ताप को ग्रहण करने की अधिक क्षमता होती है अतः धरातल ठण्डा ही रहता है, जबकि पृथ्वी से विकिरित तापमान ऊपरी भागों में एकत्रित हो जाती है तथा वायुमण्डल का ऊपरी भाग अधिक गर्म हो जाता है। इससे । विलोमता की स्थिति उत्पन्न हो जाती है।

4. शान्त वायु-जिन क्षेत्रों में रात्रि को वायु शान्त पाई जाती है, वहाँ धरातल से तापमान के विकिरण की प्रक्रिया निरन्तर होती रहने के कारण धरातल शीघ्र ही ठण्डी हो जाता है। फलतः धरातल के निकटवर्ती भाग में वायु की परतें भी शीतल हो जाती हैं। इसके साथ ही वायुमण्डल के ऊपरी भागों में ताप के एकत्रीकरण द्वारा तापमान बढ़ जाता है। इस प्रकार विलोमता की स्थिति पैदा हो जाती है।

5. हिमाच्छादित प्रदेश-जिन शीतप्रधान क्षेत्रों में हिम जमी रहती है, वहाँ धरातल पर निम्न तापमान पाए जाते हैं। क्योंकि हिम सूर्य की किरणों को शीघ्र की परावर्तित कर देती है। इस प्रकार धरातल तापमान ग्रहण नहीं कर पाता तथा ने ही गर्म हो पाता है। अत: ताप विलोमता की स्थिति उत्पन्न हो जाती है।

प्रश्न 11. वायुमण्डल किस प्रकार गर्म होता है? विकिरण की इस प्रक्रिया के कारण एवं महत्त्व बताइए।
उत्तर-सूर्य द्वारा विकिरित सौर ऊर्जा को सूर्यापत कहते हैं। पृथ्वी पर यह लघु तरंगों के रूप में आता है। पृथ्वी द्वारा विकिरित ऊर्जा ही भौमिक या पार्थिव विकिरण कहलाती है। वायुमण्डल पार्थिव विकिरण एवं शोषित विकिरण द्वारा गर्म होता है, किन्तु इसके गर्म होने में पार्थिव या भौतिक विकिरण का विशेष योगदान होता है। इसका मुख्य कारण निम्नलिखित है-

वायुमण्डल लघु तरंगों को अवशोषित नहीं कर पाता। फिर भी वायुमण्डल को बहुत कम ऊर्जा प्राप्त होती है। किन्तु लम्बी तरंगों को वायुमण्डल शीघ्र अवशोषित कर लेता है, क्योंकि वायुमण्डल में व्याप्त गैस, जलवाष्प, कार्बन डाइऑक्साइड अधिक ऊष्मा ग्रहण करने में सहयोग प्रदान करते हैं। इससे वायुमण्डल की निचली परतें ऊपरी परतों की अपेक्षा अधिक गर्म होती हैं।

प्रश्न 12. समपात रेखाओं के खिसकने की प्रवृत्ति किस ओर होती है-स्थल की ओर या जल की ओर? व्याख्या कीजिए।
उत्तर-उत्तरी गोलार्द्ध में जनवरी की समताप रेखाएँ ध्रुवों पर महासागरों की ओर तथा भूमध्य रेखा पर महाद्वीपों की ओर झुकी हुई होती है। जबकि दक्षिणी गोलार्द्ध में इनकी प्रवृत्ति विपरीत होती है अर्थात् भूमध्यरेखा पर महासागरों की ओर तथा ध्रुवों पर महाद्वीपों की ओर जुलाई महीने में समपात रेखाओं की प्रवृत्ति स्थल से समुद्र पर करते हुए भूमध्यरेखा की ओर तथा स्थलखण्ड को पार करते हुए ध्रुवों की ओर होती है। अतः तापमान प्रवणता ग्रीष्म ऋतु में शीत ऋतु की अपेक्षा मन्द होती है।

इस प्रकार समताप रेखाएँ उत्तरी गोलार्द्ध की अपेक्षा दक्षिणी गोलार्द्ध में अधिक नियमित होती हैं। वस्तुत: समताप रेखाओं की प्रवृत्ति अधिकतर स्थलखण्ड की ओर खिसकने की होती है क्योंकि स्थल का स्वभाव ऊष्मा को शीघ्रता से ग्रहण करने का होता है जबकि सागर या जलीय क्षेत्र देर में ऊष्मा ग्रहण करते हैं तथा देर में ही ठण्डे होते हैं। अतः ग्रीष्मकाल में समदाब रेखाएँ ध्रुवों की ओर तथा शीतकाल में भूमध्य रेखा की ओर खिसकती हैं।

प्रश्न 13. तापमान असंगति क्या है? यह कितने प्रकार की होती है? ।
उत्तर-तापमान असंगति-समुद्र तल से विभिन्न स्थलों की ऊँचाई, जल एवं स्थल की विषमता, प्रचलित पवन तथा समुद्री धाराओं के कारण एक ही अक्षांश पर स्थित विभिन्न स्थानों के तापमान में अन्तर पाया जाता है। किसी स्थान के औसत तापमान और उस अक्षांश के औसत तापमान के अन्तर को ‘तापमान असंगति’ कहते हैं। यह तापमान के औसत से विचलन की मात्रा और दिशा को प्रदर्शित करती है। तापमान असंगति उत्तरी गोलार्द्ध में अधिकतम तथा दक्षिणी गोलार्द्ध में न्यूनतम पाई जाती है। तापमाने असंगति के प्रकार–तापमान असंगति दो प्रकार की होती है-(i) ऋणात्मक तथा (ii) धनात्मक।

दीर्घ उत्तरीय प्रश्न

प्रश्न 1. तापमान के क्षैतिज वितरण को प्रभावित करने वाले कारकों की व्याख्या कीजिए तथा पृथ्वी
को ताप कटिबन्धों में विभाजित कीजिए। |
या तापमान के क्षैतिज वितरण को प्रभावित करने वाले कारकों की विवेचना कीजिए।
या भूमण्डल को प्रमुख ताप कटिबन्धों में विभक्त कर वर्णन कीजिए।
उत्तर- वायुमण्डल के तापमान को प्रभावित करने वाले कारक
धरातल पर तापमान का वितरण जलवायु को निर्धारित करता है। इसके अतिरिक्त वनस्पति, जीव-जन्तुओं एवं मानव तथा उसके क्रियाकलाप तापमान द्वारा प्रभावित होते हैं। वायुमण्डल का ताप निम्नलिखित कारकों द्वारा प्रभावित होता है

1. अक्षांशीय स्थिति-सूर्यातप अक्षांशीय स्थिति पर निर्भर करता है। विषुवत् रेखा पर सूर्यातप की . “अधिकतम मात्रा होती है। विषुवत् रेखा से उच्च अक्षांशों (उत्तर एवं दक्षिण) की ओर बढ़ने पर। ताप में कमी होती जाती है, क्योंकि सूर्य की किरणों के सापेक्ष तिरछेपन का प्रभाव पड़ता है। विषुवत् रेखा से ध्रुवों की ओर औसत तापमान क्रमशः घटता जाता है, परन्तु अधिकतम तापमान विषुवत् रेखा पर न होकर कर्क रेखा (ग्रीष्म ऋतु) तथा मकर रेखा (शीत ऋतु) पर होता है, क्योंकि सूर्य की स्थिति उत्तरायण एवं दक्षिणायण क्रमश: 6-6 माह में बदलती रहती है।

2. समुद्र-तल से ऊँचाई-समुद्र तल से लम्बवत् दूरी बढ़ने के साथ-साथ ताप में कमी होती जाती है। धरातल के समीप वाली वायु सबसे अधिक गर्म होती है तथा इसकी परतें भी मोटी होती हैं, जबकि ऊँचाई में वृद्धि के साथ-साथ वायु की परतें हल्की होती चली जाती हैं तथा वायु का तापमान भी कम होता जाता है। समुद्र-तल से प्रति 165 मीटर की ऊँचाई पर 1° सेग्रे ताप में कमी होती जाती है, जिस कारण उच्च पर्वतीय क्षेत्रों में हिमपात होता है।

3. थल एवं जल का वितरण-स्थल की यह प्रकृति होती है कि वे सूर्यातप के प्रभाव से शीघ्र ही गर्म हो जाते हैं तथा शीघ्र ही ठण्डे हो जाते हैं। इसके विपरीत जल क्षेत्र देर से गर्म एवं देर से ही ठण्डे होते हैं। इसी कारण धरातलीय ताप में भिन्नता पायी जाती है। जल के प्रभाव से उनके निकटवर्ती भागों की जलवायु समकारी होने की प्रवृत्ति रखती है, जबकि महाद्वीपों के आन्तरिक भागों में थल की अधिकता के कारण जलवायु में विषमता उत्पन्न हो जाती है।

4. समुद्र से दूरी-समुद्रतटीय भागों का तापमान समान रहता है, क्योंकि इन भागों में दैनिक तापान्तर बहुत ही कम होता है, परन्तु जैसे-जैसे सागरीय तटों से स्थल भागों की दूरी बढ़ती जाती है, | तापान्तर भी बढ़ता जाता है।

5. सागरीय धाराएँ-विषुवत् रेखा से ध्रुवों की ओर गर्म जलधाराएँ प्रवाहित होती हैं, जो शीतोष्ण एवं शीत कटिबन्धीय प्रदेशों के तटीय भागों के तापमान में वृद्धि कर देती हैं। इसके विपरीत ध्रुवीय प्रदेशों से विषुवतीय प्रदेशों की ओर शीतल जलधाराएँ प्रवाहित होती हैं, जो उष्ण कटिबन्ध के तटीय प्रदेशों के तापमान में कमी कर देती हैं। दो विपरीत धाराओं के मिलने से कोहरे की उत्पत्ति होती है।

6. प्रचलित पवनें-निम्न अक्षांशों से उच्च अक्षांशों की ओर प्रवाहित उष्ण पवनें तापमान को बढ़ा देती हैं, जबकि ध्रुवों की ओर से प्रवाहित पवनें निम्न अक्षांशों के तापमान में कमी कर देती हैं। प्रचलित पवनों के कारण ही जलधाराओं का प्रभाव तटवर्ती क्षेत्रों तक पहुँचता है। जल भागों से स्थल भागों की ओर प्रवाहित पवनें तापान्तर में कमी कर देती हैं। इसी प्रकार चक्रवातों के आगमन के समय तापमान बढ़ जाता है तथा समाप्ति पर घट जाता है।

7. धरातलीय विषमता-सामान्य रूप से किसी स्थान के तापमान पर धरातल के ढाल तथा उसकी प्रकृति का प्रभाव पड़ता है। हिमाच्छादित प्रदेशों में सूर्यातप के परावर्तन के कारण तापमान कम हो जाता है। मरुस्थलीय भागों में बलुई मिट्टी के द्वारा ताप का अधिक अवशोषण करने के फलस्वरूप तापमान में एकाएक वृद्धि हो जाती है। पर्वतीय भागों के जो ढाल सूर्य की किरणों के सामने पड़ते हैं, वे अधिक सूर्यातप ग्रहण कर लेते हैं। शीतल पवनों के मार्ग में पर्वतीय अवरोध आ जाने के कारण यह किसी प्रदेश के तापमान को कम करने से रोकता है।

8. मिट्टी की प्रकृति-अधिक गहरे रंग की मिट्टियाँ ताप का अधिक अवशोषण कर लेती हैं जिससे तापमान में वृद्धि हो जाती है। इसके विपरीत हल्के रंग की मिट्टियाँ कम मात्रा में ताप का अवशोषण करती हैं, जिससे तापमान में वृद्धि नहीं हो पाती। काली मिट्टी के प्रदेशों में 8 से 14 प्रतिशत तक ताप का परावर्तन हो जाता है। बालू-प्रधान क्षेत्रों में चीका मिट्टी की अपेक्षा ताप का अधिक अवशोषण होता है।

9. मेघाच्छादन-सूर्य की किरणों को पृथ्वी तक पहुँचने में बादलों द्वारा बाधा उत्पन्न होती है, क्योंकि ये ताप को परावर्तित कर देते हैं। बादलरहित क्षेत्रों में तापमान अधिक होता है। ऐसे प्रदेशों में जहाँ पर हर समय बादल छाये रहते हैं, तापमान अधिक नहीं हो पाता है।

तापमान का क्षैतिजीय वितरण

सामान्य रूप से धरातल पर तापमान का वितरण समताप रेखाओं (Isotherms) द्वारा प्रदर्शित किया। जाता है। समुद्र-तल से समान तापमान वाले स्थानों को मिलाने वाली रेखाओं को ‘समताप रेखा’ कहा जाता है। ये रेखाएँ पूर्व से पश्चिम दिशा में मानचित्रों पर निर्मित की जाती हैं, जो प्रायः अक्षांश रेखाओं के समानान्तर ही होती हैं। इससे स्पष्ट होता है कि तापमान के क्षैतिजीय वितरण पर अक्षांशों का सबसे अधिक प्रभाव पड़ता है। स्थल एवं जलखण्डों के मिलन-स्थलों पर समताप रेखाओं में झुकाव आ जाता है। इसका प्रमुख कारण जल एवं थल भागों के गर्म एवं ठण्डा होने की प्रकृति में पर्याप्त अन्तर होना है। दक्षिणी गोलार्द्ध में जल की अधिकता के कारण समताप रेखाएँ दूर-दूर तथा सपाट होती हैं। इसके विपरीत उत्तरी गोलार्द्ध में थल भाग की अधिकता के कारण समताप रेखाएँ पास-पास तथा तापमान की भिन्नता के कारण अधिक झुकाव वाली होती हैं। समताप रेखाएँ समीप होने से तापमान की तीव्रता को. प्रकट करती हैं।

तापमान को प्रादेशिक वितरण भी क्षैतिजीय वितरण को ही प्रकट करता है। ताप वितरण की समानता तथा उनकी विशेषताओं को आधार मानकर सम्पूर्ण ग्लोब को कुछ मण्डलों में विभाजित कर लिया जाता है। इन मण्डलों का विभाजन तथा सीमांकन अक्षांशीय आधार पर किया जाता है। इस आधार पर ग्लोब को निम्नलिखित तीन कटिबन्धों में विभाजित किया जा सकता है
UP Board Solutions for Class 11 Geography Fundamentals of Physical Geography Chapter 9 Solar Radiation, Heat Balance and Temperature (सौर विimg 5
1. उष्ण कटिबन्ध
पृथ्वी की घूर्णन गति के कारण कर्क तथा मकर रेखाओं (23 1/2%° उत्तरी एवं 23 1/2° दक्षिणी अक्षांश) के मध्य सूर्य की किरणें वर्ष में दो बार लम्बवत् चमकती हैं। इसीलिए भूमध्य रेखा के आस-पास वाले भागों में उच्चतम तापमान रहने के कारण शीत ऋतु नाममात्र को भी नहीं होती, जबकि कर्क तथा मकर रेखाओं के समीपवर्ती भागों में ग्रीष्म एवं शीत ऋतुएँ क्रमशः आरम्भ हो जाती हैं। अत: विषुवत् रेखा के समीप दोनों ओर शीत ऋतु रहित इस कटिबन्ध को उष्ण कटिबन्ध

2. शीतोष्ण कटिबन्ध ।
इस कटिबन्ध का विस्तार 23 1/2° से.66 1/2%° उत्तरी एवं दक्षिणी अक्षांशों (ध्रुव वृत्तों) के मध्य है। यहाँ पर सूर्य की किरणों के सापेक्ष तिरछापन प्रारम्भ हो जाता है। इस कटिबन्ध में दिन तथा रात की अवधि ऋतु के अनुसार घटती-बढ़ती रहती है, परन्तु यह अवधि 24 घण्टे से अधिक नहीं हो पाती है। तापमान में घट-बढ़ के कारण ही इसे शीतोष्ण कटिबन्ध कहते हैं। स्थिति के अनुसार इसे निम्नलिखित दो भागों में बाँटा जा सकता है

(अ) उत्तरी शीतोष्ण कटिबन्ध-इस कटिबन्ध का विस्तार कर्क रेखा से उत्तरी ध्रुव वृत्त तक है। सूर्य की उत्तरायण स्थिति में उत्तरी गोलार्द्ध में ग्रीष्म ऋतु होती है। इसी कारण यहाँ ग्रीष्म तथा शीतकाल : का तापान्तर भी अधिक होता है।

(ब) दक्षिणी शीतोष्ण कटिबन्ध-इस कटिबन्ध का विस्तार मकर रेखा से दक्षिणी ध्रुव वृत्त तक है। सूर्य की दक्षिणायण स्थिति में दक्षिणी गोलार्द्ध में ग्रीष्म ऋतु हो जाती है; अत: ऋतुओं के क्रमशः परिवर्तन के कारण ग्रीष्म एवं शीतकाल में तापान्तर अधिक होता है।

3. शीत कटिबन्ध
इस कटिबन्ध का विस्तार दोनों गोलार्डो में 66 1/2° से उत्तरी एवं दक्षिणी ध्रुव केन्द्रों तक है। इन क्षेत्रों में सूर्य की किरणों के सापेक्ष तिरछेपन का व्यापक प्रभाव पड़ता है जिससे औसत तापमान निम्न रहता है। दिन-रात की अवधि 24 घण्टे से अधिक हो जाती है। ध्रुवों पर छ: महीने के दिन तथा क्रमशः छः महीने की रातें होती हैं। सूर्य की किरणें कभी भी लम्बवत् नहीं चमकती हैं। इसीलिए यह कटिबन्ध हिम से ढका रहता है। इसे दो निम्नलिखित भागों में बाँटा जा सकता है

(अ) उत्तरी शीत कटिबन्ध-इस कटिबन्ध का विस्तार उत्तरी ध्रुव वृत्त से उत्तरी ध्रुव तक है। यहाँ पर कठोर शीत पड़ती है तथा वर्ष भर ये भाग हिम से ढके रहते हैं।
(ब) दक्षिणी शीत कटिबन्ध-इस ताप कटिबन्ध का विस्तार दक्षिणी ध्रुव वृत्त से दक्षिणी ध्रुव तक है। सूर्य की किरणों के सापेक्ष तिरछेपन के कारण छ: माह के दिन तथा छ: माह की रातें होती हैं। यह कटिबन्ध सदैव बर्फ से ढका रहता है। यहाँ पर कठोर शीत पड़ती है।

प्रश्न 2. ताप की विलोमता या व्युत्क्रमण से आप क्या समझते हैं? यह कितने प्रकार की होती है?
या वायुमण्डल में तापमान विलोम का वर्णन कीजिए।
उत्तर-तापमान का विलोम अथवा ताप का व्युत्क्रमण
वायुमण्डल में ऊँचाई के अनुसार तापमान कम होता जाता है। ऐसा मुख्य रूप से 8 किमी से 18 किमी की ऊँचाई तक ही होता है। सामान्य रूप से प्रति किमी की ऊँचाई पर 65° सेग्रे ताप कम हो जाता है, परन्तु कुछ विशेष परिस्थितियों में तापमान घटने के स्थान पर बढ़ जाता है, जिसे ऋणात्मक ताप-पतन दर (Negative Lapse Rate of Temperature) कहते हैं। इससे तापमान की ऊर्ध्वाधर प्रवणता बदल जाती है। इस अवस्था में वायु की गर्म परतें ऊपर पहुँच जाती हैं तथा शीतल परतें नीचे आ जाती हैं। तापमान की इस दशा को तापमान का विलोम या तापमान का प्रतिलोमन अथवा ताप का व्युत्क्रमण कहा जाता है।
UP Board Solutions for Class 11 Geography Fundamentals of Physical Geography Chapter 9 Solar Radiation, Heat Balance and Temperature (सौर विimg 6
ताप का यह प्रतिलोमन धरातल के निकट भी हो सकता है तथा ऊँचाई पर भी। परन्तु धरातल के निकट होने वाला प्रतिलोमन अस्थायी होता है, जबकि ऊँचाई पर होने वाला प्रविंलोमन स्थायी होता है; क्योंकि अधिक ऊँचाई पर वायु की परतों को ठण्डा होने में अधिक समय लग जाता है। तापीय प्रतिलोमन ध्रुवीय प्रदेशों, हिमाच्छादित प्रदेशों एवं घाटियों में अधिक होता है। गर्म एवं ठण्डी धाराओं के मिलन-स्थल पर भी इस प्रकार की अवस्थाएँ उत्पन्न हो जाती हैं। तापमान के सम्बन्ध में ट्रिवार्था ने कहा है कि “ऐसी अवस्था जिसमें शीत-प्रधान वायु धरातल के चित्र निकट व उष्ण वायु ऊपर पायी जाती है, तापमान का व्युत्क्रमण कहलाता है।”

तापीय विलोमता के प्रकार

ताप की विलोमता के निम्नलिखित प्रकार हैं
1. धरातलीय विलोमता (Surface Inversion)-इस प्रकार की ताप विलोमता पृथ्वी के विकिरण द्वारा अधिक ऊर्जा के प्रवाहित होने से होती है। विपरीत अर्थात् शीतल एवं उष्ण वायुराशियों के आगमन से भी धरातलीय विलोमता उत्पन्न हो जाती है।

2. उच्च धरातलीय विलोमता (Upper Surface Inversion)-धरातल से कुछ ऊँचाई पर वायु-राशियों के अवतलन से इस प्रकार की विलोमता उत्पन्न होती है। यह प्रतिचक्रवातों पर निर्भर करता है। वायु की उष्ण परतें ऊपर तथा शीतल परतें नीचे हो जाती हैं। इससे वायुमण्डल में स्थिरता आ जाती है।

वायुमण्डल की ओजोन परत के ऊपर सर्वाधिक ताप तथा निचली परत में कम ताप मिलता है। यह भी उच्च धरातलीय विलोमता का उदाहरण है।

3. वाताग्री विलोमता (Frontal Inversion)-जब उष्ण एवं शीत प्रधान वायुराशियाँ पास-पास होती हैं, तो उष्ण वायु शीतल वायु के ऊपर फैल जाती है। जिस ओर इन वायुराशियों का ढालें होता है, उसे वाताग्र (Front) कहते हैं। तापक्रम एवं आर्द्रता के कारण यह विलोमता उत्पन्न होती है।

तापीय विलोमता का महत्त्व

मानव तथा आर्थिक विकास पर तापीय विलोमती का प्रभाव महत्त्वपूर्ण है। इसके द्वारा मौसम तथा जलवायु दोनों ही प्रभावित होते हैं। मेघों की बनावट, वर्षा तथा वायुमण्डल की पारदर्शकता पर इसका विशेष प्रभाव पड़ता है। इससे कोहरे की उत्पत्ति होती है। गर्म एवं ठण्डी धाराओं के मिलने से भी कोहरे की उत्पत्ति होती है। यह मछली व्यवसाय के लिए भी उपयोगी है। कहीं-कहीं पर कोहरा फसलों के लिए लाभदायक होता है। कोहरे के कारण अरब में यमन की पहाड़ियों पर कहवे की खेती सम्भव हो पायी है, क्योंकि यह सूर्य की किरणों के तीक्ष्ण प्रभाव को कम कर देता है, परन्तु कोहरे के प्रभाव से फसलों को सूर्य का प्रकाश नहीं मिल पाता, जिससे नाइट्रोजन की कमी हो जाती है तथा फसलें पीली पड़ जाती हैं। पहाड़ी ढालों पर फलों की खेती में ताप का व्युत्क्रमण सहायक होता है।

We hope the UP Board Solutions for Class 11 Geography: Fundamentals of Physical Geography Chapter 9 Solar Radiation, Heat Balance and Temperature (सौर विकिरण, ऊष्मा संतुलन एवं तापमान) help you. If you have any query regarding UP Board Solutions for Class 11 Geography: Fundamentals of Physical Geography Chapter 9 Solar Radiation, Heat Balance and Temperature (सौर विकिरण, ऊष्मा संतुलन एवं तापमान), drop a comment below and we will get back to you at the earliest.

UP Board Solutions for Class 11 Geography: Fundamentals of Physical Geography Chapter 13 Water

UP Board Solutions for Class 11 Geography: Fundamentals of Physical Geography Chapter 13 Water (महासागरीय जल)

These Solutions are part of UP Board Solutions for Class 11 Geography. Here we have given UP Board Solutions for Class 11 Geography: Fundamentals of Physical Geography Chapter 13 Water (महासागरीय जल)

पाठ्य-पुस्तक के प्रश्नोत्तर

1. बहुवैकल्पिक प्रश्न
प्रश्न (i) उस तत्त्व की पहचान करें जो जलीय चक्र का भाग नहीं है-
(क) वाष्पीकरण
(ख) वर्षण
(ग) जलयोजन
(घ) संघनन
उत्तर-(ग) जलयोजन।।

प्रश्न (i) महाद्वीपीय ढाल की औसत गहराई निम्नलिखित के बीच होती है|
(क) 2-20 मीटर ।
(ख) 20-200 मीटर
(ग) 200-2,000 मीटर ।
(घ) 2,000-20,000 मीटर
उत्तर-(ख) 20-200 मीटर।

प्रश्न (ii) निम्नलिखित में से कौन-सी लघु उच्चावच आकृति महासागरों में नहीं पाई जाती है?
(क) समुद्री टोला ।
(ख) महासागरीय गभीर
(ग) प्रवालद्वीप ।
(घ) निमग्न द्वीप
उत्तर-(ग) प्रवालद्वीप।

प्रश्न (iv) लवणता को प्रति समुद्री जल में घुले हुए नमक (ग्राम) की मात्रा से व्यक्त किया जाता है
(क) 10 ग्राम
(ख) 100 ग्राम
(ग) 1,000 ग्राम
(घ) 10,000 ग्राम
उत्तर-(ग) 1,000 ग्राम

प्रश्न (v) निम्न में से कौन-सा सबसे छोटा महासागर है?
(क) हिन्द महासागर
(ख) अटलाण्टिक महासागर
(ग) आर्कटिक महासागर ।
(घ) प्रशान्त महासागर
उत्तर-(ग) आर्कटिक महासागर।

2. निम्नलिखित प्रश्नों के उत्तर लगभग 30 शब्दों में दीजिए
प्रश्न (i) हम पृथ्वी को नीला ग्रह क्यों कहते हैं?
उत्तर-जल हमारे सौरमण्डल का दुर्लभ पदार्थ है। सौरमण्डल में पृथ्वी ग्रह के अतिरिक्त अन्यत्र कहीं जल नहीं है। इस दृष्टि से पृथ्वी के जीवन सौभाग्यशाली हैं कि यह एक जलीय ग्रह है, अन्यथा पृथ्वी पर जीव-जन्तुओं का अस्तित्व ही नहीं होता। वस्तुतः सौभाग्य से पृथ्वी के धरातल पर जल की प्रचुर आपूर्ति है। इसीलिए पृथ्वी को नीला ग्रह कहा जाता है।

प्रश्न (ii) महाद्वीपीय सीमान्त क्या होता है?
उत्तर-महाद्वीपीय सीमान्त वह क्षेत्र है जहाँ महासागर महाद्वीपों से मिलते हैं। प्रत्येक महाद्वीप का सीमान्त उथले समुद्रों तथा खाड़ियों से घिरा होता है। इसकी ढाल प्रवणता अत्यन्त कम होती है, जिसका औसत लगभग 1 डिग्री या इससे भी कम हो सकता है।

प्रश्न (ii) विभिन्न महासागरों के सबसे गहरे गर्गों की सूची बनाइए।
उत्तर-महासागरीय गर्त महासागरों के सबसे गहरे भाग होते हैं। अभी तक महासागरों में लगभग 57 गर्ते की खोज की गई है जिसमें सबसे अधिक गर्त प्रशान्त महासागर में स्थित है। प्रमुख महासागरीय गर्गों की संख्या इस प्रकार है1. प्रशान्त महासागर 32, 2. अटलाण्टिक महासागर 19, 3. हिन्द महासागर 6.

प्रश्न (iv) ताप प्रवणता क्या है?
उत्तर-महासागरीय गहराई में जहाँ तापमान में तीव्र कमी आती है उसे ताप प्रवणता कहते हैं। ऐसा अनुमान है कि जल के कुल आयतन का लगभग 90 प्रतिशत गहरे महासागर में ताप प्रवणता के नीचे पाया जाता है। इस क्षेत्र में तापमान 0° सेल्सियस पहुँच जाता है।

प्रश्न (v) समुद्र में नीचे जाने पर आप ताप की किन परतों का सामना करेंगे? गहराई के साथ तापमान में भिन्नता क्यों आती है?
उत्तर-महासागर की सतह से विभिन्न गहराई तक जल के तापमान के आधार पर कई परतें मिलती हैं। सामान्यतः मध्य एवं निम्न अक्षांशों में ऐसी हीं निम्नलिखित तीन ताप परतें मिलती हैं-

  • गर्म महासागरीय जल की सबसे ऊपरी परत जो लगभग 500 मीटर मोटी होती है, का तापमान 20°C से 25°C के बीच होता है।
  • ताप प्रवणता परतं जो पहली परत के नीचे स्थित होती है, में गहराई बढ़ने के साथ तापमान में तीव्र | गिरावट आती है।
  • बहुत अधिक ठण्डी परत जो गम्भीर महासागरीय तली तक विस्तृत होती है।

महासागरों में उच्च तापमान प्रायः उसकी ऊपरी सतह पर ही पाया जाता है, क्योंकि महासागर का यह भाग प्रत्यक्ष रूप से सूर्य की ऊष्मा प्राप्त करता है। इसके साथ गहराई पर जाने में सूर्य का ताप कम प्राप्त होता है, इसलिए सागरीय जल के तापमान में गहराई बढ़ने के साथ-साथ भिन्नताएँ मिलती हैं।

प्रश्न (vi) समुद्री जल की लवणता क्या है?
उत्तर-महासागरीय जल के खारेपन अथवा उसमें स्थित लवण की मात्रा को ही महासागरीय लवणता कहते हैं। महासागरीय जल की औसत लवणता लगभग 35 प्रति हजार अर्थात् 1000 ग्राम समुद्री जल में 35 ग्राम लवण पाया जाता है।

3. निम्नलिखित प्रश्नों के उत्तर लगभग 150 शब्दों में दीजिए
प्रश्न (i) जलीय चक्र के विभिन्न तत्त्व किस प्रकार अन्तर-सम्बन्धित है?
उत्तर–जल एक चक्र के रूप में महासागर से धरातल पर और धरातल से महासागर तक चलने वाली पक्रिया है। यह चक्र पृथ्वी पर, पृथ्वी के नीचे तथा ऊपर वायुमण्डल में जल के संचलन की व्यवस्था करता है। पृथ्वी पर जलचक्र करोड़ों वर्षों से कार्यरत है और आगे भी पृथ्वी पर जब तक जीवन है, यह चक्र सक्रिय रहेगा। सभी प्रकार के जीव इसी जलचक्र पर निर्भर हैं। जलचक्र से सम्बन्धित तत्त्व, जो परस्पर अन्तर-सम्बन्धित पक्रिया में जलचक्र को सक्रिय रखते हैं, निम्नलिखित हैं-

1. वाष्पीकरण-वाष्पीकरण जलचक्र का सबसे महत्त्वपूर्ण घटक है। महासागर से वायुमण्डल का परिसंचलन इसी प्रक्रिया द्वारा सम्पन्न होता है। इस प्रक्रिया में सौर ताप से जल गर्म होकर वाष्प के रूप में वायुमण्डल में जाता है।

2. संघनन-जल के गैसीय अवस्था से द्रवीय अवस्था में परिवर्तन को संघनन कहते हैं। जलचक्र में संघनन का कार्य वायुमण्डल में सम्पन्न होता है। इसी प्रक्रिया में महासागर का जल वायुमण्डल से धरातल पर पहुँचता है।

3. अवक्षेपण-इस प्रक्रिया में वायुमण्डले की जलवाष्प जल-बूंदों में जलवृष्टि के रूप में पृथ्वी पर आती है।

इस प्रकार वाष्पीकरण, संघनन एवं अवक्षेपण तत्त्वों द्वारा जलचक्र की प्रक्रिया सतत् चलती रहती है। (चित्र 13.1)।
UP Board Solutions for Class 11 Geography Fundamentals of Physical Geography Chapter 13 Water (महासागरीय जल) img 1

प्रश्न (ii) महासागरों के तापमान वितरण को प्रभावित करने वाले कारकों को परीक्षण कीजिए।
उत्तर-महासागरीय जल के तापमान वितरण को प्रभावित करने वाले कारक निम्नलिखित हैं
1. अक्षांश-ध्रुवों की ओर सौर विकिरण की मात्रा घटने के कारण महासागरों के सतही जल का तापमान विषुवत् वृत्त से ध्रुवों की ओर घटता जाता है।

2. स्थल तथा जल का असमान वितरण-उत्तरी गोलार्द्ध के महासागर दक्षिणी गोलार्द्ध के महासागरों की अपेक्षा स्थल के बहुत बड़े भाग से सम्बद्ध हैं। इसलिए उत्तरी गोलार्द्ध दक्षिणी | गोलार्द्ध की अपेक्षा अधिक ऊष्मा ग्रहण करता है।

3. प्रचलित हवाएँ-स्थलों की ओर से महासागरों की ओर चलने वाली हवाएँ समुद्री सतह के गर्म जल को तट से दूर धकेल देती हैं जिसके परिणामस्वरूप नीचे का ठण्डा जल ऊपर की ओर आ
जाता है। परिणामस्वरूप इस प्रक्रिया से समुद्र के तापमान में वृद्धि हो जाती है।

4. महासागरीय धाराएँ-गर्म समुद्री धाराएँ ठण्डे क्षेत्रों के जल का तापमान बढ़ा देती हैं, जबकि ठण्डी धाराएँ गर्म समुद्री क्षेत्रों के जल का तापमान कम कर देती हैं। उदाहरण के लिए-गल्फ-स्ट्रीम गर्म जलधारा यूरोप के पश्चिमी तट के जल का तापमान बढ़ा देती है। इसके विपरीत लेब्रेडोर की ठण्डी जलधारा उत्तरी अमेरिका के उत्तरी-पूर्वी तट के तापमान को कम कर देती है।

परीक्षोपयोगी प्रश्नोत्तर

बहुविकल्पीय प्रश्न
प्रश्न 1. निम्नलिखित में से कौन महासागरीय तली में सबसे ऊपर स्थित होता है?
(क) महाद्वीपीय मग्नतट
(ख) महाद्वीपीय मग्नढाल
(ग) महासागरीय द्रोणी
(घ) महासागरीय गर्त
उत्तर-(क) महाद्वीपीय मग्नतट।

प्रश्न 2. निम्नलिखित में विश्व का सर्वाधिक लवणता वाला सागर कौन-सा है?
(क) मृत सागर ।
(ख) बाल्टिक सागर
(ग) काला सागर
(घ) अजोव सागर
उत्तर-(क) मृत सागर।

अतिलघु उत्तरीय प्रश्न

प्रश्न 1. महासागरीय मग्न तट से आप क्या समझते हैं?
उत्तर-यह समुद्र के नितल का अति मन्द ढालयुक्त भाग है, जो महाद्वीप के चारों ओर फैला हुआ है।

प्रश्न 2. महासागरीय जल की लवणता को समझाइए।
उत्तर-सागरीय जल में लवणों की उपस्थिति से उत्पन्न खारेपन को महासागरीय जल की लवणता कहा जाता है।

प्रश्न 3. महासागरों के तलीय उच्चावच का रेखाचित्र बनाइए।
उत्तर-
UP Board Solutions for Class 11 Geography Fundamentals of Physical Geography Chapter 13 Water (महासागरीय जल) img 2

प्रश्न 4. विश्व के महासागरीय गत का वितरण लिखिए।
उतर-विश्व के लगभग 7% भाग पर, महासागरीय गर्गों का विस्तार है। कुल 57 गर्गों में से 32 प्रशान्त महासागर में, 19 अटलाण्टिक महासागर में और 6 हिन्द महासागर में स्थित हैं। मेरियाना (प्रशान्त महासागर) नाम का महासागरीय गर्त लगभग 11 किमी गहरा है जो विश्व का सर्वाधिक गहरा महासागरीय गर्त है।

प्रश्न 5. विश्व के किस भाग में महाद्वीपीय मग्नतट की अनुपस्थिति मिलती है?
उतर-विश्व में दक्षिणी अमेरिका के पश्चिमी तट पर महाद्वीपीय मग्नतट लगभग अनुपस्थित मिलते हैं।

प्रश्न 6. महासागरीय जल के तापमान वितरण की प्रमुख विशेषता क्या है?
उत्तर-महासागरीय जल के तापमान वितरण में क्षेत्रीय विविधता पाई जाती है। भूमध्यरेखा के समीप महासागरीय जल सबसे अधिक गर्म और ध्रुवों की ओर क्रमशः ठण्डा होता जाता है।

प्रश्न 7. विश्व के सर्वाधिक लवणता वाले क्षेत्र बतलाइए।
उत्तर-विश्व में सर्वाधिक लवणता वाले क्षेत्र आयनमण्डल में पाए जाते हैं। अटलाण्टिक महासागर में आयनमण्डलों के समीप लवणता लगभग 37 प्रति हजार है। स्थल से घिरे समुद्रों में संयुक्त राज्य अमेरिका की ग्रेट साल्ट लेक में 220, मृत सागर में 240 तथा तुर्की की वान झील में 330 प्रति हजार लवणती सबसे अधिक है।

प्रश्न 8. विश्व के समुद्रों की औसत लवणता मात्रा कितनी है?
उत्तर-समुद्र के एक हजार ग्राम जल में औसत 35 ग्राम लवण घोल के रूफ़ में विद्यमान हैं। इस प्रकार विश्व के समुद्री जल की औसत लवणता 35 प्रति हजार (35%) है।

प्रश्न 9. विश्व के कम एवं अधिक लवणता वाले क्षेत्रों के नाम लिखिए।
उत्तर-विश्व के कम और अधिक लवणता वाले क्षेत्र निम्नानुसार हैं

  • भूमध्य रेखा पर कम लवणता,
  • व्यापारिक पवनों के क्षेत्रों (आयनमण्डल) के समीप कम लवणता,
  • पछुआ पवनों के क्षेत्रों में कम लवणता,
  • ध्रुवीय प्रदेशों में कम लवणता।

प्रश्न 10. सागरीय मैदानों का विस्तार कहाँ मिलता है?
उत्तर-सागरीय मैदानों का विस्तार 20° उत्तरी अक्षांश से 60° दक्षिणी अक्षांशों के मध्य अधिक पाया जाता है। महासागरों के विचार से प्रशान्त महासागर में सागरीय मैदान अधिक मिलते हैं।

प्रश्न 11. महाद्वीपीय ढालों की उत्पत्ति कैसे हुई है?
उत्तर-महासागरीय ढाल महासागरीय तल का एक सँकरा भाग होता है। इनकी उत्पत्ति महाद्वीपों के किनारे मुड़ने तथा अवसादों की मोटी परत एकत्रित होने के फलस्वरूप हुई है।

प्रश्न 12. विश्व के प्रसिद्ध महासागरीय पठार का नाम व स्थिति लिखिए।
उत्तर-विश्व का प्रसिद्ध महासागरीय पठार अटलाण्डिटक महासागर के मध्य में स्थित मध्य अटलाण्टिक कटक’ है। इसके अतिरिक्त पूर्वी प्रशान्त महासागर में स्थित ‘एल्बटरॉस पठार’ भी सागरीय पठार का अच्छा उदाहरण है।

प्रश्न 13. महासागरीय जल की लवणता समझाइए।
उत्तर-सागरीय जल के भाग तथा उसमें घुले हुए पदार्थों के भार के अनुपात को लवणता कहते हैं। एक किग्रा समुद्री जल में घुले हुए ठोस पदार्थों की मात्रा ही लवणता है। सामान्यतया महाद्वीपीय जल में प्रति हजार ग्राम में 35 ग्राम लवणता पाई जाती है (35%)।

प्रश्न 14. पृथ्वी के कितने भाग पर जल पाया जाता है?
उत्तर-पृथ्वी के 71% भाग पर जल पाया जाता है।

प्रश्न 15. समुद्र विज्ञान से आप क्या समझते हैं?
उत्तर-समुद्र विज्ञान वह विज्ञान है जिसमें समुद्र के जल, जलधारा, ज्वारभाटा तथा अन्य सम्बन्धित तथ्यों का अध्ययन किया जाता है।

प्रश्न 16. प्रमुख महासागरों के नाम लिखिए।
उत्तर-प्रमुख महासागरों के नाम निम्नलिखित हैं–

  1. प्रशान्त महासागर,
  2. अटलाण्टिक महासागर,
  3. हिन्द महासागर,
  4. आर्कटिक महासागर,
  5. दक्षिणी हिम महासागर।।

प्रश्न17. विश्व का सबसे बड़ा महासागर एवं सबसे गहरा गर्त कौन-सा है?
उत्तर-विश्व का सबसे बड़ा महासागर प्रशान्त महासागर एवं सबसे गहरा गर्त मेरियाना (11,033 मीटर) है जो प्रशान्त महासागर में स्थित है। |

लघु उत्तरीय प्रश्न

प्रश्न 1. महासागरीय तलीय उच्चावच पर प्रकाश डालिए तथा इसकी लपरेखा स्पष्ट कीजिए।
उत्तर-महासागरीय तल
महासागरों की तली को धरातल अत्यन्त विषम होता है। भूपटल की भाँति सागरीय तली में भी पर्वत, पठार, मैदान, गर्त आदि पाए जाते हैं, जिन्हें महासागरीय उच्चावच कहते हैं। पृथ्वी के ऊँचे भागों की अपेक्षा महासागर कहीं अधिक गहरे हैं। महासागरों की गहराई पर प्रकाश डालते हुए प्रो० जॉन मूरे ने लिखा है-“3,500 मीटर से अधिक ऊँचा भाग समस्त भूमण्डल का मात्र 1% है, जबकि समुद्रों में 3,500 मीटर से अधिक गहरे भाग 46% हैं। वस्तुतः महासागरीय नितल का अधिकांश भाग 3 किमी से 6 किमी तक गहरा है।

महासागरीय तल की रूपरेखा

महासागरीय नितल को उच्चावच की दृष्टि से निम्नलिखित भागों में वर्गीकृत किया जा सकता है

  1. महाद्वीपीय मग्नतट (Continental Shelf),
  2. महाद्वीपीय ढाल (Continental Slope),
  3. गहन सागरीय मैदान (Deep Sea Plains),
  4. महासागरीय पठार (Oceanic Plateaus),
  5. महासागरीय गर्त (Oceanic Deeps)।

प्रश्न 2. महाद्वीपीय मग्नतट का क्या अर्थ है? इनकी मुख्य विशेषताएँ बतलाइए।
उत्तर-महाद्वीपीय मग्नतट महासागरों व महाद्वीपों के मिलन-स्थल होते हैं। इसका ढाल 1° से 3° तक, गहराई 200 मीटर तक तथा चौड़ाई कुछ किमी से 1,000 किमी तक होती है। विश्व में सबसे अधिक मग्नतट अन्ध महासागर में विद्यमान हैं। महाद्वीपीय मग्नतटों की मुख्य विशेषताएँ निम्नांकित हैं–

  • पृथ्वी पर महासागरों के कुल क्षेत्रफल का लगभग 7.5 से 8.5% भाग महाद्वीपीय मग्नतट के रूप में अवस्थित है।
  • महाद्वीपीय मग्नतट समुद्री खाद्य पदार्थों की उपलब्धता, मत्स्य आखेट और खनिज तेल एवं गैस उत्पादन के प्रमुख क्षेत्र होते हैं।
  • महाद्वीपीय मग्नतट मत्स्य उत्पादन के अनुकूल क्षेत्र होते हैं। विश्व के विशालतम मत्स्य संग्रहण क्षेत्र डॉगर बैंक और ग्राण्ड बैंक इन्हीं तटों पर स्थित मिलते हैं।
  • ये तट प्रकाश और गर्मी की उपस्थिति के कारण जल-जीवों तथा सागरीय वनस्पति के विपुल । भण्डार होते हैं।

प्रश्न 3. महासागरीय लवणता वितरण की विभिन्नता के दो महत्त्वपूर्ण कारणों का उल्लेख कीजिए।
उत्तर-महासागरों में लवणता वितरण की भिन्नता के दो महत्त्वपूर्ण कारण निम्नांकित हैं
1: स्वच्छ जल की आपूर्ति-महासागरों में स्वच्छ जल की आपूर्ति जितनी अधिक मात्रा में होती है, लवणता उतनी ही कम होती है। इसीलिए भूमध्य रेखा के निकट वर्षा की अधिकता के कारण | लवणता कम तथा आयन रेखाओं के निकट कम वर्षा होने के कारण लवणता अधिक मिलती है।

2. वाष्पीकरण की मात्रा एवं तीव्रता-वाष्पीकरण की मात्रा की अधिकता के कारण लवणता की मात्रा में वृद्धि होती है। कर्क व मकर रेखाओं के निकट निर्मल आकाश व प्रखर सूर्य की किरणों के कारण वाष्पीकरण की मात्रा अधिक रहती है। इसी कारण लाल सागर में लवणता 40% मिलती है।

प्रश्न 4. अन्ध महासागर एवं प्रशान्त महासागर की तापमान विभिन्नताओं पर प्रकाश डालिए।
उत्तर-अन्ध महासागर-अन्ध महासागर में गर्म एवं ठण्डी धाराओं का प्रभाव समताप रेखाओं के वितरण पर विशेष रूप से षड़ता है। उत्तरी अन्ध महासागर में समताप रेखाएँ पश्चिम की ओर परस्पर मिलती हैं, जबकि उत्तर-पूर्व में समताप रेखाएँ दूर-दूर स्थित हैं। मध्यवर्ती अन्ध महासागर में समताप रेखाओं का वितरण बड़ा ही असंयमित है, क्योंकि यहाँ सागर एवं मौसम की दिशा अनिश्चित रहती है।

प्रशान्त महासागर-प्रशान्त महासागर में समताप रेखाएँ प्रायः अक्षांश रेखाओं के समानान्तर मिलती हैं, क्योंकि यह महासागर आकार में सबसे बड़ा है, जिससे स्थलीय क्षेत्रों एवं पवनों का विशेष प्रभाव यहाँ नहीं पड़ता है। यहाँ विषुवत् रेखा के समीपवर्ती भागों में तापमान 25° सेल्सियस पाया जाता है जो घटते-घटते 60° उत्तरी एवं दक्षिणी अक्षांशों के समीपवर्ती भागों में हिमांक बिन्दु के समीप पहुँच जाता है। दक्षिणी प्रशान्त महासागर में स्थलखण्ड की कमी के कारण समताप रेखाएँ लगभग पूर्व-पश्चिम दिशा में ही विस्तृत मिलती हैं।

प्रश्न 5. महासागरीय उच्चावच की दो आकृतियों-मध्य महासागरीय कटक एवं समुद्री टीला का वर्णन कीजिए।
उत्तर-1. मध्य महासागरीय कटक-मध्य महासागरीय कटक पर्वतों की दो श्रृंखलाओं से बनी आकृति है जो एक विशाल अवनमन द्वारा अलग होती है। इन पर्वत श्रृंखलाओं के शिखर की ऊँचाई 2,500 मीटर तक हो सकती है। किन्तु इनमें से कुछ समुद्र की सतह तक भी पहुँच जाती हैं; जैसे—आइसलैण्ड, जो मध्य अटलाण्टिक कटक का एक भाग है।

2. समुद्री टीला-ये नुकीले शिखरों वाले सागरीय पर्वत हैं। ये पर्वत या टीले महासागरीय सतह तक | नहीं पहुँच पाते हैं। इनकी उत्पत्ति ज्वालामुखी द्वारा होती है। इनकी ऊँचाई प्रायः 3,000 से 4,500 मीटर के आसपास होती है।

प्रश्न 6. प्रशान्त महासागर के उच्चावच की तुलना हिन्द महासागर के उच्चावच से कीजिए।
उत्तर- प्रशान्त महासागर एवं हिन्द महासागर के उच्चावच की तुलना
UP Board Solutions for Class 11 Geography Fundamentals of Physical Geography Chapter 13 Water (महासागरीय जल) img 3

प्रश्न 7, थर्मोक्लाइन तथा होलोक्लाइन में अन्तर बताइए।
उत्तर-थर्मोक्लाइन-थर्मोक्लाइन होलोक्लाइन के नीचे होती है। यहाँ लवणता की मात्रा बहुत कम पाई जाती है। इसकी मात्रा 34.6 तथा 34.9 प्रतिशत तक होती है। इसी को थर्मोक्लाइन क्षेत्र कहा जाता है।
होलोक्लाइन-इसकी स्थिति ऊपरी सतह पर उथले धरातल पर होती है। यहाँ उच्च लवणता पाई जाती है। इसके बाद लवणता कम होती जाती है।

प्रश्न 8. महासागरीय जल के तापमान के क्षैतिज वितरण का वर्णन कीजिए।
उत्तर-महासागरीय जल के तापमान का क्षैतिज वितरण
महासागरीय जल के तापमान का क्षैतिज वितरण पर भूमध्य-रेखा का विशेष प्रभाव पड़ता है प्रायः विषुवत् रेखा से ध्रुवों की ओर प्रत्येक अक्षांश पर औसत रूप से 1/2°C ताप कम हो जाता है, परन्तु दक्षिणी गोलार्द्ध में महासागरीय जल का तापमान उत्तरी गोलार्द्ध की अपेक्षा कम पाया जाता है। महासागरीय जल के तापमान का क्षैतिज वितरण मानचित्रों में समताप रेखाओं (Isotherms) द्वारा प्रदर्शित किया जा सकता है। हिन्द महासागर में 15°C की समताप रेखा अधिकतम तापमान के प्रदेशों को घेरती है (चित्र 13.3)। महासागरों के उत्तर-पश्चिमी भागों में समताप रेखाएँ देशान्तर रेखाओं के लगभग समान्तर हैं। मध्यवर्ती अन्ध महासागर में समताप रेखाएँ बड़ी ही असंयमित हैं, क्योंकि यहाँ मौसम की दशाएँ अनिश्चित रहती हैं। भूमध्यसागर में अन्ध महासागर की अपेक्षा तापमान उच्च रहता है। इसके विपरीत बाल्टिक सागर एवं हडसन नदी की खाड़ी में तापमान कम रहता है। कैरिबियन सागर में तापमान उच्च रहता है, क्योंकि व्यापारिक पवनें इस सागर की ओर चलती हैं।
UP Board Solutions for Class 11 Geography Fundamentals of Physical Geography Chapter 13 Water (महासागरीय जल) img 4
प्रशान्त महासागर में समताप रेखाएँ प्रायः अक्षांश रेखाओं के समानान्तर मिलती हैं। किन्तु दक्षिणी प्रशान्त महासागर में स्थलखण्ड की कमी के कारण समस्राप रेखाएँ लगभग पूर्व-पश्चिम दिशा में ही विस्तृत मिलती हैं। ॥

दीर्घ उत्तरीय प्रश्न

प्रश्न 1. महासागरों के तल के विन्यास का वर्णन कीजिए।
या महासागरों के सामान्य तलीय उच्चावच का वर्णन कीजिए।
या महाद्वीपीय मग्ल ढाल क्या है?
उत्तर-महासागरीय तल-पृथ्वीतल के 70.8% भाग पर जल का विस्तार मिलती है। जल का यह भण्डार स्थिर है। लगभग 29.2% भाग पर स्थलमण्डल का विस्तार पाया जाता है। यदि सागरों एवं महासागरों के सम्पूर्ण जल को स्थल पर फैला दिया जाए तो पृथ्वीतल पर तीन किमी गहरा सागर हिलोरें लेने लगेगा। इस प्रकार उत्तरी गोलार्द्ध में थल भाग (75%) की अधिकता के कारण उसे स्थल गोलार्द्ध एवं दक्षिणी गोलार्द्ध में जल के आधिक्य (90%) के कारण उसे जल गोलार्द्ध कहा जाता है।

पृथ्वीतल पर यह जल महासागरों, सागरों, खाड़ियों एवं झीलों आदि से मिलता है। प्रशान्त, अन्ध, हिन्द, आर्कटिक एवं अण्टार्कटिका–पाँच महासागर तथा भूमध्य, उत्तरी मलय, कैलीफोर्निया, लाल तथा अण्डमान आदि प्रमुख सागर हैं। फारस, हड़सन, मैक्सिको तथा बंगाल की खाड़ियाँ महत्त्वपूर्ण स्थान रखती हैं। झीलें सागरीय तटों के समीप तथा महाद्वीपों के आन्तरिक भागों में स्थित हैं। सुपीरियर, मिशीगन, घूरन, इरी, ओण्टेरिया, विक्टोरिया, बाल्कश, मानसरोवर आदि मुख्य झीलें हैं। महासागरों की औसत गहराई 3,800 मीटर है।

आधुनिक वैज्ञानिक युग में यन्त्रों, उपकरणों एवं गोताखोरों द्वारा सागरों एवं महासागरों की तली के उच्चावचों के विषय में पर्याप्त जानकारी प्राप्त हुई है। अब तो महासागरों की तली के मानचित्र भी बना लिये गये हैं। पश्चिमी प्रशान्त महासागर की गहराई सबसे अधिक अर्थात् 11.9 किमी है।

महासागरीय तली का विन्यास–महासागरीय तली के विन्यास को जानने के लिए निम्नलिखित बातों का ज्ञान होना अति आवश्यक है
(अ) सागरतल की गहराई एवं (ब) उस स्थान पर जलयान की स्थिति।
सागरतलों की जानकारी के लिए वैज्ञानिकों ने ध्वनि-तरंगों की प्रतिध्वनि विधि को खोज निकाला है। सागरों की गहराई जलयानों में लगे स्वचालित यन्त्रों द्वारा एक ग्राफ पर स्वयं ही अंकित होती रहती है। इस प्रक्रिया में जलयान के निचले भाग द्वारा जल में ध्वनि-तंरगें उत्पन्न की जाती हैं, जो सागरों की तली से टकराकर वापस लौटती हैं। इससे पता चलता है कि सागरीय तल सपाट नहीं है। इसमें बहुत-से पर्वत, पहाड़ियाँ, खाइयाँ एवं समतल मैदान मिलते हैं। ये खाइयाँ इतनी गहरी होती हैं कि इसमें विश्व के सबसे ऊँचे पर्वत ‘एवरेस्ट’ की चोटी भी समा सकती है। सागरों एवं महासागरों में विभिन्न स्थलाकृतियाँ देखने को मिलती हैं, जिनका विवरण निम्नवत् है

1. महाद्वीपीय मग्न तट-सागरों एवं महासागरों में अथाह जलराशि होती है जिससे यह आस-पास के तटीय भागों में फैल जाती है। अत: महाद्वीपों या स्थलों के वे भाग जो जलमग्न होते हैं, महाद्वीपीय मग्न तट कहलाते हैं। इन भागों में जल छिछला होता है तथा गहराई भी 200 फैदम तक होती है। इनका ढाल स्थल से सागर की ओर होता है।

महाद्वीपीय मग्न तट की तली सभी भागों में समान नहीं होती, इनमें गड्ढे, टीले, घाटियाँ आदि पाये जाते हैं। कहीं-कहीं पर इनका तल कठोर शैलों द्वारा निर्मित होता है। कुछ भागों में बालू एवं कीचड़ के जमाव भी मिलते हैं। इनमें कुछ भाग ऊपर उठ जाते हैं, जो सागरीय जल के द्वीप के समान दिखाई पड़ते हैं। महाद्वीपीय मग्न तट कहीं पर ऊँचे उठ रहे हैं और कहीं पर नीचे धंस रहे हैं। इन पर अपरदन कारकों द्वारा अवसादों का निर्माण होता रहता है। सूर्य के प्रकाश के कारण महाद्वीपीय मग्न तट पर वनस्पति तथा जन्तु जीवित रहते हैं। ये क्षेत्र महत्त्वपूर्ण मत्स्य उत्पादक क्षेत्रों के रूप में विकसित हो गये हैं।

2. महाद्वीपीय मग्न ढाल-महाद्वीपीय मग्न् तट के किनारे पर जब ढाल अचानक ही तेज हो जाता है। तो उसे महाद्वीपीय मग्न ढाल कहते हैं। यह ढाल 35 से 61 मीटर प्रति किमी होता है। इसका एक सिरा मग्न तट से जुड़ा होता है तथा दूसरा सिरा समुद्री फर्श से मिल जाता है।
UP Board Solutions for Class 11 Geography Fundamentals of Physical Geography Chapter 13 Water (महासागरीय जल) img 5

3. गहरे सागरीय बेसिन-सागरों एवं महासागरों का 2/3 भाग गहरे बेसिन या फर्श द्वारा निर्मित है। इसकी लम्बाई 37 से 43 किमी तक होती है। यहाँ लम्बी पहाड़ियाँ, पठार, ज्वालामुखी, पर्वत शिखर आदि स्थलाकृतियाँ पायी जाती हैं। सागरीय जल में ये पहाड़ियाँ द्वीप की भाँति दिखाई देती हैं। इस प्रकार की स्थलाकृतियाँ प्रशान्त महासागर में देखने को मिलती हैं।

4. सागरीय गर्त-सागरीय तली में स्थित लम्बे, सँकरे एवं गहरे स्थल-स्वरूप को सागरीय गर्त कहते हैं। प्रशान्त महासागर एवं कैरेबियन सागर में यह गर्त अधिक पाये जाते हैं। इनकी गहराई 7 से 9 किमी तक होती है। पर्वत-निर्माणकारी घटनाओं द्वारा इन सागरीय गतें की उत्पत्ति होती है।

5. अन्तःसागरीय गम्भीर खड्ड-महाद्वीपीय मग्न तट और मग्न ढालों पर ‘वी’-आकार के तीव्र ढाल वाली दीवारों के साथ बने गड्ढों को अन्त:सागरीय गम्भीर खड्ड कहते हैं। सागरों में ये खड्ड नदियों के मुहानों के पास होते हैं। इनकी गहराई 2 से 3 किमी तक होती है।

6. सागरीय पर्वत-सागरीय फर्श पर ऊँची परन्तु शीर्षयुक्त जलमग्न स्थलाकृति को सागरीय पर्वत कहते हैं। इनका आकार शंकु की भाँति होता है। अलास्का खाड़ी में इस प्रकार के अनेक पर्वत देखे जा सकते हैं।

7. सागरीय कटक-सागरीय भागों में फैली लम्बी एवं सँकरे आकार की जलमग्न पर्वत-श्रेणियाँ सागरीय कटक कहलाती हैं। अन्ध महासागर में इस प्रकार की अनेक़ स्थलाकृतियाँ मिलती हैं। प्रशान्त महासागर में ये कटक नहीं मिलतीं। हिन्द महासागर में इनका विस्तार उत्तर-दक्षिण दिशा में है।

प्रश्न 2. महासागरों में लवणता के असमान वितरण का वर्णन कीजिए तथा उसके कारणों की विवेचना कीजिए।
या महासागरीय लवणता से आप क्या समझते हैं? उसके वितरण को प्रभावित करने वाले चार कारकों की व्याख्या कीजिए।
उत्तर-महासागरीय जल की लवणता–सम्पूर्ण ग्लोब के 70.8% भाग पर जलमण्डल का विस्तार है। परन्तु सागरों एवं महासागरों का यह जल पीने-योग्य नहीं होता, क्योंकि इसमें अनेक लवणों का मिश्रण रहता है। सागरीय जल में लवणों की उपस्थिति से उत्पन्न खारेपन को महासागरीय जल की लवणता कहा जाता है। खारेपन की यह मात्रा उन सभी खनिजों से मिलती है जो इनके जल में स्वतन्त्र रूप से एक निश्चित अनुपात में मिलते रहते हैं। भिन्न-भिन्न सागरों एवं महासागरों में लवणता की मात्रा में भिन्नता पायी जाती है। यह लवणता प्रति 1000 ग्राम जल में घुले हुए नमक द्वारा प्रकट की जाती है। उदाहरण के लिए, यदि 1,000 ग्राम जल में 21 ग्राम नमक है तो इस जल की लवणता 21 प्रति सहस्र होगी।
UP Board Solutions for Class 11 Geography Fundamentals of Physical Geography Chapter 13 Water (महासागरीय जल) img 6
भूमि पर प्रवाहित होता हुआ जल अर्थात् नदियाँ प्रतिवर्ष 16 करोड़ टन खनिज पदार्थ बहाकर सागरों एवं महासागरों के गर्भ में जमा करती हैं। इस जल में कार्बोनेट, सोडियम तथा सिलिकेट आदि लवणों की अधिकता होती है। इस जल में 35 ग्राम नमक प्रति 1,000 ग्राम होता है। सागरों एवं महासागरों के जल का । खारापन अधिक होता है, क्योंकि इनके जल का मैग्नीशियम सल्फेट वाष्पीकरण होता रहता है जिससे इनमें नमक की मात्रा की वृद्धि होती रहती है। सागरीय जल में सल्फेट तथा क्लोराइड आदि लवण अधिक मिलते हैं। अतः इस जल की लवणता का मूल कारण नदियों का जल, जल का – वाष्पीकरण अधिक मात्रा में होना, समुद्री जल-जीव एवं रासायनिक क्रियाओं को होना है।

लवणता की रचना—यह अनुमान लगाया गया है कि सागरों एवं महासागरों के जल में लवणता की मात्रा 50 लाख अरब टन है। सामान्य रूप से प्रति 1,000 ग्राम जल में लवणता की औसत मात्रा 35 ग्राम है; अर्थात् 3.5 प्रतिशत नमक है। इन लवणों में सोडियम क्लोराइड सबसे अधिक होता है। प्रति 1,000 ग्राम सागरीय जल में विभिन्न लवणों की मात्रा संलग्न तालिका के अनुसार है।।

सागरीय जल में लवणों का अनुपात सभी स्थानों पर एकजैसा मिलता है, परन्तु उनकी मात्रा में परिवर्तन हो सकता है। इसका प्रमुख कारण सागरीय जलधाराओं का एक स्थान से दूसरे स्थान पर प्रवाहित होते रहना है। इसी कारण यह अनुपात सदैव स्थिर रहता है।

सागरीय जल की लवणता में भिन्नता के कारण ।

सागरों एवं महासागरों के जल की लवणता में भिन्नता के निम्नलिखित कारण हैं
1. स्वच्छ जल की पूर्ति–जलाशयों में स्वच्छ जल की पूर्ति लवणता की मात्रा को कम कर देती है। उदाहरण के लिए, विषुवत् रेखा के समीपवर्ती भागों में स्वच्छ जले की पूर्ति के कारण सागरीय लवणता कम पायी जाती है। इसके विपरीत उपोष्ण तथा शीतोष्ण कटिबन्धीय भागों के सागरों तथा महासागरों में स्वच्छ जल की कमी के कारण लवणता अधिक पायी जाती है। इसी कारण भूमध्यसागरीय जल में लवणता की मात्रा अधिक पायी जाती है।

2. वाष्पीकरण-वाष्पीकरण क्रिया में जल का बहुत-सा भाग वाष्प बनकर वायुमण्डल में मिल जाता है। इससे सागरीय जल की लवणता में वृद्धि हो जाती है। वाष्पीकरण की अधिकता उच्च ताप, शुष्क वायु, वायु की तेज गति एवं आकाश की स्वच्छता पर निर्भर करती है। उष्ण कटिबन्ध में इस प्रकार की दशाएँ पायी जाती हैं, जिससे इन प्रदेशों में स्थित सागरों में लवणता की मात्रा भी। अधिक मिलती है। इसके विपरीत ध्रुवीय प्रदेशों में निम्न तापमान एवं वाष्पीकरण की कैमी के कारण लवणता कम पायी जाती है।

3. पवनों की प्रकृति-पवनों की तीव्रता एवं शुष्कता जल के अधिक वाष्पीकरण में सहायक होती है; अतः ऐसे क्षेत्रों में सागरीय लवणता भी अधिक मिलती है। यही कारण है कि कर्क एवं मकर रेखाओं के समीपवर्ती सागरीय भागों में लवणता की अधिकता पायी जाती है।

4. सागरीय धाराएँ-समुद्र-तल की ऊपरी सतह में नीचे की सतह की अपेक्षा अधिक लवणता होती है। सागरों में जो धाराएँ प्रवाहित होती हैं, वे ऊपरी सतह के जल को बहा ले जाती हैं, जिससे उस स्थान की लवणता कम हो जाती है। ऊपरी सतह का यह जल जिन भागों में पहुँचता है, वहाँ सागरीय जल की लवणता में वृद्धि कर देता है।

5. जल-जीवों की उपस्थिति-महासागरीय जीव भी लवणता को प्रभावित करते हैं। जिन सागरीय भागों में स्वच्छ एवं मृदु जल होता है, उसमें सिलिको एवं कैल्सियम कार्बोनेट की अधिकता होती है, परन्तु इस जल में उत्पन्न इन तत्त्वों का शोषण जल-जीवों द्वारा कर लिया जाता है, जिससे सागरीय जल की लवणता में वृद्धि हो जाती है।

लवणता का वितरण ।

यदि हम ग्लोब पर स्थित जलाशयों का अध्ययन करें तो पता चलता है कि सबसे कम सागरीय खारापन ध्रुवीय प्रदेशों में मिलता है। इसके विपरीत संबसे अधिक खारापन कर्क एवं मकर रेखाओं के निकटवर्ती सागरीय भागों में पाया जाता है। इसका मुख्य कारण उच्च ताप, कम वर्षा, स्वच्छ आकाश, गर्म शुष्क एवं तीव्र वायु प्रवाह है। सागरीय लवणता का वितरण निम्नलिखित है

1. महासागरीय लवणता-कर्क एवं मकर रेखाओं के समीपवर्ती भागों में लवणता की मात्रा सबसे अधिक अर्थात् 3.6 प्रतिशत है। इनसे आगे ध्रुवों की ओर लवणता की मात्रा कम होती जाती है। भूमध्य रेखा पर लवणता की मात्रा 3.4 प्रतिशत है, जिसका कारण स्वच्छ जल की प्राप्ति का होना है। ध्रुवीय प्रदेशों में लवणता की मात्रा 3.0 प्रतिशत रह जाती है या इससे भी कम मिलती है, जिसका प्रमुख कारण ताप में कमी, वाष्पीकरण का कम होना तथा हिम द्वारा शुद्ध जल की प्राप्ति का होते रहना है।

सागरों में सबसे अधिक लवणता सारगैसो सागर (उत्तरी अटलांटिक महासागर) में है, जहाँ पर इसकी मात्रा 3.8 प्रतिशत है। इसका कारण उच्च ताप, कम वर्षा, आकाश की स्वच्छता, उष्ण एवं शुष्क पवनों का प्रवाहित होना तथा सूर्य की किरणों की तीव्रता का होना है। वाष्पीकरण की तीव्रता सागरीय जल के खारेपन में वृद्धि करती रहती है।।

2. सागरीय लवणता-सागरीय लवणता महासागरों से भिन्न होती है। इन सागरों का सम्बन्ध खाड़ियों तथा जलडमरूमध्य द्वारा महासागरों में होता है। भूमध्ये सागर में सबसे अधिक लवणता 3.9% है। स्वेज नहर के समीप यह मात्रा बढ़कर 4.1% हो जाती है। फारस की खाड़ी में 4.8% लवणता की मात्रा मिलती है। इसका मुख्य कारण वर्षा का अभाव, स्वच्छ जल की कमी, उच्च ताप एवं वाष्पीकरण की तीव्रता का होना है। काला सागर में लवणता की मात्रा 1.8% है। उत्तरी ध्रुव के निकटवर्ती भागों में लवणता और भी कम हो जाती है; जैसे—-बाल्टिक सागर में 1.5%,
बोथानिया की खाड़ी में 0.8% तथा फिनलैएड की खाड़ी में केवल 0.2% रह जाती है।

3. आन्तरिक जलाशयों में लवणता-इस वर्ग में आन्तरिक सागर एवं झीलें सम्मिलित हैं। विश्व में लवणता की सबसे अधिक मात्रा जोर्डन के समीप मृत सागर में 23.8% है। इसका प्रमुख कारण उच्च तापमान, अत्यधिक वाष्पीकरण तथा शुष्क एवं उष्ण पवनों का प्रवाहित होना है। कैस्पियन,सागर के दक्षिणी भाग में काराबुगा खाड़ी में लवणता 17.0% तथा उत्तरी भाग में केवल 1.4% है। इसका प्रमुख कारण कैस्पियन सागर के उत्तरी भाग में यूराल तथा वोल्गा नदियों द्वारा स्वच्छ जल की पूर्ति करते रहना है। झीलों में सर्वाधिक लवणता की मात्रा तुर्की की वान झील में 33.0% है। उत्तरी अमेरिका महाद्वीप की महान झीलों में भी लवणता की मात्रा अधिक मिलती है, जहाँ पर सुपीरियर झील में यह मात्रा 22.0% है।

इस प्रकार उपर्युक्त विवरण से स्पष्ट है कि उच्च ताप, वर्षा का अभाव, अत्यधिक वाष्पीकरण, उष्ण एवं शुष्क पवनों का प्रवाह, स्वच्छ जल की आपूर्ति का पूर्ण अभाव तथा स्वच्छ एवं स्पष्ट आकाश आदि तथ्य सागरीय लवणता को प्रभावित करते हैं।

प्रश्न 3. महासागरीय जल के तापमान के लम्बवत वितरण की विशेषताओं की विवेचना कीजिए।
उत्तर- महासागरीय जल के तापमान का लम्बवत् वितरण
महासागरीय जल के तापमान का प्रमुख स्रोत सूर्य है। इसके अतिरिक्त भूगर्भ का ताप, जल को आपसी दबाव भी ताप प्रदान करते हैं। वायुमण्डल की भाँति जलमण्डल में गति के कारण ताप के वितरण में भिन्नता मिलती है। महासागरीय ताप वितरण की विशेषताओं को विवरण निम्नलिखित है–
UP Board Solutions for Class 11 Geography Fundamentals of Physical Geography Chapter 13 Water (महासागरीय जल) img 7
महासागरीय जल अधिकतम ताप सूर्य से प्राप्त करता है जिस कारण सागरों की ऊपरी परत का जल सर्वाधिक ताप ग्रहण करता है। और गहराई के साथ जल में ताप की उपस्थिति कम होती जाती है, परन्तु तापमान की ह्रास दर सभी गहराइयों पर एक-सी है नहीं होती है। प्राय: 2,000 मीटर की गहराई तक तापमान तेजी से घटता है। 180 मीटर की गहराई का 16° सेल्सियस तापमान है। 2,000 मीटर की गहराई पर घटकर केवल 2° सेल्सियस रह जाता है, परन्तु 4,000 मीटर की गहराई तक केवल 0.4° सेल्सियस ही घटता है तथा वहाँ 1.6° सेल्सियस ताप पाया जाता है। ऐसा अनुमान है कि आयतन की दृष्टि से लगभग 85% महासागरीय जल का तापमान 2 से 4° सेल्सियस के मध्य रहता है (चित्र 13.5)।।

महासागरों में तोप के लम्बवत् वितरण पर जलमग्न अवरोधों का बड़ा प्रभाव पड़ता है। ये अवरोध महासागरों के ताप में विभिन्नताएँ पैदा करते हैं। उदाहरण के लिए लाल सागर में 2,100 मीटर की गहराई पर भी 21° सेल्सियस ताप पाया जाता है, जबकि हिन्द महासागर में इस गहराई पर केवल 2° सेल्सियस ताप पाया जाता है। वस्तुत: जलमग्न अवरोधों के कारण ही महासागरीय जल के लम्बवत् ताप वितरण में अन्तर पाया जाता है; क्योंकि ये अवरोध जल का मिश्रण नहीं होने देते हैं।

We hope the UP Board Solutions for Class 11 Geography: Fundamentals of Physical Geography Chapter 13 Water (महासागरीय जल) help you. If you have any query regarding UP Board Solutions for Class 11 Geography: Fundamentals of Physical Geography Chapter 13 Water (महासागरीय जल), drop a comment below and we will get back to you at the earliest.